You are on page 1of 120

(001~042)개념RPM3-2(정답) 2014.11.

24 02:53 PM 페이지1 다민 2540DPI 175LPI

이홍섭 지음

다양한 유형의 우수한 문제를 통하여 수학의 문제해결력을 높일 수 있는

개념원리수학 익힘책 [알피엠]

중 수학
3 -2
정답과 풀이
01 대푯값과 산포도 02/ 02 피타고라스 정리 14/

03 피타고라스 정리의 활용 26/ 04 삼각비 42/

05 삼각비의 활용 55/ 06 원과 직선 66/

07 원주각 79/ 08 원주각의 활용 92/

내신만점 테스트 104


(001~042)개념RPM3-2(정답) 2014.11.24 02:53 PM 페이지2 다민 2540DPI 175LPI

Ⅰ 통계

01 대푯값과산포도 0012 주어진 자료는 크기순으로 나열되어 있고, 전체 자료


의 중앙에 있는 값은 19이므로 중앙값은 19시간이다.
또, 가장 많이 나타난 값이 22이므로 최빈값은 22시간이다.
8+5+4+10+7+2
0001 (평균)=
6
중앙값:19시간, 최빈값:22시간

36
= =6 6 5+6+7+8+9 35
6
0013 (평균)=
5
=
5
=7이므로 편차는 차

80+85+95+93+77+86 례로 -2, -1, 0, 1, 2이다. -2, -1, 0, 1, 2


0002 (평균)=
6
516 10+16+17+13+14 70
=
6
=86 86 0014 (평균)=
5
=
5
=14이므로 편

차는 차례로 -4, 2, 3, -1, 0이다. -4, 2, 3, -1, 0


18+20+21+22+24+26+24+21
0003 (평균)=
8
85+77+94+72+97 425
=
176
=22 22
0015 (평균)=
5
=
5
=85이므로
8
편차는 차례로 0, -8, 9, -13, 12이다.
0, -8, 9, -13, 12
0004 자료를 작은 값부터 크기순으로 나열하면
80, 80, 90, 100, 130
19+23+34+27+31+22 156
이므로 중앙값은
5+1
=3번째 자료의 값인 90이다. 90 0016 (평균)=
6
=
6
=26이
2
므로 편차는 차례로 -7, -3, 8, 1, 5, -4이다.
0005 자료를 작은 값부터 크기순으로 나열하면 -7, -3, 8, 1, 5, -4
3, 4, 5, 7, 8, 9
2+4+6+8+10 30
이므로 중앙값은
6
=3번째와
6
+1=4번째 자료의 값인 5 0017 (평균)=
5
=
5
=6이므로
2 2
와 7의 평균인 (-4)¤ +(-2)¤ +0¤ +2¤ +4¤ 40
(분산)= = =8
5 5
5+7
=6 6 (표준편차)='8=2'2 분산:8, 표준편차:2'2
2

0006 자료를 작은 값부터 크기순으로 나열하면 1+5+6+6+7 25


0018 (평균)=
5
=
5
=5이므로
1, 3, 4, 5, 6, 7, 9, 10
8 8 (-4)¤ +0¤ +1¤ +1¤ +2¤ 22
이므로 중앙값은 =4번째와 +1=5번째 자료의 값인 5 (분산)= = =4.4
2 2 5 5
와 6의 평균인 (표준편차)='∂4.4 분산:4.4, 표준편차:'∂4.4
5+6
=5.5 5.5
2 8+9+6+15+12 50
0019 (평균)=
5
=
5
=10이므로

0007 자료를 작은 값부터 크기순으로 나열하면


(분산)=
(-2)¤ +(-1)¤ +(-4)¤ +5¤ +2¤
=
50
=10
68, 69, 76, 83, 87, 95, 97 5 5
7+1 (표준편차)='ß10 분산:10, 표준편차:'ß10
이므로 중앙값은 =4번째 자료의 값인 83이다. 83
2
7+10+11+15+8+9 60
0008 3 0020 (평균)=
6
=
6
=10이므로

(-3)¤ +0¤ +1¤ +5¤ +(-2)¤ +(-1)¤


(분산)=
0009 없다. 6
40 20
= =
0010 2 6 3
20 2'ß15
(표준편차)=Ƭ =
0011 9, 10 3 3

2 정답과 풀이
(001~042)개념RPM3-2(정답) 2014.11.24 02:53 PM 페이지3 다민 2540DPI 175LPI

● 본문 009쪽 ~ 010쪽 ●

20 2'ß15 11+15
분산: , 표준편차: =13(시간)
3 3 2
∴ b=13(시간)
4+2+7+5+7+4+9+2 40 ∴ a+b=25+13=38 38
0021 (평균)=
8
=
8
=5이므로

(분산)
0027 자료를 작은 값부터 크기순으로 나열하면
(-1)¤ +(-3)¤ +2¤ +0¤ +2¤ +(-1)¤ +4¤ +(-3)¤ 1, 1, 3, 4, 4, 5, 5, 6, 7, 8, 8, 8
=
8
1_2+3+4_2+5_2+6+7+8_3
44 ∴ a=
= =5.5 12
8
60
(표준편차)='∂5.5 분산:5.5, 표준편차:'∂5.5 = =5(회)
12
또한 중앙값은 6번째와 7번째 자료의 값인 5와 5의 평균이므로
78+84+96+76+90+86+74+72
0022 (평균)=
8 b=
5+5
=5(회)
2
656 ∴ a+b=5+5=10 10
= =82
8
이므로
(분산) 0028 평균이 25회이므로
12+14+21+(20+a)+(20+a)+29+30+31+31+32
(-4)¤ +2¤ +14¤ +(-6)¤ +8¤ +4¤ +(-8)¤ +(-10)¤ =25
= 10
8
240+2a
496 =25, 240+2a=250
= =62 10
8
∴ a=5
(표준편차)='ß62 분산:62, 표준편차:'ß62
이때 경기 출전 횟수는 크기순으로 나열되어 있으므로 중앙값
은 5번째와 6번째 자료의 값인 25와 29의 평균인
0023 도수분포표를 완성하면 다음과 같다.
25+29
=27(회) 27회
기록(초) 도수(명) (계급값)_(도수) (편차)¤ _(도수) 2
5이상~15미만 2 10_2=20 (-18)¤ _2=648
15이상~25미만 5 20_5=100 (-8)¤ _5=320 0029 나머지 세 수를 p, q, r(단, p…q…r)라고 할 때, 중
이상 미만
25 ~35 8 30_8=240 2¤ _8=32 앙값이 가장 큰 경우 9개의 정수를 작은 수부터 크기순으로 나
35이상~45미만 5 40_5=200 12¤ _5=720
열하면 다음과 같다.
합계 20 560 1720
2, 4, 4, 7, 8, 9, p, q, r
560 따라서 중앙값은 5번째 수이므로 중앙값이 될 수 있는 가장 큰
∴ (평균)= =28(초) 28초
20
수는 8이다. 8

0024 A=5, B=32, C=1720


0030 볼링공에 적힌 수를 작은 수부터 크기순으로 나열하면
7, 8, 8, 9, 10, 11, 11, 11, 12, 13
1720
0025 (분산)=
20
=86 이므로
7+8_2+9+10+11_3+12+13
(표준편차)='ß86초 분산:86, 표준편차:'ß86초 m=
10
100
= =10(파운드)
0026 A조의 자료를 작은 값부터 크기순으로 나열하면 10
10, 23, 25, 32, 47 또한 중앙값은 5번째와 6번째의 자료의 값인 10과 11의 평균
이므로 중앙값은 3번째 자료의 값인 25시간이다. 이므로
∴ a=25(시간) 10+11
a= =10.5(파운드)
B조의 자료를 작은 값부터 크기순으로 나열하면 2
8, 9, 11, 15, 20, 24 한편 최빈값은 자료의 값 중 가장 많이 나타난 값이므로
이므로 중앙값은 3번째와 4번째 자료의 값인 11과 15의 평 b=11(파운드)
균인 ∴ m+a+b=10+10.5+11=31.5 31.5

01. 대푯값과 산포도 3


(001~042)개념RPM3-2(정답) 2014.11.24 02:53 PM 페이지4 다민 2540DPI 175LPI

RPM 알피엠

0031 바둑 급수를 나타내는 수를 작은 수부터 크기순으로 또한 중앙값은 10번째와 11번째 자료의 값인 8과 8의 평균이
나열하면 므로
3, 4, 7, 7, 8, 8, 8, 9, 9 8+8
b= =8
따라서 최빈값은 가장 많이 나타난 8급이므로 최빈값에 해당 2
하는 학생은 창훈, 진수, 태연이다. 창훈, 진수, 태연 한편 최빈값은 도수가 가장 큰 변량이므로 c=7
∴ a=b>c ①

0032 줄기와 잎 그림에서 주어진 자료는 다음과 같다.


5, 7, 10, 11, 16, 16, 20, 23, 25, 34 0035 전체 학생 수는
5+7+10+11+16+16+20+23+25+34 3+5+12+6+4=30(명)
a=
10 3_3+5_5+7_12+9_6+11_4
(평균)=
167 30
= =16.7(시간)
10 216
= =7.2(권)
중앙값은 5번째와 6번째의 자료의 값인 16과 16의 평균이므로 30
16+16 또한 중앙값은 15번째, 16번째 값이 속해 있는 6권 이상 8권
b= =16(시간)
2 6+8
미만인 계급의 계급값이므로 =7(권)이다.
한편 최빈값은 자료의 값 중 가장 많이 나타난 값이므로 2
c=16(시간) 한편 최빈값은 도수가 가장 큰 계급인 6권 이상 8권 미만의 계
∴ a+b+c=16.7+16+16=48.7 48.7 6+8
급값이므로 =7(권)
2
평균:7.2권, 중앙값:7권, 최빈값:7권
0033 평균이 6이므로
3_2+5_2+7_2+8_2+9+a
=6
10 0036 도수의 총합이 20이므로
55+a 2+a+b+4+3=20 ∴ a+b=11 yy ㉠
=6, 55+a=60 ∴ a=5 yy`
10
또한 평균이 4시간이므로
이때 자료를 작은 값부터 크기순으로 나열하면 2_2+3_a+4_b+5_4+6_3
=4
3, 3, 5, 5, 5, 7, 7, 8, 8, 9 20
이므로 중앙값은 5번째와 6번째 자료의 값인 5와 7의 평균이 3a+4b+42
=4, 3a+4b+42=80
20
므로
∴ 3a+4b=38 yy ㉡
5+7
=6 yy` ㉠, ㉡을 연립하여 풀면
2
a=6, b=5 yy`
또한 최빈값은 자료의 값 중 가장 많이 나타난 값이므로 5이다.
이때 중앙값은 10번째와 11번째 자료의 값인 4와 4의 평균인
yy`
4+4
따라서 중앙값과 최빈값의 합은 =4(시간) yy`
2
6+5=11 yy`
또한 최빈값은 도수가 가장 큰 변량이므로 3시간이다. yy`
11 따라서 중앙값과 최빈값의 합은
단계 채점요소 배점 4+3=7(시간) yy`
a의 값 구하기 40% 7시간
중앙값 구하기 20%
단계 채점요소 배점
최빈값 구하기 20%
a, b의 값 구하기 40%
중앙값과 최빈값의 합 구하기 20%
중앙값 구하기 20%
최빈값 구하기 20%

0034 도수의 총합이 20이므로 중앙값과 최빈값의 합 구하기 20%

2+6+5+x+3=20 ∴ x=4
6_2+7_6+8_5+9_4+10_3
a=
20 0037 중앙값이 14이므로 13…a…27
자료를 작은 값부터 크기순으로 나열하면
160
= =8 9, 13, a, 27
20

4 정답과 풀이
(001~042)개념RPM3-2(정답) 2014.11.24 02:53 PM 페이지5 다민 2540DPI 175LPI

● 본문 010쪽 ~ 013쪽 ●

이고 중앙값은 2번째와 3번째의 자료의 값인 13과 a의 평균이 0043 편차의 합은 항상 0이므로


므로 -3-4+3+6+x=0 ∴ x=-2
13+a E의 수학 성적은 평균보다 2점이 낮으므로
=14, 13+a=28
2 76-2=74(점) ③
∴ a=15
27+9+13+15 64
∴ (평균)=
4
=
4
=16 16 0044 편차의 합은 항상 0이므로
-2-5+x+2+1=0 ∴ x=4
따라서 영어 성적의 분산은
0038 최빈값이 28이므로 b=28 (-2)¤ +(-5)¤ +4¤ +2¤ +1¤ 50
= =10 10
중앙값이 26이므로 6번째와 7번째의 자료의 값의 평균이 26이다. 5 5
a+28
=26, a+28=52
2
0045 ㄱ. 평균을 m점이라고 하면
∴ a=24
(B의 점수)=(m-1)점, (C의 점수)=(m+2)점
∴ b-a=28-24=4 4
따라서 B와 C의 점수의 차는 3점이다.
ㄴ. D의 편차가 0이므로 D의 점수는 평균과 같다.
0039 자료의 개수가 7개이므로 자료를 작은 값부터 크기순
ㄷ. (분산)=
(-2)¤ +(-1)¤ +2¤ +0¤ +1¤
=
10
=2
으로 나열할 때 4번째 자료의 값이 중앙값이 되고 a<b이므로 5 5
a=9 ∴ (표준편차)='2점
또한 평균이 11분이므로 ㄹ. 점수가 가장 낮은 학생은 편차가 가장 작은 A이다.
4+8+22+13+3+a+b 따라서 옳은 것은 ㄴ, ㄷ, ㄹ이다. ⑤
=11, 50+a+b=77
7
∴ a+b=27
0046 평균이 9이므로
따라서 a=9, b=18이므로 7+8+x+(x+1)+(x+2)
b-a=18-9=9 ⑤ =9 yy`
5
18+3x
=9, 18+3x=45 ∴ x=9
5
0040 평균이 10, 중앙값이 12이므로 따라서 각 변량은 7, 8, 9, 10, 11이므로 yy`
a+12+b (-2)¤ +(-1)¤ +0¤ +1¤ +2¤
=10, a+12+b=30 10
3 (분산)= = =2
5 5
∴ a+b=18 yy ㉠
∴ (표준편차)='2 yy`
한편 1…a<12, 12<b<18 yy ㉡
'2
㉠, ㉡을 모두 만족하는 a, b의 순서쌍 (a, b)는
단계 채점요소 배점
(1, 17), (2, 16), (3, 15), (4, 14), (5, 13)
평균을 이용하여 식 세우기 30%
이므로 b-a의 최댓값은 17-1=16이고 최솟값은 13-5=8
각 변량 구하기 30%
이다.
표준편차 구하기 40%
따라서 b-a의 최댓값과 최솟값의 합은
16+8=24 ④
0047 중앙값과 최빈값이 5이므로 a…b…c라고 하면
a=5, b=5
2¤ +1¤ +(-1)¤ +0¤ +(-2)¤
0041 (분산)=
5
또한 평균이 4이므로

10 1+3+5+5+c
= =2 =4
5 5

∴ (표준편차)='2점 ② 14+c=20 ∴ c=6


(-3)¤ +(-1)¤ +1¤ +1¤ +2¤ 16
∴ (분산)= = =3.2 ④
5 5
0042 ⑤ 편차의 절댓값이 작을수록 변량은 평균에 가깝다.
따라서 옳지 않은 것은 ⑤이다. ⑤ 0048 (전체 학생의 총점)=(250+200)_76=34200(점),

01. 대푯값과 산포도 5


(001~042)개념RPM3-2(정답) 2014.11.24 02:53 PM 페이지6 다민 2540DPI 175LPI

RPM 알피엠

(남학생의 총점)=250_72=18000(점)이므로 0053 변량 a, b, c, d의 평균이 6이고 표준편차가 5, 즉 분


(여학생의 총점)=34200-18000=16200(점) 산이 25이므로
16200 (a-6)¤ +(b-6)¤ +(c-6)¤ +(d-6)¤
∴ (여학생의 평균)= =81(점) ③ =25
200 4
∴ (a-6)¤ +(b-6)¤ +(c-6)¤ +(d-6)¤ =100 ③
0049 B분단의 평균이 80점이므로
70+85+x+90+95+65
=80 0054 평균이 2이므로
6
x+y
405+x=480 ∴ x=75 =2 ∴ x+y=4
2
또한 A, B 두 분단 전체의 평균이 76점이므로 또한 분산이 2이므로
80+70+85+70+65+y+80_6 (x-2)¤ +(y-2)¤
=76 =2
6+6 2
850+y=912 ∴ y=62 x¤ +y¤ -4(x+y)+8=4
∴ x-y=75-62=13 13 ∴ x¤ +y¤ =4(x+y)-4=4_4-4=12 ②

0050 남학생과 여학생의 평균이 같고 분산이 각각 5¤ , 7¤ 이


0055 편차의 합은 항상 0이므로
므로 (편차)¤ 의 총합은 각각
a-2+b+4-1=0
5¤ _20=500, 7¤ _20=980
∴ a+b=-1 yy ㉠ yy`
따라서 전체 학생 40명의 (편차)¤ 의 총합은
또한 분산이 6.8이므로
500+980=1480이므로
a¤ +(-2)¤ +b¤ +4¤ +(-1)¤
1480 =6.8
(분산)= =37 5
40
a¤ +b¤ +21=34, a¤ +b¤ =13 yy`
∴ (표준편차)='ß37점 'ß37점
∴ (a+b)¤ -2ab=13 yy ㉡
㉠을 ㉡에 대입하면
0051 A, B 두 조의 평균이 같고 분산이 각각 ('6)¤ , a¤ 이
(-1)¤ -2ab=13
므로 (편차)¤ 의 총합은 각각
∴ 2ab=-12 yy`
6_8=48, a¤ _12=12a¤
-12
따라서 전체 20명의 (편차)¤ 의 총합은
단계 채점요소 배점
48+12a¤
편차의 합이 0임을 이용하여 a+b의 값 구하기 30%
이때 분산은 3¤ =9이므로
분산의 정의를 이용하여 a¤ +b¤ 의 값 구하기 30%
48+12a¤
=9 2ab의 값 구하기 40%
20
48+12a¤ =180, a¤ =11
∴ a='ß11 (∵ a>0) 'ß11 0056 변량 x, y, z의 평균이 8이므로
x+y+z
=8 ∴ x+y+z=24
3
0052 평균이 5이므로
7+6+5+x+y 또한 변량 x, y, z의 분산이 4이므로
=5
5 (x-8)¤ +(y-8)¤ +(z-8)¤
=4
18+x+y=25 3

∴ x+y=7 ∴ (x-8)¤ +(y-8)¤ +(z-8)¤ =12


또한 분산이 2이므로 따라서 변량 x+4, y+4, z+4, 12에 대하여
2¤ +1¤ +0¤ +(x-5)¤ +(y-5)¤ (x+4)+(y+4)+(z+4)+12 x+y+z+24
=2 (평균)= =
5 4 4

5+(x-5)¤ +(y-5)¤ =10, x¤ +y¤ -10(x+y)=-45 24+24 48


= = =12
4 4
∴ x¤ +y¤ =10(x+y)-45=10_7-45=25
1
그런데 x¤ +y¤ =(x+y)¤ -2xy이므로 ∴ (분산)= {(x+4-12)¤ +(y+4-12)¤
4
25=7¤ -2xy ∴ xy=12 ④ +(z+4-12)¤ +(12-12)¤ }

6 정답과 풀이
(001~042)개념RPM3-2(정답) 2014.11.24 02:53 PM 페이지7 다민 2540DPI 175LPI

● 본문 013쪽 ~ 014쪽 ●

1 (a-10)¤ +(b-10)¤ +(c-10)¤ +(d-10)¤


∴ (분산)= {(x-8)¤ +(y-8)¤ +(z-8)¤ } =3
4 4
1 ∴ (a-10)¤ +(b-10)¤ +(c-10)¤ +(d-10)¤ =12
= _12=3 3
4 따라서 변량 2a-3, 2b-3, 2c-3, 2d-3에 대하여
(2a-3)+(2b-3)+(2c-3)+(2d-3)
(평균)=
0057 ⑴ 변량 a, b, c, d, e의 평균이 5이므로 4
a+b+c+d+e 2(a+b+c+d)-12
=5 =
5 4
∴ a+b+c+d+e=25 2_40-12 68
= = =17
4 4
또한 변량 a, b, c, d, e의 표준편차가 2, 즉 분산이 4이므로
1
(a-5)¤ +(b-5)¤ +(c-5)¤ +(d-5)¤ +(e-5)¤ ∴ (분산)= [{(2a-3)-17}¤ +{(2b-3)-17}¤
=4 4
5
+{(2c-3)-17}¤ +{(2d-3)-17}¤ ]
∴ (a-5)¤ +(b-5)¤ +(c-5)¤ +(d-5)¤ +(e-5)¤ =20
1
따라서 변량 3a, 3b, 3c, 3d, 3e에 대하여 = {4(a-10)¤ +4(b-10)¤ +4(c-10)¤
4
3a+3b+3c+3d+3e 3(a+b+c+d+e)
(평균)= = +4(d-10)¤ }
5 5
=(a-10)¤ +(b-10)¤ +(c-10)¤ +(d-10)¤
3_25
= =15 =12 12
5
1
(분산)= {(3a-15)¤ +(3b-15)¤ +(3c-15)¤
5
0059 변량 x¡, x™, x£, x¢, x∞의 평균이 6이므로
=+(3d-15)¤ +(3e-15)¤ }
x¡+x™+x£+x¢+x∞
9 =6
= {(a-5)¤ +(b-5)¤ +(c-5)¤ 5
5
∴ x¡+x™+x£+x¢+x∞=30
=+(d-5)¤ +(e-5)¤ }
또한 변량 x¡, x™, x£, x¢, x∞의 분산이 12이고
9
= _20=36 (변량)¤ 의 총합
5 (분산)= -(평균)¤ 이므로
(변량의 개수)
∴ (표준편차)='3å6=6
x¡¤ +x™¤ +x£¤ +x¢¤ +x∞¤
⑵ 변량 a, b, c의 평균이 6이므로 12= -6¤
5
a+b+c
=6 ∴ a+b+c=18 ∴ x¡¤ +x™¤ +x£¤ +x¢¤ +x∞¤ =240
3
따라서 변량 x¡, x™, x£, x¢, x∞, 15, 18에 대하여
또한 변량 a, b, c의 분산이 8이므로
x¡+x™+x£+x¢+x∞+15+18
(a-6)¤ +(b-6)¤ +(c-6)¤ (평균)=
=8 7
3
30+15+18 63
따라서 변량 a-2, b-2, c-2에 대하여 = = =9
7 7
(a-2)+(b-2)+(c-2) a+b+c-6 x¡¤ +x™¤ +x£¤ +x¢¤ +x∞¤ +15¤ +18¤
(평균)= = (분산)= -9¤
3 3 7
18-6 12 240+225+324 789
= = =4 = -81= -81
3 3 7 7
{(a-2)-4}¤ +{(b-2)-4}¤ +{(c-2)-4}¤ 222 222
∴ (분산)= = 평균:9, 분산:
3 7 7
(a-6)¤ +(b-6)¤ +(c-6)¤
=
3
=8 0060 다음과 같이 표를 만들어 구한다.
⑴ 평균:15, 표준편차:6 ⑵8 국어 성적(점) 도수(명) (계급값)_(도수) (편차)¤ _(도수)
50이상~60미만 1 55_1=55 (-17)¤ _1=289
60이상~70미만 3 65_3=195 (-7)¤ _3=147
0058 변량 a, b, c, d의 평균이 10이므로
70이상~80미만 4 75_4=300 3¤ _4=36
a+b+c+d
=10 ∴ a+b+c+d=40 80이상
~90미만
2 85_2=170 13¤ _2=338
4
또한 변량 a, b, c, d의 분산이 3이므로 합계 10 720 810

01. 대푯값과 산포도 7


(001~042)개념RPM3-2(정답) 2014.11.24 02:53 PM 페이지8 다민 2540DPI 175LPI

RPM 알피엠

720 770
∴ m= =72(점) m= =77(점)
10 10
810 810
s¤ = =81 (분산)= =81
10 10
∴ m-s¤ =72-81=-9 ① s='ß81=9(점)
∴ m+s=77+9=86 ③

0061 {(편차)_(도수)}의 합은 항상 0이므로


0064 주어진 히스토그램을 도수분포표로 나타내면 다음과
(-4)_2+(-2)_1+0_3+2_x+4_1=0
같다.
2x=6 ∴ x=3
(-4)¤ _2+(-2)¤ _1+0¤ _3+2¤ _3+4¤ _1 횟수(회) 도수(명) (계급값)_(도수) (편차)¤ _(도수)
∴ (분산)= 이상 미만
2+1+3+3+1 2 ~ 4 1 3_1=3 (-4)¤ _1=16
64 4이상~ 6미만 2 5_2=10 (-2)¤ _2=8
= =6.4 ⑤
10 6 이상
~ 8미만
4 7_4=28 0¤ _4=0
8이상~10미만 2 9_2=18 2¤ _2=8
이상 미만
10 ~12 1 11_1=11 4¤ _1=16
0062 도수의 총합이 20명이므로
합계 10 70 48
2+a+b+6+3=20
∴ a+b=9 yy ㉠ 70
yy` (평균)= =7(회)
10
이때 평균이 62점이므로
48
40_2+50_a+60_b+70_6+80_3 ∴ (분산)= =4.8 4.8
=62 10
20
50a+60b+740=1240
0065 도수의 총합이 10일이므로 3회 이상 5회 미만인 계급
∴ 5a+6b=50 yy ㉡ yy`
의 도수는
㉠, ㉡을 연립하여 풀면 10-(2+1+2)=5(일)
a=4, b=5 yy`
주어진 히스토그램을 도수분포표로 나타내면 다음과 같다.
1
∴ s¤ = {(40-62)¤ _2+(50-62)¤ _4+(60-62)¤ _5 횟수(회) 도수(일) (계급값)_(도수) (편차)¤ _(도수)
20
+(70-62)¤ _6+(80-62)¤ _3} 3이상~ 5미만 5 4_5=20 (-2)¤ _5=20

1 5이상~ 7미만 2 6_2=12 0¤ _2=0


= _2920=146 yy`
7이상~ 9미만 1 8_1=8 2¤ _1=4
20
이상 미만
∴ a+b+s¤ =4+5+146=155 yy` 9 ~11 2 10_2=20 4¤ _2=32
155 합계 10 60 56

단계 채점요소 배점 60
(평균)= =6(회)
10
도수의 총합을 이용하여 식 세우기 20%
56
평균을 이용하여 식 세우기 20% ∴ (분산)= =5.6 5.6
10
a, b의 값 구하기 20%
s¤ 의 값 구하기 20%
a+b+s¤ 의 값 구하기 20%
0066 도수의 총합이 12명이므로 6편 이상 8편 미만인 계급
의 도수는
12-(1+3+2+2)=4(명) yy`

0063 주어진 히스토그램을 도수분포표로 나타내면 다음과 주어진 히스토그램을 도수분포표로 나타내면 다음과 같다.
같다. 편수(편) 도수(명) (계급값)_(도수) (편차)¤ _(도수)
이상 미만
과학 성적(점) 도수(명) (계급값)_(도수) (편차)¤ _(도수) 4 ~ 6 1 5_1=5 (-4)¤ _1=16
55이상~65미만 1 60_1=60 (-17)¤ _1=289 6이상~ 8미만 4 7_4=28 (-2)¤ _4=16
이상 미만 이상 미만
65 ~75 3 70_3=210 (-7)¤ _3=147 8 ~10 3 9_3=27 0¤ _3=0
75이상~85미만 4 80_4=320 3¤ _4=36 10이상~12미만 2 11_2=22 2¤ _2=8
이상 미만 이상 미만
85 ~95 2 90_2=180 13¤ _2=338 12 ~14 2 13_2=26 4¤ _2=32
합계 10 770 810 합계 12 108 72

8 정답과 풀이
(001~042)개념RPM3-2(정답) 2014.11.24 02:53 PM 페이지9 다민 2540DPI 175LPI

● 본문 014쪽 ~ 016쪽 ●

108 따라서 옳지 않은 것은 ⑤이다. ⑤


(평균)= =9(편) yy`
12

(분산)=
72
=6 0071 산포도는 변량들이 평균 주위에 흩어져 있는 정도를
12
하나의 수로 나타내는 값이므로 표준편차가 클수록 산포도는
∴ (표준편차)='6편 yy`
커지고 자료가 평균을 중심으로 멀리 흩어져 있음을 뜻한다.
'6편
따라서 두 학급의 산포도를 비교하면 성적이 더 고르게 분포한
단계 채점요소 배점
학급을 알 수 있다. ⑤
6편 이상 8편 미만인 계급의 도수 구하기 20%
평균 구하기 40%
0072 주어진 자료의 중앙값과 최빈값을 차례로 구하면 다음
표준편차 구하기 40%
과 같다.
① 4.5, 없다.
0067 ①, ② 평균이 같으므로 어느 과목의 성적이 더 우수하 ② 5, 없다.
다고 할 수 없다. ③ 4, 5
③, ④, ⑤ 2'3<4이므로 음악 성적의 표준편차가 더 작다. ④ 5, 5
따라서 음악 성적이 미술 성적보다 고르다. ③ ⑤ 6, 없다.
따라서 중앙값과 최빈값이 서로 같은 것은 ④이다. ④
0068 평균을 중심으로 밀집되어 있다는 것은 표준편차가 작
은 것을 말하고 표준편차가 작으면 분산이 작다.
0073 윗몸일으키기 횟수는 크기순으로 나열되어 있으므로
따라서 평균을 중심으로 성적이 가장 밀집되어 있는 반은 1반
중앙값은 10번째와 11번째 자료의 값인 14와 15의 평균인
이다. 1반
14+15
=14.5(회) ∴ a=14.5(회)
2
0069 ㄱ. 두 팀 모두 안타 수의 평균이 10개이므로 또한 최빈값은 자료의 값 중 가장 많이 나타난 값이므로
13+13+8+a+10 b=15(회)
=10에서
5
∴ a+b=14.5+15=29.5 29.5
44+a=50 ∴ a=6
9+5+b+18+12
=10에서
5 0074 도수의 총합이 20명이므로
44+b=50 ∴ b=6 2+3+5+a+3+1=20 ∴ a=6
ㄴ. A팀의 분산은 이때 중앙값은 10번째와 11번째 자료의 값인 3과 4의 평균인
(13-10)¤ +(13-10)¤ +(8-10)¤ +(6-10)¤ +(10-10)¤ 3+4
=3.5(권)
5 2
38 또한 4권을 읽은 학생이 가장 많으므로 최빈값은 4권이다.
= =7.6
5 따라서 중앙값과 최빈값의 합은
이므로 A팀의 표준편차는 '∂7.6개이다. 3.5+4=7.5(권) ④
B팀의 분산은
(9-10)¤ +(5-10)¤ +(6-10)¤ +(18-10)¤ +(12-10)¤ 0075 최빈값이 5이므로 나머지 4개의 변량 중 5가 3개 이
5 상이어야 한다.
110
= =22 5가 4개일 때, 평균은
5
2+4_2+5_4 30
이므로 B팀의 표준편차는 'ß22개이다. = +5
7 7
따라서 A팀과 B팀의 표준편차는 같지 않다.
이므로 5가 3개, 나머지 1개가 a(a+5)일 때 평균은 5이다.
ㄷ. B팀의 표준편차가 A팀의 표준편차보다 크므로 최근 다섯
2+4_2+5_3+a 25+a
경기에서 B팀의 타격력이 A팀의 타격력보다 기복이 심 = =5
7 7
하다. 25+a=35 ∴ a=10
따라서 옳은 것은 ㄱ, ㄷ이다. ㄱ, ㄷ 따라서 7개의 변량 중 가장 큰 값은 10이다. 10

0070 ⑤ 도수분포표에서 도수가 가장 큰 계급의 계급값은


0076 자료 A의 중앙값이 40이므로 자료 A를 작은 값부터
최빈값이다. 크기순으로 나열했을 때, 3번째 자료의 값이 40이다.

01. 대푯값과 산포도 9


(001~042)개념RPM3-2(정답) 2014.11.24 02:53 PM 페이지10 다민 2540DPI 175LPI

RPM 알피엠

∴ a=40 2+4+0+x-2=0 ∴ x=-4


자료 B의 중앙값이 50이므로 자료 B를 작은 값부터 크기순으 2¤ +4¤ +0¤ +(-4)¤ +(-2)¤
∴ (분산)=
로 나열했을 때, 3번째와 4번째 자료의 값의 평균이 50이다. 5
40+b 40
이때 a=40이므로 =50 ∴ b=60 = =8 8
2 5

∴ b-a=60-40=20 20
0082 변량 3x¡-1, 3x™-2, 3x£-3, 3x¢-4의 평균이 8이
0077 편차의 합은 항상 0이므로 므로
2-4+x-2+(1-2x)=0 (3x¡-1)+(3x™-2)+(3x£-3)+(3x¢-4)
=8
4
-3-x=0 ∴ x=-3
3(x¡+x™+x£+x¢)-10
이때 C와 E의 성적은 각각 =8
4
72+x=72-3=69(점),
∴ x¡+x™+x£+x¢=14
72+(1-2x)=72+7=79(점)
따라서 변량 x¡, x™, x£, x¢의 평균은
이므로 C와 E의 수학 성적의 평균은
x¡+x™+x£+x¢ 14
69+79 148 = =3.5 ③
= =74(점) ④ 4 4
2 2

{(편차)¤ _(도수)}의 총합
0083 6점이 1개, 7점이 2개, 8점이 4개, 9점이 2개, 10점
0078 (분산)=
(도수)의 총합
이므로 이 1개이므로

(-3)¤ _5+(-2)¤ _1+(-1)¤ _3+1¤ _5+2¤ _3+3¤ _3 6_1+7_2+8_4+9_2+10_1 80


(평균)= = =8(점)
(분산)= 10 10
20
(6-8)¤ _1+(7-8)¤ _2+(8-8)¤ _4+(9-8)¤ _2+(10-8)¤ _1
96 ∴ (분산)=
= =4.8 10
20
∴ (표준편차)='∂4.8점 '∂4.8점 12
= =1.2 1.2
10

0079 변량 6, 7, 8, 9, 10은 변량 1, 2, 3, 4, 5에 각각 5를
더한 것이다.
0084 중앙값이 20이므로 17…a…26
자료를 작은 값부터 크기순으로 나열하면
따라서 각 변량에 일정한 수를 더해도 표준편차는 변하지 않으
14, 17, a, 26
므로 변량 6, 7, 8, 9, 10의 표준편차는 '2이다. ①
이고 중앙값은 2번째와 3번째 자료의 값인 17과 a의 평균이므로
다른풀이
17+a
변량 6, 7, 8, 9, 10에 대하여 2
=20, 17+a=40 ∴ a=23
6+7+8+9+10 40
(평균)= = =8 14+17+23+26 80
5 5 (평균)= 4
= =20
4
(6-8)¤ +(7-8)¤ +(8-8)¤ +(9-8)¤ +(10-8)¤
(분산)= ∴ m=20
5
∴ a-m=23-20=3 ⑤
10
= =2
5
∴ (표준편차)='2 0085 도수의 총합이 10명이므로 70점 이상 80점 미만인 계
급의 도수는
0080 세 학생의 평균을 구해 보면 7점으로 모두 같지만 영 10-(1+2+2)=5(명)
철, 유준, 주완의 순으로 변량이 평균 주위에 밀집되어 있다. 주어진 히스토그램을 도수분포표로 나타내면 다음과 같다.
이때 표준편차가 작을수록 변량이 평균 주위에 밀집되어 있으 과학 성적(점) 도수(명) (계급값)_(도수) (편차)¤ _(도수)
므로 s¡, s™, s£의 대소 관계는 50이상~60미만 1 55_1=55 (-18)¤ _1=324
s¡<s£<s™ ② 60이상~70미만 2 65_2=130 (-8)¤ _2=128
70이상~80미만 5 75_5=375 2¤ _5=20
0081
이상 미만
학생 D의 편차를 x회라고 하면 편차의 합은 항상 0이 80 ~90 2 85_2=170 12¤ _2=288
므로 합계 10 730 760

10 정답과 풀이
(001~042)개념RPM3-2(정답) 2014.11.24 02:53 PM 페이지11 다민 2540DPI 175LPI

● 본문 016쪽 ~ 018쪽 ●

730 따라서 변량 2a+3, 2b+3, 2c+3, 2d+3, 2e+3, 2f+3에


(평균)= =73(점)
10 대하여
760 (평균)
∴ (분산)= =76 76
10
(2a+3)+(2b+3)+(2c+3)+(2d+3)+(2e+3)+(2f+3)
=
6
0086 자료를 작은 값부터 크기순으로 나열할 때, 5번째 자
2(a+b+c+d+e+f)+18
료의 값이 중앙값 2이고 a>b이므로 b=2 =
6
이때 평균이 2이므로 2_48+18 114
= = =19
8-5+8+a+4-2-3+2-1 6 6
=2
9 (분산)
11+a=18 ∴ a=7 1
= [{(2a+3)-19}¤ +{(2b+3)-19}¤ +{(2c+3)-19}¤
(분산) 6
1 +{(2d+3)-19}¤ +{(2e+3)-19}¤ +{(2f+3)-19}¤
= {(8-2)¤ +(-5-2)¤ +(8-2)¤ +(7-2)¤ +(4-2)¤
9 1
= {(2a-16)¤ +(2b-16)¤ +(2c-16)¤ +(2d-16)¤
+(-2-2)¤ +(-3-2)¤ +(2-2)¤ +(-1-2)¤ } 6
200 +(2e-16)¤ +(2f-16)¤ }
=
9 4{(a-8)¤ +(b-8)¤ +(c-8)¤ +(d-8)¤ +(e-8)¤ +( f-8)¤ }
=
200 10'2 10'2 6
∴ (표준편차)=Æ… =
9 3 3 4_24 96
= = =16
6 6
0087 평균이 10이므로 ∴ (표준편차)='1å6=4
5+x+7+y+9 따라서 평균과 표준편차의 합은
=10
5
19+4=23 23
x+y+21=50 ∴ x+y=29
또한 표준편차가 '3, 즉 분산이 3이므로
(5-10)¤ +(x-10)¤ +(7-10)¤ +(y-10)¤ +(9-10)¤ 0090 세 수 a, b, c의 평균이 6이므로
=3 a+b+c
5 =6 ∴ a+b+c=18
3
x¤ +y¤ -20(x+y)+235=15
또한 세 수 a, b, c의 표준편차가 '2, 즉 분산이 2이므로
∴ x¤ +y¤ =20(x+y)-220=20_29-220=360
(a-6)¤ +(b-6)¤ +(c-6)¤
∴ 2xy=(x+y)¤ -(x¤ +y¤ ) =2
3
=29¤ -360=481 ②
(a-6)¤ +(b-6)¤ +(c-6)¤ =6에서
a¤ +b¤ +c¤ -12(a+b+c)+108=6
0088 남학생과 여학생의 평균이 같고 분산이 각각 20, 4이
a¤ +b¤ +c¤ =12(a+b+c)-102
므로 (편차)¤ 의 총합은 각각
=12_18-102=114
20_20=400, 4_20=80
한편 (a+b+c)¤ =a¤ +b¤ +c¤ +2(ab+bc+ca)이므로
따라서 전체 학생 40명의 (편차)¤ 의 총합은
18¤ =114+2(ab+bc+ca)
400+80=480
∴ ab+bc+ca=105
480
(분산)= =12 따라서 세 수 ab, bc, ca의 평균은
40
∴ (표준편차)='ß12=2'3(점) 2'3점 ab+bc+ca 105
= =35 35
3 3

0089 변량 a, b, c, d, e, f의 평균이 8이므로


a+b+c+d+e+f
=8
0091 주어진 꺾은선그래프를 도수분포표로 나타내면 다음
6 과 같다.
∴ a+b+c+d+e+f=48
무게(g) 90 100 110 120 130 140 합계
또, 표준편차가 2, 즉 분산이 4이므로
1반(명) 2 5 8 8 2 1 26
(a-8)¤ +(b-8)¤ +(c-8)¤ +(d-8)¤ +(e-8)¤ +( f-8)¤ 2반(명) 1 4 9 7 4 3 28
=4
6
∴ (a-8)¤ +(b-8)¤ +(c-8)¤ +(d-8)¤ +(e-8)¤ +( f-8)¤ =24 ㄱ. 1반 학생의 최빈값은 110 g, 120 g의 2개이다.

01. 대푯값과 산포도 11


(001~042)개념RPM3-2(정답) 2014.11.24 02:53 PM 페이지12 다민 2540DPI 175LPI

RPM 알피엠

ㄴ. 2반 학생 중에서 도수가 가장 큰 값은 110 g이므로 최빈값 ① 평균은 세 과목이 모두 같다.


은 110 g이다. ② (수학의 분산)<(국어의 분산)<(영어의 분산)이므로 국어
ㄷ. 1반 학생의 평균은 의 산포도가 수학의 산포도보다 크다.
90_2+100_5+110_8+120_8+130_2+140_1 ③ 산포도가 가장 작은 과목은 수학이다.
26 ④ 영어의 분산이 가장 크므로 표준편차가 가장 큰 과목은 영
2920 1460 어이다.
= = (g)
26 13
⑤ 수학의 분산이 가장 작으므로 수학 점수의 분포가 평균 주
2반 학생의 평균은
위에 가장 밀집되어 있다.
90_1+100_4+110_9+120_7+130_4+140_3
28 따라서 옳지 않은 것은 ④이다. ④

3260 815
= = (g)
28 7 0094 편차의 합은 항상 0이므로
ㄹ. 2반 학생은 28명이므로 중앙값은 14번째와 15번째 자료 x+1+0+(-1)+2=0 ∴ x=-2 yy`

110+120 (평균)=(변량)-(편차)이므로
의 값인 110 g과 120 g의 평균인 =115(g)이
2 (평균)=10-(-2)=12(편) yy`
므로 최빈값 110 g과 다르다. (-2)¤ +1¤ +0¤ +(-1)¤ +2¤ 10
(분산)= = =2이므로
따라서 옳은 것은 ㄱ, ㄴ이다. ㄱ, ㄴ 5 5
표준편차는 '2편이다. yy`
0092 자료 A의 평균과 분산을 각각 mÅ, vÅ라고 하면 평균:12편, 표준편차:'2편
a_2+2a_1+3a_2
mÅ= =2a 단계 채점요소 배점
5
x의 값 구하기 30%
(a-2a)¤ _2+(2a-2a)¤ _1+(3a-2a)¤ _2 4
vÅ= = a¤ 평균 구하기 30%
5 5
표준편차 구하기 40%
또한 자료 B의 평균과 분산을 각각 mı, vı라고 하면
b_4+2b_2+3b_4
mı= =2b
10 0095 추가한 두 변량을 각각 x, y라고 하면
(b-2b)¤ _4+(2b-2b)¤ _2+(3b-2b)¤ _4 4 변량 8, 10, 12, x, y의 평균이 9이므로
vı= = b¤
10 5
8+10+12+x+y
4 4 =9
이때 vÅ=vı이므로 a¤ = b¤ 5
5 5
30+x+y=45 ∴ x+y=15 yy`
∴ a=b (∵ a>0, b>0)
또한 변량 8, 10, 12, x, y의 분산이 4이므로
∴ k=1 1
(-1)¤ +1¤ +3¤ +(x-9)¤ +(y-9)¤
=4
5
70+80+70+85+70
0093 (국어의 평균)=
5
11+(x-9)¤ +(y-9)¤ =20
x¤ +y¤ -18(x+y)+173=20
375
= =75(점) x¤ +y¤ =18(x+y)-173+20
5
60+60+65+100+90 375 =18_15-173+20=117 yy`
(영어의 평균)= = =75(점)
5 5 따라서 (x+y)¤ -2xy=x¤ +y¤ 이므로
70+75+75+75+80 375 15¤ -2xy=117 ∴ xy=54 yy`
(수학의 평균)= = =75(점)
5 5
54
(-5)¤ +5¤ +(-5)¤ +10¤ +(-5)¤
(국어의 분산)= 단계 채점요소 배점
5
x+y의 값 구하기 30%
200
= =40
5 x¤ +y¤ 의 값 구하기 30%

(-15)¤ +(-15)¤ +(-10)¤ +25¤ +15¤ xy의 값 구하기 40%


(영어의 분산)=
5
1400
=
5
=280 0096 도수의 총합이 20명이므로
4+a+b+5+2=20
(-5)¤ +0¤ +0¤ +0¤ +5¤ 50
(수학의 분산)= = =10 ∴ a+b=9 yy ㉠ yy`
5 5

12 정답과 풀이
(001~042)개념RPM3-2(정답) 2014.11.24 02:53 PM 페이지13 다민 2540DPI 175LPI

● 본문 018쪽 ~ 019쪽 ●

또한 평균이 59점이므로 어도 2개는 10이 되어야 한다.


40_4+50_a+60_b+70_5+80_2 즉, a, b, c의 값을 10, 10, x라 하면
=59
20 4, 5, 5, 7, 10, 10, 10, x
50a+60b+670=1180 이때 중앙값이 8점이므로 위의 자료를 작은 값부터 크기순으
∴ 5a+6b=51 yy ㉡ yy`
로 나열하면 4번째와 5번째 값의 평균이 8이다.
㉠, ㉡을 연립하여 풀면 즉, 7…x…10이어야 하므로
a=3, b=6 yy`
7+x
(중앙값)= =8
따라서 분산은 2
1 ∴ x=9
{(40-59)¤ _4+(50-59)¤ _3+(60-59)¤ _6
20 ∴ a+b+c=10+10+9=29 29
+(70-59)¤ _5+(80-59)¤ _2}
1444+243+6+605+882
=
20 0099 학생 6명의 수학 성적의 총합은
70_6=420(점)
3180
= =159 yy`
이때 나머지 5명의 평균은
20
159 420-70 350
= =70(점)
5 5
단계 채점요소 배점
학생 6명의 (편차)¤ 의 총합은 20_6=120이고 빠진 학생의 편
도수의 총합을 이용하여 식 세우기 20%
차는 0이므로 나머지 5명의 분산은
평균을 이용하여 식 세우기 20%
1
a, b의 값 구하기 20% [{6명의 (편차)¤ 의 총합}-{빠진 한 학생 점수의 (편차)¤ }]
5
분산 구하기 40%
1
= (120-0)
5
=24 24
0097 반지름의 길이의 평균이 4이므로
a+b+c
3
=4 ∴ a+b+c=12 yy`
0100 잘못 계산된 한 학생을 제외한 나머지 9명의 몸무게를
반지름의 길이의 표준편차가 '3, 즉 분산이 3이므로 x¡, x™, y, xª kg이라고 하면 잘못 계산된 몸무게의 평균이
(a-4)¤ +(b-4)¤ +(c-4)¤ 50 kg이므로
=3
3 x¡+x™+y+xª+55
=50
10
(a-4)¤ +(b-4)¤ +(c-4)¤ =9에서
a¤ +b¤ +c¤ -8(a+b+c)+48=9 ∴ x¡+x™+y+xª=445

a¤ +b¤ +c¤ =8(a+b+c)-39 따라서 실제 10명의 몸무게의 평균은

=8_12-39=57 yy` x¡+x™+y+xª+45 445+45 490


= = =49(kg)
10 10 10
따라서 세 원의 넓이는 각각 a¤ p, b¤ p, c¤ p이므로
또한 잘못 계산된 몸무게의 표준편차가 4 kg, 즉 분산이 16이
세 원의 넓이의 평균은
므로
a¤ p+b¤ p+c¤ p (a¤ +b¤ +c¤ )p
= (변량)¤ 의 총합
3 3 (분산)= -(평균)¤ 에서
(변량의 개수)
57p
= =19p yy`
x¡¤ +x™¤ +y+xª¤ +55¤
3 -50¤ =16
10
19p
따라서 실제 10명의 몸무게의 분산은
단계 채점요소 배점
x¡¤ +x™¤ +y+xª¤ +45¤
a+b+c의 값 구하기 30% -49¤
10
a¤ +b¤ +c¤ 의 값 구하기 30%
x¡¤ +x™¤ +y+xª¤ +(55-10)¤
세 원의 넓이의 평균 구하기 40% = -(50-1)¤
10
x¡¤ +x™¤ +y+xª¤ +55¤
= -50¤ +(-100)+100-1
10
0098 a, b, c를 제외한 자료에서 5의 도수가 2로 가장 크고 =16-100+100-1
10의 도수가 1이므로 최빈값이 10점이 되려면 a, b, c 중 적 =15 15

01. 대푯값과 산포도 13


(001~042)개념RPM3-2(정답) 2014.11.24 02:53 PM 페이지14 다민 2540DPI 175LPI

RPM 알피엠
Ⅱ 피타고라스 정리

0101 A 학교의 남학생 수와 여학생 수를 각각 a명, b명이


라 하고 B 학교의 남학생 수와 여학생 수를 각각 c명, d명이
02 피타고라스정리

라고 하자.
A, B 두 학교의 전체 평균이 각각 74점, 84점이므로 0102 x¤ =1¤ +('3)¤ , x¤ =4

71a+76b 2 ∴ x=2 (∵ x>0) 2


=74 ∴ a= b
a+b 3
81c+90d 0103 10¤ =x¤ +6¤ , x¤ =64
=84 ∴ c=2d yy ㉠
c+d ∴ x=8 (∵ x>0) 8
또한 A, B 두 학교의 여학생 전체의 평균이 84점이므로
76b+90d
=84 ∴ d=
4
b yy ㉡ 0104 8¤ =4¤ +x¤ , x¤ =48
b+d 3 ∴ x=4'3 (∵ x>0) 4'3
㉡을 ㉠에 대입하면

c=2_
4 8
b= b 0105 x¤ =2¤ +(3'2)¤ , x¤ =22
3 3
∴ x='ß22 (∵ x>0) 'ß22
따라서 A, B 두 학교의 남학생 전체의 평균은

71a+81c
71_`;3@;`b+81_`;3*;`b 0106 x="√13¤ -5¤ ='∂144=12
x= = y="√9¤ +12¤ ='∂225=15 x=12, y=15
a+c
;3@;`b+`;3*;`b

123`b 0107
790 x="√2¤ +2¤ ='8=2'2
= 1123 =79(점)
3
79점 y="√(2'2)¤ +(2'2)¤ ='ß16=4 x=2'2, y=4
13`b
10
3
0108 ㈎ SAS ㈏ △BFL ㈐ BFML
㈑ LMGC ㈒ BC” ¤

0109 (색칠한 부분의 넓이)=36+64=100(cm¤ )


100 cm¤

0110 (색칠한 부분의 넓이)=22-15=7(cm¤ ) 7 cm¤

0111 ⑴ △BFE에서 BF”=9-6=3(cm)이므로


EF”="√6¤ +3¤ ='ß45=3'5(cm)
이때 EFGH는 정사각형이므로
( EFGH의 둘레의 길이)=4_3'5=12'5(cm)
⑵ EFGH=(3'5)¤ =45(cm¤ )
⑴ 12'5 cm ⑵ 45 cm¤

0112 ⑴ △ABE에서 BE”="√13¤ -5¤ ='∂144=12(cm)


BF”=AE”=5 cm이므로
EF”=12-5=7(cm)
⑵ EFGH는 정사각형이므로 EFGH=7¤ =49(cm¤ )
⑴ 7 cm ⑵ 49 cm¤

0113 ㄱ. ('3)¤ =1¤ +('2)¤ (직각삼각형)


ㄴ. ('5)¤ =1¤ +2¤ (직각삼각형)
ㄷ. 7¤ +3¤ +(4'3)¤
ㄹ. 6¤ +4¤ +5¤
ㅁ. 10¤ =6¤ +8¤ (직각삼각형)

14 정답과 풀이
(001~042)개념RPM3-2(정답) 2014.11.24 02:53 PM 페이지15 다민 2540DPI 175LPI

● 본문 019쪽 ~ 025쪽 ●

ㅂ. 14¤ +5¤ +12¤ 0124 AB” ¤ =BC”_BH”이므로


따라서 직각삼각형인 것은 ㄱ, ㄴ, ㅁ이다. ㄱ, ㄴ, ㅁ x¤ =(16+9)_16=400 ∴ x=20 (∵ x>0)
A’H” ¤ =BH”_CH”이므로
0114 3¤ >2¤ +2¤ ∴ 둔각삼각형 둔각삼각형 y¤ =16_9=144 ∴ y=12 (∵ y>0)
x=20, y=12
0115 ('7 )¤ =('3)¤ +2¤ ∴ 직각삼각형 직각삼각형
0125 A’H” ¤ =BH”_CH”이므로
0116 9¤ <6¤ +8¤ ∴ 예각삼각형 예각삼각형 6¤ =4x ∴ x=9
△AHC에서 y="√6¤ +9¤ ='∂117=3'ß13
0117 ('ß10 )¤ >('5)¤ +2¤ ∴ 둔각삼각형 둔각삼각형 x=9, y=3'ß13

0118 7¤ <4¤ +6¤ ∴ 예각삼각형 예각삼각형 0126 x=øπ6¤ -(2'5 )¤ ='1å6=4


AB”_CH”=BC”_AC”이므로
0119 ('ß10 )¤ =1¤ +3¤ ∴ 직각삼각형 직각삼각형 6_y=4_2'5
4'5 4'5
∴ y= x=4, y=
0120 삼각형의 세 변의 길이 사이의 관계에 의해 3 3
4-3<x<4+3 ∴ 1<x<7 yy ㉠
∠A가 예각이므로 0127 AB” ¤ =BC”_BH”이므로
x¤ <3¤ +4¤ , x¤ <25 8¤ =10_(10-x)
∴ 0<x<5 yy ㉡ 64=100-10x, 10x=36
㉠, ㉡에서 1<x<5 1<x<5 18
∴ x=
5
18 32
0121 삼각형의 세 변의 길이 사이의 관계에 의해 BH”=10-
5
=
5
이므로
6-3<x<6+3 ∴ 3<x<9 yy ㉠
32 ¤ 576 24 18 24
∠A가 예각이므로 y=æ≠8¤ -{ } =æ≠ = x= , y=
5 25 5 5 5
x¤ <6¤ +3¤ , x¤ <45
∴ 0<x<3'5 yy ㉡ 0128 6¤ +5¤ =7¤ +x¤ , x¤ =12
㉠, ㉡에서 3<x<3'5 3<x<3'5 ∴ x=2'3 (∵ x>0) 2'3

0122 삼각형의 세 변의 길이 사이의 관계에 의해 0129 6¤ +8¤ =5¤ +x¤ , x¤ =75


2-1<x<2+1 ∴ 1<x<3 ∴ x=5'3 (∵ x>0) 5'3
그런데 x가 가장 긴 변의 길이이므로 x>2
∴ 2<x<3 yy ㉠ 0130 5¤ +('5)¤ =x¤ +3¤ , x¤ =21
또한 ∠A가 둔각이므로 ∴ x='ß21 (∵ x>0) 'ß21
x¤ >1¤ +2¤ , x¤ >5
∴ x>'5 yy ㉡ 0131 (2'5)¤ +x¤ =4¤ +8¤ , x¤ =60
㉠, ㉡에서 '5<x<3 '5<x<3 ∴ x=2'ß15 (∵ x>0) 2'ß15

0123 삼각형의 세 변의 길이 사이의 관계에 의해 0132 5¤ +6¤ =4¤ +x¤ , x¤ =45


6-4<x<6+4 ∴ 2<x<10 ∴ x=3'5 (∵ x>0) 3'5
그런데 x가 가장 긴 변의 길이이므로 x>6
∴ 6<x<10 yy ㉠ 0133 x¤ +4¤ =('5)¤ +5¤ , x¤ =14
또한 ∠A가 둔각이므로 ∴ x='ß14 (∵ x>0) 'ß14
x¤ >4¤ +6¤ , x¤ >52
∴ x>2'ß13 yy ㉡ 0134 4¤ +8¤ =x¤ +6¤ , x¤ =44
㉠, ㉡에서 2'ß13<x<10 2'ß13<x<10 ∴ x=2'ß11 (∵ x>0) 2'ß11

02. 피타고라스 정리 15
(001~042)개념RPM3-2(정답) 2014.11.24 02:53 PM 페이지16 다민 2540DPI 175LPI

RPM 알피엠

0135 ('7 )¤ +2¤ =3¤ +x¤ , x¤ =2 0143 △ABD에서 AD”="√3¤ +4¤ ='ß25=5(cm)
∴ x='2 (∵ x>0) '2 CD”=AD”=5 cm이므로 BC”=4+5=9(cm)
따라서 △ABC에서
0136 (색칠한 부분의 넓이)=26+13 AC”="√3¤ +9¤ ='ß90=3'ß10(cm) 3'ß10 cm
=39(cm¤ ) 39 cm¤
0144 CD”=x cm라고 하면 △ADC에서 AC”="√3¤ -x¤ (cm)
0137 (색칠한 부분의 넓이)=△ABC 따라서 △ABC에서
1 (2'5)¤ =('5+x)¤ +("√3¤ -x¤ )¤
= _6_4
2
20=5+2'5x+x¤ +9-x¤
=12(cm¤ ) 12 cm¤
3 3'5 3'5
2'5x=6 ∴ x= = (cm) cm
'5 5 5
0138 (x+2)¤ =x¤ +8¤ , 4x=60
∴ x=15 ③ 0145 삼각형의 각의 이등분선의 성질에 의하여
AB”:AC”=BD”:CD”=12:6=2:1 yy`

△ABC에서
0139 지면에서 나무가 부러진 부분까 A
지의 높이를 x m라고 하면 나무가 부 BC”="√12¤ -6¤ ='∂108=6'3(cm) yy`
(10-x) m
러진 부분에서 쓰러진 지점까지의 거리 xm 1 1
이므로 CD”= BC”= _6'3=2'3(cm) yy`
3 3
는 (10-x) m이다.
B
4m C 따라서 △ADC에서
(10-x)¤ =4¤ +x¤ , 20x=84
AD”=øπ(2'3)¤ +6¤ ='4å8=4'3(cm) yy`
∴ x=4.2(m)
4'3 cm
따라서 지면에서 나무가 부러진 부분까지의 높이는 4.2 m이
다. 4.2 m 단계 채점요소 배점
BD”:DC”=2:1임을 이해하기 30%

0140 직각삼각형 ABC의 넓이가 4'5 cm¤ 이므로 BC”의 길이 구하기 20%
CD”의 길이 구하기 20%
1
_4_AC”=4'5 ∴ AC”=2'5(cm) AD”의 길이 구하기 30%
2
∴ AB”=øπ4¤ +(2'5)¤ ='3å6=6(cm) 6 cm 참고 삼각형의 각의 이등분선의 성질
△ABC에서 ∠A의 이등분선이 BC”와 A
0141 △ABC에서 만나는 점을 D라고 하면
AB”="√8¤ +6¤ ='∂100=10(cm) yy` AB”:AC”=BD”:CD”
B C
이때 직각삼각형에서 빗변의 중점은 외심과 일치하므로 D

1 1
CD”=AD”=BD”= AB”= _10=5(cm) yy`
2 2 0146 AC”="√x¤ +x¤ ='2x
2 2 10 AD”="√('2x)¤ +x¤ ='3x
∴ CG”= CD”= _5= (cm) yy`
3 3 3
AE”="√('3x)¤ +x¤ =2x
10
cm AF”="√(2x)¤ +x¤ ='5x
3
AG”="√('5x)¤ +x¤ ='6x
단계 채점요소 배점
그런데 AG”=3'6이므로
AB”의 길이 구하기 20%
CD”의 길이 구하기 40%
'6x=3'6 ∴ x=3
CG”의 길이 구하기 40% △AGF에서 AF”=3'5, FG”=3이므로
1 9'5 9'5
△AGF= _3'5_3=
2 2 2
0142 △AHC에서 A’H=
” "√5¤ -3¤ ='ß16=4(cm)
따라서 △ABH에서 0147 ⑴ AB”=a라고 하면
AB”="√6¤ +4¤ ='ß52=2'ß13(cm) 2'ß13 cm AB”=BC”=CD”=DE”=EF”=a

16 정답과 풀이
(001~042)개념RPM3-2(정답) 2014.11.24 02:53 PM 페이지17 다민 2540DPI 175LPI

● 본문 025쪽 ~ 028쪽 ●

AC”="√a¤ +a¤ ='2a 0152 꼭짓점 D에서 BC”에 내린 수


A
4 cm D
AD”=øπ('2a)¤ +a¤ ='3a 선의 발을 H라고 하면 10 cm
AE”=øπ('3a)¤ +a¤ =2a BH”=AD”=4 cm
B C
AF”=øπ(2a)¤ +a¤ ='5a ∴ HC”=12-4=8(cm) H
12 cm
그런데 AF”=2'5이므로 '5a=2'5 ∴ a=2 △DHC에서
∴ x=2'2 D’H=
” "√10¤ -8¤ ='ß36=6(cm)
⑵ AC”=øπ('2)¤ +('3)¤ ='5 따라서 △DBH에서
AD”=øπ('5)¤ +('3)¤ ='8=2'2 BD”="√4¤ +6¤ ='ß52=2'ß13(cm) ③
AE”=øπ(2'2)¤ +('3)¤ ='1å1
∴ x=AF”=øπ('1å1 )¤ +('3 )¤ ='1å4
⑴ 2'2 ⑵ '1å4 0153 두 꼭짓점 A, D에서 BC” A 4 cm D
에 내린 수선의 발을 각각 E, F라 5 cm
5 cm
고 하면
B C
0148 OB”=O’A'”="√2¤ +2¤ ='8=2'2(cm) EF”=AD”=4 cm yy` E
10 cm
F

∴ OC”=O’B'”="√(2'2)¤ +2¤ ='ß12=2'3(cm) ABCD가 등변사다리꼴이므로


2'3 cm 1
BE”=FC”= _(10-4)=3(cm) yy`
2
△ABE에서 AE”="√5¤ -3¤ ='ß16=4(cm) yy`

1
0149 O’O'”=x라고 하면 ∴ ABCD=
2
_(4+10)_4=28(cm¤ ) yy`

OB”=O’A'”="√x¤ +x¤ ='2x


28 cm¤
OC”=O’B'”="√('2x)¤ +x¤ ='3x
단계 채점요소 배점
OD”=O’C'”="√('3x)¤ +x¤ =2x
EF”의 길이 구하기 20%
그런데 OD”=6'3이므로
BE” 의 길이 구하기 20%
2x=6'3 ∴ x=3'3 3'3 AE” 의 길이 구하기 30%
ABCD의 넓이 구하기 30%

0150 꼭짓점 A에서 BC”에 내린 수 A 12 cm D


선의 발을 H라고 하면
13 cm 0154 DC”∥EB”이므로 D
I
HC”=AD”=12 cm △EBA=△EBC A
E '7 cm H
∴ BH”=17-12=5(cm) B C 또한 △EBC=△ABF
H
△ABH에서 17 cm
(SAS 합동)이므로 B L C
4 cm
A’H=
” "√13¤ -5¤ ='∂144=12(cm) △ABF=△EBA
1 이때 △ABC에서 F G
∴ ABCD= _(12+17)_12=174(cm¤ ) M
2
AB”=øπ4¤ -('7)¤ ='9=3(cm)이므로
174 cm¤
1
△ABF=△EBA= ADEB
2
1 9 9
0151 대각선 AC를 그으면 A =
2
_3_3= (cm¤ )
2 2
cm¤
D
△ABC에서
4 cm
AC”="√4¤ +3¤ ='ß25=5(cm) 2'3 cm
△ACD에서 0155 DC”∥EB”이므로 △EBA=△EBC
B 3 cm C
△ABF™△EBC(SAS 합동)이므로 △ABF=△EBC
AD”="√5¤ -(2'3)¤ ='ß13(cm)
BF”∥A’M이
” 므로 △ABF=△BFL
1 1
∴ ABCD= _3_4+ _2'3_'ß13 ∴ △EBA=△EBC=△ABF=△BFL
2 2
=6+'ß39(cm¤ ) (6+'ß39) cm¤ 따라서 넓이가 다른 것은 ② △BCH이다. ②

02. 피타고라스 정리 17
(001~042)개념RPM3-2(정답) 2014.11.24 02:53 PM 페이지18 다민 2540DPI 175LPI

RPM 알피엠

0156 BC” ¤ =AB” ¤ +AC” ¤ 따라서 PQRS는 정사각형이므로


= GBAF+ ACDE PQRS=QR” ¤ =(2'3-2)¤ =16-8'3 16-8'3
=120+49=169
∴ BC”=13(cm) (∵ BC”>0) 13 cm 0163 ① BQ”=AP”=5 cm이므로 △ABQ에서
AQ”="√13¤ -5¤ ='∂144=12(cm)
0157 △ABC에서 ② △ABQ=
1
_12_5=30(cm¤ )
2
AB”="√12¤ -6¤ ='∂108=6'3(cm) yy`
③ AQ”=AP”+PQ”에서 12=5+PQ”
1 1
∴ △FDG= BDGF= AB” ¤ ∴ PQ”=7(cm)
2 2
1 ④ PQRS=PQ” ¤ =7¤ =49(cm¤ )
= _(6'3)¤ =54(cm¤ ) yy`
2 ⑤ ABCD=AB” ¤ =13¤ =169(cm¤ )
54 cm¤
1
∴ PQRS+ ABCD
4
단계 채점요소 배점
따라서 옳지 않은 것은 ⑤이다. ⑤
AB”의 길이 구하기 30%
△FDG의 넓이 구하기 70%
0164 AE”=CG”=3 cm이므로 △ABE에서
BE”="√6¤ -3¤ ='ß27=3'3(cm)
0158 EFGH는 정사각형이므로
BF”=CG”=3 cm이므로 EF”=(3'3-3) cm
EF” ¤ =225 ∴ EF”=15(cm) (∵ EF”>0)
따라서 EFGH는 한 변의 길이가 (3'3-3) cm인 정사각형
△AFE에서 AF”="√15¤ -12¤ ='ß81=9(cm)
이므로 둘레의 길이는
BF”=AE”=12 cm이므로 AB”=9+12=21(cm)
4(3'3-3)=12('3-1)(cm) 12('3-1) cm
∴ ABCD=AB” ¤ =21¤ =441(cm¤ ) 441 cm¤

0165 PQRS는 정사각형이고, 넓이는 9이므로


0159 D’H=
” AE”=8 cm이므로
PQ” ¤ =9 ∴ PQ”=3 (∵ PQ”>0)
A’H”=14-8=6(cm)
BQ”=AP”=3이므로 BP”=3+3=6
△AEH에서 EH”="√8¤ +6¤ ='∂100=10(cm)
△ABP에서 AB”="√3¤ +6¤ ='ß45=3'5
따라서 EFGH는 정사각형이므로
∴ ABCD=AB” ¤ =(3'5)¤ =45 45
EFGH=EH” ¤ =10¤ =100(cm¤ ) 100 cm¤

0166 △ACE는 AC”=CE”인 직각이등변삼각형이므로


0160 △AEH에서 EH”="√x¤ +y¤ ='ß15
1
따라서 EFGH는 정사각형이므로 AC” ¤ =40, AC” ¤ =80
2
EFGH=EH” ¤ =('ß15)¤ =15 15 ∴ AC”='8å0=4'5(cm) (∵ AC”>0)
△ABC에서 BC”=øπ(4'5)¤ -4¤ ='6å4=8(cm)이고
0161 AE”=x cm라고 하면 △AEH에서 CD”=AB”=4 cm이므로
EH”="√x¤ +x¤ ='2x(cm) BD”=8+4=12(cm)
그런데 EFGH는 정사각형이고, 둘레의 길이가 8'2 cm이 DE”=BC”=8 cm이므로
므로 1
ABDE= _(4+8)_12=72(cm¤ ) ④
2
4_'2x=8'2 ∴ x=2(cm)
따라서 AB”=2AE”=2_2=4(cm)이므로
0167 △ACE는 AC”=CE”인 직각이등변삼각형이고
( ABCD의 둘레의 길이)=4AB”=4_4=16(cm)
AB”=CD”=3 cm이므로
16 cm
AC”=øπ('3)¤ +3¤ ='1å2=2'3(cm)
1
0162 BQ”=AP”=2이므로 △BCQ에서 ∴ △ACE=
2
_2'3_2'3=6(cm¤ ) ①
CQ”="√4¤ -2¤ ='ß12=2'3
CR”=AP”=2이므로 QR”=2'3-2 0168 △ACE는 AC”=CE”인 직각이등변삼각형이므로

18 정답과 풀이
(001~042)개념RPM3-2(정답) 2014.11.24 02:53 PM 페이지19 다민 2540DPI 175LPI

● 본문 028쪽 ~ 031쪽 ●

AC”=CE”=x cm라고 하면 단계 채점요소 배점


x¤ +x¤ =(10'2)¤ , 2x¤ =200 가장 긴 변의 길이가 6일 때 x의 값 구하기 50%
x¤ =100 ∴ x=10(cm) (∵ x>0) 가장 긴 변의 길이가 x일 때 x의 값 구하기 50%

△ABC에서 BC”="√10¤ -6¤ ='6å4=8(cm)이고


DE”=BC”=8 cm, CD”=AB”=6 cm이므로 0173 ① 7¤ <4¤ +6¤ ∴ 예각삼각형
1 ② 9¤ >4¤ +8¤ ∴ 둔각삼각형
ABDE= _(6+8)_14=98(cm¤ ) ②
2
③ 9¤ <6¤ +7¤ ∴ 예각삼각형
④ 10¤ <6¤ +9¤ ∴ 예각삼각형
0169 ∠C=90˘이면 빗변의 길이가 x+3이므로 ⑤ 15¤ =9¤ +12¤ ∴ 직각삼각형 ②
(x+3)¤ =x¤ +(x-3)¤ , x¤ -12x=0
x(x-12)=0 ∴ x=0 또는 x=12 yy ㉠ 0174 ④ a¤ <b¤ +c¤ 이면 ∠A<90˘이지만 a가 가장 긴 변의

그런데 변의 길이는 양수이므로 길이인지 알 수 없으므로 △ABC가 반드시 예각삼각형이

x-3>0 ∴ x>3 yy ㉡ 되는 것은 아니다. ④

㉠, ㉡에서 x=12 ③
0175 ① ('3)¤ +2¤ =('7)¤ ∴ 직각삼각형
② ('5)¤ +3¤ >('1å0 )¤ ∴ 예각삼각형
0170 ㄱ. 2¤ =1¤ +('3)¤ (직각삼각형) ③ 4¤ +5¤ >6¤ ∴ 예각삼각형
ㄴ. (6'3 )¤ +6¤ +6¤ ④ 6¤ +8¤ <12¤ ∴ 둔각삼각형
ㄷ. ('ß41 )¤ =4¤ +5¤ (직각삼각형) ⑤ 9¤ +12¤ =15¤ ∴ 직각삼각형 ②, ③
ㄹ. 4¤ =('7 )¤ +3¤ (직각삼각형)
ㅁ. (4'2 )¤ +(2'2)¤ +3¤ 0176 x가 가장 긴 변의 길이이므로 삼각형의 세 변의 길이
ㅂ. 6¤ +('ß15 )¤ +4¤ 사이의 관계에 의해
따라서 직각삼각형인 것은 ㄱ, ㄷ, ㄹ이다. ㄱ, ㄷ, ㄹ 8<x<6+8 ∴ 8<x<14 yy ㉠ yy`

⑴ 예각삼각형이 되려면 x¤ <6¤ +8¤ , x¤ <100


∴ 0<x<10 (∵ x>0) yy ㉡
0171 2x+1이 가장 긴 변의 길이이므로 삼각형의 세 변의
㉠, ㉡에서 8<x<10 yy`
길이 사이의 관계에 의해
⑵ 둔각삼각형이 되려면 x¤ >6¤ +8¤ , x¤ >100
2x+1<x+(2x-1) ∴ x>2
∴ x>10 (∵ x>0) yy ㉢
주어진 삼각형이 직각삼각형이므로
㉠, ㉢에서 10<x<14 yy`
(2x+1)¤ =x¤ +(2x-1)¤
⑴ 8<x<10 ⑵ 10<x<14
x¤ -8x=0, x(x-8)=0
∴ x=0 또는 x=8 단계 채점요소 배점

x>2이므로 x=8 삼각형이 되기 위한 x의 값의 범위 구하기 20%


예각삼각형이 되는 x의 값의 범위 구하기 40%
따라서 구하는 직각삼각형의 넓이는
둔각삼각형이 되는 x의 값의 범위 구하기 40%
1
_8_15=60 60
2
0177 삼각형의 세 변의 길이 사이의 관계에 의해
5-4<x<5+4 ∴ 1<x<9 yy ㉠
0172 ⁄ 가장 긴 변의 길이가 6일 때
또한 ∠A가 예각이므로
6¤ =3¤ +x¤ , x¤ =27
x¤ <4¤ +5¤ , x¤ <41 ∴ 0<x<'ß41 yy ㉡
∴ x=3'3 (∵ x>0) yy`
㉠, ㉡에서 1<x<'ß41 1<x<'ß41
¤ 가장 긴 변의 길이가 x일 때
x¤ =3¤ +6¤ , x¤ =45 0178 ㈎ (a+x)¤ ㈏ x¤ +y¤ ㈐ b¤ ㈑ >
∴ x=3'5 (∵ x>0) yy`

⁄, ¤에서 x=3'3 또는 x=3'5 0179 삼각형의 세 변의 길이 사이의 관계에 의해


3'3, 3'5 12-5<a<12+5 ∴ 7<a<17 yy ㉠

02. 피타고라스 정리 19
(001~042)개념RPM3-2(정답) 2014.11.24 02:53 PM 페이지20 다민 2540DPI 175LPI

RPM 알피엠

또한 ∠A의 대변의 길이가 a이므로 ∠A<90˘가 되려면 단계 채점요소 배점


a¤ <12¤ +5¤ , a¤ <169 ∴ 0<a<13 yy ㉡ BH”=k cm라고 할 때 AH”를 k로 나타내기 30%
㉠, ㉡에서 7<a<13 BH”의 길이 구하기 40%

따라서 자연수 a는 8, 9, 10, 11, 12의 5개이다. 5개 BC”의 길이 구하기 30%

0184 △ABC에서 BC”=øπ(4'3)¤ +4¤ ='6å4=8(cm)


0180 삼각형의 세 변의 길이 사이의 관계에 의해
¤
AC” =CH”_CB”이므로
8-4<x<8+4 ∴ 4<x<12
4¤ =CH”_8 ∴ CH”=2(cm)
그런데 8이 가장 긴 변이므로 x<8
△AHC에서 AH”="√4¤ -2¤ ='1å2=2'3(cm)
∴ 4<x<8 yy ㉠ yy`
1 1
∠C가 둔각이 되려면 이때 MC”= BC”= _8=4(cm)이므로
2 2
8¤ >4¤ +x¤ , x¤ <48
MH”=4-2=2(cm)
∴ 0<x<4'3 yy ㉡ yy`
1
㉠, ㉡에서 4<x<4'3 yy` ∴ △AMH= _2_2'3=2'3(cm¤ ) 2'3 cm¤
2
4<x<4'3
0185 △ABC에서 AB”="√25¤ -20¤ ='∂225=15(cm)
단계 채점요소 배점
AB”_AC”=BC”_AH”이므로
x의 값의 범위 구하기 40%
15_20=25_AH”
∠C가 둔각이기 위한 x의 값의 범위 구하기 40%
∴ AH”=12(cm) ②
x의 값의 범위 구하기 20%

0186 △ABC에서 BC”=øπ(6'5)¤ +12¤ ='∂324=18(cm)

0181 △ABH에서 x="√3¤ +6¤ ='ß45=3'5 AB”_AC”=BC”_AH”이므로


AB” ¤ =BH”_BC”이므로 6'5_12=18_AH”
(3'5 )¤ =3y ∴ y=15 x=3'5, y=15 ∴ AH”=4'5(cm) 4'5 cm

0187 △ABC에서 BC”="√20¤ -16¤ ='∂144=12(cm)


0182 ⑴ A’H” ¤ =BH”_CH”이므로 BC” ¤ =CH”_CA”이므로
6¤ =x_(15-x), x¤ -15x+36=0 36
12¤ =y_20 ∴ y= (cm)
(x-3)(x-12)=0 ∴ x=3 또는 x=12 5
그런데 BH”>CH”이므로 또한 BA”_BC”=AC”_BH”이므로
x=12 48
12_16=20_x ∴ x= (cm)
⑵ BC” ¤ =BA”_BH”이므로 5
6¤ =BA”_3'3 ∴ BA”=4'3(cm) 48 36 12 12
∴ x-y= - =
5 5 5 5
AH”=4'3-3'3='3(cm)
AC” ¤ =AB”_AH”이므로 0188 AB”:BC”=2:3이므로
x¤ =4'3_'3=12 AB”=2k cm, BC”=3k cm(k>0)라고 하면
∴ x=2'3 (∵ x>0) △ABC에서 AC”="√(3k)¤ +(2k)¤ ='1å3k
⑴ 12 ⑵ 2'3 AB”_BC”=AC”_BH”이므로
4'1å3
2k_3k='1å3k_8 ∴ k= (cm)
3
0183 BH”=k cm(k>0)라고 하면 AH”=2k cm yy`
¤ 4'1å3
CH” =AH”_BH”이므로 ∴ BC”=3k=3_ =4'1å3(cm) 4'1å3 cm
3
(3'2)¤ =2k_k, 18=2k¤
k¤ =9 ∴ k=3(cm) (∵ k>0) yy` 0189 △ABO에서 AB”="√3¤ +2¤ ='ß13
BH”=3 cm이므로 △BCH에서 AB” ¤ +CD” ¤ =AD” ¤ +BC” ¤ 이므로
BC”=øπ3¤ +(3'2)¤ ='2å7=3'3(cm) yy` ('ß13)¤ +(2'5)¤ =5¤ +BC” ¤ , BC” ¤ =8
3'3 cm ∴ BC”=2'2 (∵ BC”>0) ②

20 정답과 풀이
(001~042)개념RPM3-2(정답) 2014.11.24 02:53 PM 페이지21 다민 2540DPI 175LPI

● 본문 031쪽 ~ 034쪽 ●

0190 AB” ¤ +CD” ¤ =AD” ¤ +BC” ¤ 이므로 0198 AP” ¤ +CP” ¤ =BP” ¤ +DP” ¤ 이므로
4¤ +5¤ =AD” ¤ +6¤ , AD” ¤ =5 4¤ + CP” ¤ =3¤ +5¤ , CP” ¤ =18
∴ AD”='5 (∵ AD”>0) '5 ∴ CP”=3'2(km) (∵ CP”>0)
따라서 공원 P에서 C의 집까지의 거리는 3'2 km이다.
0191 AB” ¤ +CD” ¤ =AD” ¤ +BC” ¤ 이므로 3'2 km
(2'3 )¤ +CD” ¤ =('ß22)¤ +('ß15)¤ , CD” ¤ =25
∴ CD”=5 (∵ CD”>0) 0199 AP” ¤ +CP” ¤ =BP” ¤ +DP” ¤ 이므로
따라서 △OCD에서 DP” -CP” ¤ =AP” ¤ -BP” ¤ =6¤ -5¤ =11
¤ 11
OD”="√5¤ -3¤ ='ß16=4이므로
1
0200 △ABD에서 BD”="√(4'3)¤ +4¤ ='ß64=8 yy`
△OCD= _3_4=6 6 AB”_AD”=AP”_BD”이므로
2
4_4'3=AP”_8 ∴ AP”=2'3 yy`

0192 ABCD는 등변사다리꼴이므로 AB”=CD”이고 ¤


AB” =BP”_BD”이므로
AC”⊥BD”이므로 4¤ =BP”_8 ∴ BP”=2
AB” ¤ +CD” ¤ =AD” ¤ +BC” ¤ 에서 ∴ DP”=8-2=6 yy`
2 CD” ¤ =(2'2 )¤ +6¤ , CD” ¤ =22 AP” ¤ +CP” ¤ =BP” ¤ +DP” ¤ 이므로
∴ CD”='ß22 (∵ CD”>0) 'ß22 (2'3)¤ +CP” ¤ =2¤ +6¤ , CP” ¤ =28
∴ CP”=2'7 (∵ CP”>0) yy`
0193 BE” ¤ +CD” ¤ =DE” ¤ +BC” ¤ 이므로 2'7
4¤ +(2'3 )¤ =2¤ +BC” ¤ , BC” ¤ =24
단계 채점요소 배점
∴ BC”=2'6 (∵ BC”>0) ③
BD”의 길이 구하기 20%
AP”의 길이 구하기 20%
0194 BE” ¤ +CD” ¤ =DE” ¤ +BC” ¤ 이므로
DP”의 길이 구하기 30%
6¤ +CD” ¤ =(2'3)¤ +10¤ , CD” ¤ =76 CP”의 길이 구하기 30%
∴ CD”=2'ß19 (∵ CD”>0) 2'ß19

0201 P+Q=R이므로
0195 삼각형의 두 변의 중점을 연결한 선분의 성질에 의해
1
1 1 P+Q+R=2R=2_{ _p_6¤ }
DE”= BC”= _12=6 2
2 2
=36p(cm¤ ) ③
∴ BE” +CD” =DE” ¤ +BC” ¤ =6¤ +12¤ =180
¤ ¤ 180

0202 S¡+S™=( BC”를 지름으로 하는 반원의 넓이)


0196 △ADE에서 DE”="√3¤ +2¤ ='ß13 yy`
1
△ABE에서 BE”="√2¤ +(3+5)¤ ='ß68=2'ß17 yy` = _p_8¤ =32p(cm¤ ) 32p cm¤
2
따라서 BE” ¤ +CD” ¤ =DE” ¤ +BC” ¤ 이므로
BC” ¤ -CD” ¤ =BE” ¤ -DE” ¤ 0203 ( BC”를 지름으로 하는 반원의 넓이)
=(2'ß17)¤ -('ß13)¤ =55 yy` =15p+12p=27p(cm¤ )
55 이므로
1 BC” ¤
단계 채점요소 배점 _p_{ } =27p, BC” ¤ =216
2 2
DE”의 길이 구하기 20%
∴ BC”=6'6(cm) (∵ BC”>0) 6'6 cm
BE”의 길이 구하기 40%
BC” ¤ -CD” ¤ 의 값 구하기 40%
0204 BC”를 지름으로 하는 반원의 반지름의 길이를 r cm라
고 하면
0197 AP”” ¤ +CP”” ¤ =BP”” ¤ +DP”” ¤ 이므로
1
_p_r¤ =8p, r¤ =16 ∴ r=4(cm) (∵ r>0)
5¤ +(2'1å0 )¤ =('1å1 )¤ +DP”” ¤ , DP” ¤ =54 2
∴ DP”=3'6(cm) (∵ DP”>0) ⑤ ∴ BC”=2_4=8(cm)

02. 피타고라스 정리 21
(001~042)개념RPM3-2(정답) 2014.11.24 02:53 PM 페이지22 다민 2540DPI 175LPI

RPM 알피엠

△ABC에서 AB” ="√6¤ +8¤ ='∂100=10(cm) 0212 FC”=x cm라고 하면 DF”=BF”=(8-x) cm이므로
이므로 △DFC에서
1 25 25 (8-x)¤ =x¤ +4¤ , 16x=48 ∴ x=3(cm)
R= _p_5¤ = p(cm¤ ) p cm¤
2 2 2
1
∴ △DFC= _3_4=6(cm¤ ) ④
2
0205 △ABC에서 AB”="√8¤ -4¤ ='ß48=4'3(cm)
∴ (색칠한 부분의 넓이)=△ABC
0213 AQ”=AD”=10 cm이므로
1
= _4'3_4 △ABQ에서 BQ”="√10¤ -8¤ ='ß36=6(cm)
2
∴ CQ”=10-6=4(cm)
=8'3(cm¤ ) 8'3 cm¤
PC”=x cm라고 하면 PQ”=DP”=(8-x) cm이므로

0206 (색칠한 부분의 넓이)=△ABC=30 cm¤ 이므로 △PQC에서


1 (8-x)¤ =4¤ +x¤ , 16x=48
_AB”_5=30 ∴ AB”=12(cm)
2 ∴ x=3(cm) 3 cm
따라서 △ABC에서
BC”="√12¤ +5¤ ='∂169=13(cm) 13 cm 0214 ∠FCA=∠BCA (접은 각)
∠FAC=∠BCA (엇각)
0207 (색칠한 부분의 넓이)=2△ABC ∴ ∠FCA=∠FAC
1 즉, △FAC는 FA”=FC”인 이등변삼각형이다.
=2_{ _8_6}
2
” FC”=x cm라고 하면 FD”=(10-x) cm이므로
F’A=
=48(cm¤ ) 48 cm¤
△FCD에서

0208△ABC에서 AB” ¤ +AC” ¤ =10¤ x¤ =6¤ +(10-x)¤ , 20x=136


이때 AB”=AC”이므로 2AB” ¤ =100, AB” ¤ =50 34
∴ x= (cm) ④
5
∴ AB”=5'2(cm) (∵ AB”>0)
∴ (색칠한 부분의 넓이)=△ABC
0215 ∠FAC=∠DAC (접은 각)
1
= _5'2_5'2 ∠FCA=∠DAC (엇각)
2
=25(cm¤ ) 25 cm¤ ∴ ∠FAC=∠FCA
즉, △AFC는 FA”=FC”인 이등변삼각형이다.
0209 FD”=AD”=15 cm이므로 FC”=F’A”=x cm라고 하면 BF”=(8-x) cm이므로 △ABF
△DFC에서 FC”="√15¤ -9¤ ='∂144=12(cm) 에서
∴ BF”=15-12=3(cm) 25
x¤ =6¤ +(8-x)¤ , 16x=100 ∴ x= (cm)
4
AE”=x cm라고 하면 EF”=AE”=x cm, EB”=(9-x) cm
25
이므로 △EBF에서 ∴ FC”=FA”= cm yy`
4
x¤ =(9-x)¤ +3¤ , 18x=90
△ABC에서 AC”="√6¤ +8¤ ='∂100=10(cm) yy`
∴ x=5(cm) ③
∴ (△AFC의 둘레의 길이)=FA”+FC”+AC”
25 25
0210 PB”=x cm라고 하면 CP”=AP”=(16-x) cm =
4
+
4
+10
△PBC에서 (16-x)¤ =x¤ +8¤
45
32x=192 ∴ x=6(cm) 6 cm = (cm) yy`
2
45
cm
0211 FC”=x cm라고 하면 EF”=AF”=(12-x) cm 2
1 1
EC”= BC”= _12=6(cm)이므로 △FEC에서 단계 채점요소 배점
2 2
FC”, F’A의
” 길이 구하기 60%
(12-x)¤ =6¤ +x¤ , 24x=108
AC”의 길이 구하기 20%
9 9
∴ x= (cm) cm △AFC의 둘레의 길이 구하기 20%
2 2

22 정답과 풀이
(001~042)개념RPM3-2(정답) 2014.11.24 02:53 PM 페이지23 다민 2540DPI 175LPI

● 본문 034쪽 ~ 037쪽 ●

0216 △ABH에서 BH”="√20¤ -12¤ ='∂256=16(cm) 0223 오른쪽 그림에서


P A
∴ CH”=21-16=5(cm) P+Q=△ABC 7 cm Q
4 cm
△AHC에서 AC”="√12¤ +5¤ ='∂169=13(cm) ② ∴ (색칠한 부분의 넓이)
B C
=2△ABC
0217 CD”=x cm라고 하면 BC”=(5+x) cm이므로
=2_{
1
_7_4}=28(cm¤ ) ④
2
△ABC에서
AC” ¤ =(4'5 )¤ -(5+x)¤ =55-10x-x¤ yy ㉠ 0224 ① △EBA와 △ECA는 높이는 같지만 EB”+AC”이
△ADC에서 므로 반드시 △EBA=△ECA라고 할 수 없다.
AC” ¤ =5¤ -x¤ =25-x¤ yy ㉡ 1
②, ⑤ △EBC=△ABF=△LBF= BFML
㉠, ㉡에서 55-10x-x¤ =25-x¤ 2
10x=30 ∴ x=3(cm) 3 cm 1
③ △BCH=△GCA=△GCL= LMGC
2
④ ADEB=AB” ¤ , BFGC=BC” ¤ 이고 AB”+BC”이므로
0218 A’A™”=A’B¡”="√1¤ +1¤ ='2
ADEB+ BFGC ③
A’A£”=A’B™”="√('2)¤ +1¤ ='3
A’A¢”=A’B£”="√('3)¤ +1¤ ='4=2 0225 EF” ¤ =25이므로 EF”=5(cm) (∵ EF”>0)
∴ A’™A¢”=A’A¢”-A’A™”=2-'2 2-'2 BF”=AE”=4 cm이므로
BE”=4+5=9(cm)
0219 AB” ¤ +CD” ¤ =AD” ¤ +BC” ¤ 이므로 △ABE에서 AB”="√4¤ +9¤ ='9å7(cm)
(2'2 )¤ +x¤ =(2'5)¤ +('ß13)¤ ∴ ABCD=AB” ¤ =('9å7 )¤ =97(cm¤ ) 97 cm¤
x¤ =25 ∴ x=5 (∵ x>0)
또한 △OCD에서 0226 OB”="√('2)¤ +('2)¤ ='4=2

y="√5¤ -4¤ ='9=3 OC”="√2¤ +('2)¤ ='6

∴ x+y=5+3=8 8 OD”="√('6)¤ +('2)¤ ='8=2'2


OE”="√(2'2)¤ +('2)¤ ='ß10
0220 AC”를 그으면 △ACD에서
12 cm
A 4 cm
∴ △OFE=
1
_'1å0_'2='5 ②
AC”="√3¤ +4¤ ='2å5=5(cm) D 2
3 cm
B
△ABC에서 5¤ +12¤ =13¤ 13 cm C 0227 △ABC에서 삼각형의 두 변의 중점을 연결한 선분의
즉, AB” ¤ +AC” ¤ =BC” ¤ 이므로 성질에 의해
△ABC는 ∠BAC=90˘인 직각삼각형이다. 1 1
DE”= AC”= _10=5(cm)
∴ ABCD=△ABC+△ACD 2 2
1 1 AE” +CD” =DE” ¤ +AC” ¤ 이므로
¤ ¤
= _12_5+ _4_3
2 2 7¤ +CD” ¤ =5¤ +10¤ , CD” ¤ =76
=30+6=36(cm¤ ) 36 cm¤ ∴ CD”=2'ß19(cm) (∵ CD”>0) 2'ß19 cm

0221AC” ¤ =36이므로 AC”=6(cm) (∵ AC”>0) 0228 ⁄ 빗변의 길이가 x일 때


또한 BC” ¤ =100이므로 BC”=10(cm) (∵ BC”>0) x¤ =4¤ +5¤ , x¤ =41 ∴ x='ß41 (∵ x>0)
△ABC에서 AB”="√10¤ -6¤ ='6å4=8(cm) ¤ 빗변의 길이가 5일 때

1 5¤ =x¤ +4¤ , x¤ =9 ∴ x=3 (∵ x>0)


∴ △ABC= _8_6=24(cm¤ ) 24 cm¤
2 ⁄, ¤에서 x의 값은 3, 'ß41이다. 3, 'ß41

0222 P+Q=R이므로 32p+Q=50p 0229 ㄱ. x=2이면 4¤ >2¤ +3¤ 이므로 둔각삼각형이다. (참)

∴ Q=18p(cm¤ ) ㄴ. x='7이면 4¤ =('7)¤ +3¤ 이므로 직각삼각형이다. (참)

1 AC” ¤ ㄷ. x=4'2이면 (4'2)¤ >3¤ +4¤ 이므로 둔각삼각형이다. (거짓)


_p_{ } =18p, AC” ¤ =144
2 2 ㄹ. x=6이면 6¤ >3¤ +4¤ 이므로 둔각삼각형이다. (참)
∴ AC”=12(cm) (∵ AC”>0) ① 따라서 옳은 것은 ㄱ, ㄴ, ㄹ이다. ㄱ, ㄴ, ㄹ

02. 피타고라스 정리 23
(001~042)개념RPM3-2(정답) 2014.11.24 02:53 PM 페이지24 다민 2540DPI 175LPI

RPM 알피엠

0230 점 A에서 BC”에 내린 수선의


A
6 cm
D
EC”=x cm라고 하면 BE”=DE”=(8-x) cm이므로 △DEC
발을 H라고 하면 에서
1 8 cm (8-x)¤ =x¤ +4¤ , 16x=48 ∴ x=3(cm)
BH”= _(10-6)=2(cm)
2 1
∴ △DEC= _3_4=6(cm¤ ) ②
△ABH에서 B H C 2
10 cm
AH”="√8¤ -2¤ ='6å0=2'1å5(cm)
1 0236 EB”=x cm라고 하면 DE”=AE”=(6-x) cm
∴ ABCD= _(6+10)_2'1å5
2 1 1
BD”= BC”= _6=3(cm)이므로 △EBD에서
=16'1å5(cm¤ ) 16'1å5 cm¤ 2 2
9
(6-x)¤ =x¤ +3¤ , 12x=27 ∴ x= (cm)
4
0231 꼭짓점 A에서 BC”에 내린 수 D
1 9 27 27
선의 발을 L, 그 연장선과 FG”가 만 I ∴ △EBD= _3_ = (cm¤ ) cm¤
E A 2 4 8 8
나는 점을 M이라고 하자. 7 cm H
L
△ABC에서 B C
13 cm
0237 AP” ¤ +CP” ¤ =BP” ¤ +DP” ¤ 이므로
AB”="√13¤ -7¤ ='∂120=2'ß30(cm) 40¤ +70¤ =BP” ¤ +(20'1å0 )¤ , x¤ =2500
1 F G ∴ x=50(m) (∵ x>0)
∴ △ABF=△LBF= BFML M
2
따라서 학교에서 나무 B까지 가는 데 걸리는 시간은
1 1
= ADEB= AB” ¤ 50
2 2 _60_60=60(초) 60초
3000
1
= _(2'ß30)¤ =60(cm¤ ) 60 cm¤
2
0238 △CBE는 BC”=BE”인 직각이등변삼각형이므로
1
0232 AD”가 ∠A의 이등분선이므로 2
BC” ¤ =2, BC” ¤ =4
AB” : AC”=BD” : CD”=10 : 6=5 : 3 ∴ BC”=2(cm) (∵ BC”>0)
AB”=5a, AC”=3a(a>0)라고 하면 △ABC에서 △ABC에서 AB”=øπ2¤ -('3)¤ =1(cm)이고
(5a)¤ =16¤ +(3a)¤ , 16a¤ =256 BD”=CA”='3 cm, DE”=AB”=1 cm이므로
a¤ =16 ∴ a=4 (∵ a>0)
1
ADEC= _(1+'3 )(1+'3 )
∴ AC”=3_4=12 12 2
=2+'3(cm¤ ) ③
0233 일차방정식 3x+4y=12의 그래프에서 x절편은 4,
y절편은 3이므로 0239 ∠ACB=∠ACE(접은 각), ∠ACB=∠EAC(엇각)
O’A”=4, OB”=3 ∴ ∠ACE=∠EAC
△BOA에서 AB”="√3¤ +4¤ ='ß25=5 즉, △ACE는 EA”=EC”인 이등변삼각형이다.
이때 O’A”_OB”=AB”_OH”이므로 AE”=EC”=x cm라고 하면 ED”=(8-x) cm이므로
12 12 △ECD에서
4_3=5_OH” ∴ OH”=
5 5 x¤ =(8-x)¤ +4¤ , 16x=80 ∴ x=5(cm)
1
∴ △ACE= _5_4
0234 AE”=x cm라고 하면 A’'E”=AE”=x cm 2
ED”=(25-x) cm이므로 △A'ED에서 =10(cm¤ ) 10 cm¤
(25-x)¤ =x¤ +15¤ , 50x=400
∴ x=8(cm) 8 cm 0240 삼각형의 세 변의 길이 사이의 관계에 의해
12-9<x<12+9 ∴ 3<x<21
0235 ∠ADB=∠BDE (접은 각) 그런데 x>12이므로
∠ADB=∠DBE (엇각) 12<x<21 yy ㉠ yy`

∴ ∠BDE=∠DBE 또한 ∠C<90˘이므로
즉, △BED는 BE”=DE”인 이등변삼각형이다. x¤ <9¤ +12¤ , x¤ <225

24 정답과 풀이
(001~042)개념RPM3-2(정답) 2014.11.24 02:54 PM 페이지25 다민 2540DPI 175LPI

● 본문 037쪽 ~ 039쪽 ●

∴ 0<x<15 yy`㉡ yy` 직각삼각형에서 빗변의 중점은 외심과 일치하므로


㉠, ㉡에서 12<x<15 yy` 1 1
A’M”=B’M”=C’M”= BC”= _12=6
12<x<15 2 2
∴ MÚH”=6-4=2
단계 채점요소 배점
△AMH에서 A’H”_MÚH”=A’M”_PH”이므로
삼각형의 세 변의 길이 사이의 관계를 이용하여 x의 값의 범위
40% 4'2
구하기 4'2_2=6_PH” ∴ PH”= yy`
3
∠C<90˘일 때 x의 값의 범위 구하기 40%
4'2
x의 값의 범위 구하기 20%
3

단계 채점요소 배점
0241 AB” ¤ +CD” ¤ =AD” ¤ +BC” ¤ 이므로
BC”의 길이 구하기 20%
(2'5 )¤ +5¤ =AD” ¤ +6¤ , AD” ¤ =9
CH”의 길이 구하기 20%
∴ AD”=3 (∵ AD”>0) yy`
A’H의
” 길이 구하기 20%
따라서 △AOD에서 PH”의 길이 구하기 40%
OD”="√3¤ -2¤ ='5이므로 yy`

1
△AOD=
2
_2_'5='5 yy` 0244 삼각형의 세 변의 길이 사이의 관계에 의해
12-5<a<12+5 ∴ 7<a<17 yy`㉠
'5
⁄ a>12일 때, 둔각삼각형이 되려면
단계 채점요소 배점 a¤ >5¤ +12¤ ∴ a>13 (∵ a>0) yy`㉡
AD”의 길이 구하기 40% ㉠, ㉡에서 13<a<17
OD”의 길이 구하기 40% ¤ a<12일 때, 둔각삼각형이 되려면
△AOD의 넓이 구하기 20%
12¤ >5¤ +a¤ ∴ 0<a<'∂119 yy`㉢
㉠, ㉢에서 7<a<'∂119
0242 CH”=x cm라고 하면 ⁄, ¤에서 13<a<17 또는 7<a<'∂119
AC” ¤ =CH”_CB”이므로 따라서 자연수 a의 값은 8, 9, 10, 14, 15, 16이므로 그 합은
(4'3)¤ =x(x+2), x¤ +2x-48=0 8+9+10+14+15+16=72 72
(x+8)(x-6)=0 ∴ x=6 (∵ x>0) yy`

또한 AH” ¤ =BH”_CH”이므로 0245 대각선 BD를 그으면 S¡


A D
AH” ¤ =2_6=12 S¡+S™=△ABD 3
∴ AH”=2'3(cm) (∵ AH”>0) yy` S£+S¢=△DBC S™ 4 S¢
∴ (색칠한 부분의 넓이)=△ABC ∴ (색칠한 부분의 넓이)
1 =△ABD+△DBC B C
= _8_2'3 S£
2
= ABCD
=8'3(cm¤ ) yy`
=3_4=12 12
8'3 cm¤

단계 채점요소 배점 0246 MÚN을


” 그으면 삼각형의 두 A
12
CH”의 길이 구하기 30% 변의 중점을 연결한 선분의 성질에 M
AH”의 길이 구하기 30% 1
의해 MÚN”= AC” B
N
C
색칠한 부분의 넓이 구하기 40% 2
16
1
AC” = x라고 하면 MÚ N ” = x이
2
0243 △ABC에서
고, AMNC에서 두 대각선이 직교하므로
BC”="√(4'6 )¤ +(4'3)¤ ='∂144=12 yy`
A’M” +CN” ¤ =MÚN” ¤ +AC” ¤
¤
AC” ¤ =BC”_CH”이므로
1 ¤ 5
6¤ +8¤ ={ x} +x¤ , x¤ =100, x¤ =80
(4'3 )¤ =12_CH” ∴ CH”=4 yy` 2 4
△AHC에서 A’H=
” "√(4'3)¤ -4¤ ='ß32=4'2 yy` ∴ x=4'5 (∵ x>0) 4'5

02. 피타고라스 정리 25
(001~042)개념RPM3-2(정답) 2014.11.24 02:54 PM 페이지26 다민 2540DPI 175LPI

RPM 알피엠
Ⅱ 피타고라스 정리

0247 점 M, N에서 BC”, AC”에


각각 수선을 긋고 M
A
b
03 피타고라스정리의활용
D
BP”=PQ”=QC”=a, N 6 b
E
AD”=DE”=EC”=b라고 하면
8
b 0248 "√4¤ +2¤ ='ß20=2'5(cm) 2'5 cm
B a P a Q a C
△CNP에서
(2a)¤ +b¤ =8¤ yy ㉠ 0249 "√8¤ +6¤ ='∂100=10(cm) 10 cm

△CMQ에서 a¤ +(2b)¤ =6¤ yy ㉡


㉠+㉡을 하면 5a¤ +5b¤ =100 ∴ a¤ +b¤ =20 0250 '2_3=3'2(cm) 3'2 cm

∴ MÚN”="√a¤ +b¤ ='ß20=2'5 2'5


0251 '2_4=4'2(cm) 4'2 cm

0252 x="√7¤ -3¤ ='ß40=2'ß10 2'ß10

0253 '2x=4'2 ∴ x=4 4

0254
'3
h= _6=3'3(cm)
2
'3
S= _6¤ =9'3(cm¤ )
4
h=3'3 cm, S=9'3 cm¤

0255
'3
h= _2'3=3(cm)
2
'3
S= _(2'3)¤ =3'3(cm¤ )
4
h=3 cm, S=3'3 cm¤

0256
'3
h= _8=4'3(cm)
2
'3
S= _8¤ =16'3(cm¤ )
4
h=4'3 cm, S=16'3 cm¤

0257
'3
h= _4'2=2'6(cm)
2
'3
S= _(4'2)¤ =8'3(cm¤ )
4
h=2'6 cm, S=8'3 cm¤

0258 한 변의 길이를 a라고 하면


'3
a=5'3 ∴ a=10 10
2

0259 한 변의 길이를 a라고 하면


'3
a¤ =4'3, a¤ =16 ∴ a=4 (∵ a>0) 4
4

26 정답과 풀이
(001~042)개념RPM3-2(정답) 2014.11.24 02:54 PM 페이지27 다민 2540DPI 175LPI

● 본문 039쪽 ~ 043쪽 ●

0260 6 : x=1 : 1이므로 x=6 0278 D’M”은 정삼각형 BCD의 높이이므로


6 : y=1 : '2이므로 y=6'2 x=6, y=6'2 '3
D’M”= _6=3'3 3'3
2

0261 x : 4=1 : '2이므로 x=2'2


y : 4=1 : '2이므로 y=2'2 x=2'2, y=2'2 0279 점 H는 △BCD의 무게중심이므로
2 2
D’H=
” D’M”= _3'3=2'3 2'3
3 3
0262 8 : x=2 : 1이므로 x=4
8 : y=2 : '3이므로 y=4'3 x=4, y=4'3
0280 직각삼각형 AHD에서

0263 x : 3'3=2 : '3이므로 x=6 ” øπAD” ¤ -D’H” ¤ =øπ6¤ -(2'3 )¤


A’H=
y : 3'3=1 : '3이므로 y=3 x=6, y=3 ='2å4=2'6 2'6

0264 4'2 : x=1 : '2이므로 x=8 1


0281 (정사면체의 부피)=
3
_△BCD_A’H”
8 : y=2 : '3이므로 y=4'3 x=8, y=4'3
1 '3
= _{ _6¤ }_2'6
3 4
0265 12 : x=2 : '3 ∴ x=6'3
=18'2 18'2
6'3 : y=1 : '2 ∴ y=6'6 x=6'3, y=6'6

0266 OP”="√1¤ +2¤ ='5 '5 0282 한 모서리의 길이가 2'3인 정사면체의 높이는
'6
_2'3=2'2 2'2
3
0267 OQ”="√4¤ +5¤ ='4å1 '4å1

0283 한 모서리의 길이가 12인 정사면체의 부피는


0268 PQ”="√(4-1)¤ +√(5-2)¤ ='ß18=3'2 3'2
'2
_12‹ =144'2 144'2
12
0269 PR”="√(5-1)¤ +√(1-2)¤ ='ß17 'ß17

0284 정사면체의 한 모서리의 길이를 a라고 하면


0270 O’A=
” "√2¤ +(-4)¤ ='ß20=2'5 2'5
'6
a='6 ∴ a=3 3
3
0271 AB”="√(-1-3)¤ +√{-1-(-4)}¤
='ß25=5 5
0285 정사면체의 한 모서리의 길이를 a라고 하면
'2
a‹ =18'2, a‹ =216=6‹ ∴ a=6 6
0272 AB”="√(-2-0)¤ +√(0-5)¤ ='ß29 'ß29 12

0273 AB”="√(4-7)¤ +(√-3-3)¤ 1 1


='ß45=3'5 3'5
0286 BH”=
2
BD”= _6'2=3'2
2
3'2

0274 "√4¤ +8¤ +6¤ ='∂116=2'ß29(cm) 2'ß29 cm 0287 △VBH에서


” øπVB” ¤ -BH” ¤ ="√5¤ -(3'2)¤ ='7
V’H= '7
0275 '3_4=4'3(cm) 4'3 cm

1
0276 "√x¤ +1¤ +2¤ ='ß21이므로 0288 (정사각뿔의 부피)=
3
_ ABCD_V’H”

x¤ +5=21, x¤ =16 ∴ x=4 (∵ x>0) 4 1


= _6¤ _'7
3
=12'7 12'7
0277 '3x=6'2이므로 x=2'6 2'6

03. 피타고라스 정리의 활용 27


(001~042)개념RPM3-2(정답) 2014.11.24 02:54 PM 페이지28 다민 2540DPI 175LPI

RPM 알피엠

0289 BH”=
1 1
BD”= _4'2=2'2 0296 직사각형의 가로와 세로의 길이를 각각 4k cm,
2 2 3k cm(k>0)라고 하면
△VBH에서 "√(4k)¤ +(3k)¤ =20, 5k=20 ∴ k=4(cm)
VH”=øπVB” ¤ -BH” ¤ =øπ6¤ -(2'2 )¤ ='2å8=2'7 따라서 가로의 길이는 4_4=16(cm), 세로의 길이는
1 3_4=12(cm)이므로
∴ (정사각뿔의 부피)= _ ABCD_VH”
3
ABCD=16_12=192(cm¤ ) 192 cm¤
1
= _4¤ _2'7
3
0297 직사각형의 세로의 길이를 a cm라고 하면 가로의 길
32'7
= 이는 (a-3) cm, 대각선의 길이는 (a+3) cm이므로
3
32'7 (a-3)¤ +a¤ =(a+3)¤ , a¤ -12a=0
높이:2'7, 부피:
3 a(a-12)=0 ∴ a=12 (∵ a>3)
따라서 대각선의 길이는 12+3=15(cm) ④
1 1
0290 BH”=
2
BD”= _6=3
2
0298 정사각형의 한 변의 길이를 x cm라고 하면
△VBH에서 (3x)¤ +x¤ =(10'2)¤ , 10x¤ =200 ∴ x¤ =20
VH”=øπVB” ¤ -BH” ¤ ="√8¤ -3¤ ='5å5 ∴ AC”="√(2x)¤ +x¤ ="ç5x¤
1 ='ƒ5_20='∂100
∴ (정사각뿔의 부피)= _ ABCD_VH”
3
=10(cm) 10 cm
1
= _(3'2 )¤ _'5å5
3
=6'5å5
0299 정사각형 ABCD에서 AC”='2_3=3'2
직사각형 ECFG에서
높이:'5å5, 부피:6'5å5
CG”="√(3'6)¤ +(3'2)¤ ='ß72=6'2
∴ AC”+CG”=3'2+6'2=9'2 9'2
0291 (높이)="√5¤ -3¤ ='ß16=4
1
(부피)=
3
_(밑넓이)_(높이) 0300 정사각형의 한 변의 길이를 x cm라고 하면

1 '2x=10 ∴ x=5'2(cm)
= _(p_3¤ )_4
3 따라서 정사각형의 둘레의 길이는
=12p 4_5'2=20'2(cm) ④
높이:4, 부피:12p
0301 정사각형의 한 변의 길이는 원 O의 지름의 길이와 같

0292 (높이)="√8¤ -3¤ ='ß55 으므로 3'2_2=6'2(cm)


1 따라서 정사각형의 대각선의 길이는
(부피)= _(p_3¤ )_'ß55=3'ß55p
3 '2_6'2=12(cm) ③
높이:'ß55, 부피:3'ß55p
0302 정사각형의 한 변의 길이를 a라고 하면
0293 ㈎5 ㈏3 ㈐4 '2a=2'6 ∴ a=2'3 yy`

1
따라서 원의 반지름의 길이가 _2'3='3이므로 yy`
2
0294 구하는 최단 거리는 BH”의 길이와 같으므로
(색칠한 부분의 넓이)=(2'3)¤ -p_('3)¤
BH”="√(5+3)¤ +4¤ ='ß80=4'5 4'5
=12-3p yy`

12-3p
0295 직사각형의 세로의 길이를 x cm라고 하면
"√6¤ +x¤ =2'1å3, 36+x¤ =52 단계 채점요소 배점
x¤ =16 ∴ x=4 (∵ x>0) 정사각형의 한 변의 길이 구하기 40%

따라서 직사각형의 넓이는 원의 반지름의 길이 구하기 30%


색칠한 부분의 넓이 구하기 30%
6_4=24(cm¤ ) ④

28 정답과 풀이
(001~042)개념RPM3-2(정답) 2014.11.24 02:54 PM 페이지29 다민 2540DPI 175LPI

● 본문 043쪽 ~ 046쪽 ●

0303 AC”="√10¤ +5¤ ='∂125=5'5(cm) 0308 정삼각형의 한 변의 길이를 a cm라고 하면


AD”_CD”=AC”_D’H이
” 므로 '3
a¤ =9'3, a¤ =36
10_5=5'5_D’H” 4

∴ D’H”=2'5(cm) 2'5 cm ∴ a=6(cm)(∵ a>0)


'3
∴ (정삼각형의 높이)= _6
2
0304 AC”="√16¤ +12¤ ='∂400=20
=3'3(cm) ③
DC” ¤ =CH”_C’A”이므로
36
12¤ =CH”_20 ∴ CH”=
5
3 3
AD”_CD”=AC”_D’H이
” 므로 0309 AD”=
2
AG”= _4=6(cm)
2
yy`

48 정삼각형 ABC의 높이가 6 cm이므로 정삼각형 ABC의 한


16_12=20_D’H” ∴ D’H=

5
변의 길이를 a cm라고 하면
36 48 84
∴ CH”+D’H=
” + = ④
5 5 5 '3
a=6 ∴ a=4'3(cm) yy`
2

0305
'3
BD”="√9¤ +12¤ ='∂225=15(cm) ∴ △ABC= _(4'3)¤
4
AB” ¤ =BP”_BD”이므로
=12'3(cm¤ ) yy`
27 12'3 cm¤
9¤ =BP”_15 ∴ BP”= (cm)
5
이때 △ABP≡△CDQ(RHA 합동)이므로 단계 채점요소 배점
27 AD”의 길이 구하기 30%
DQ”=BP”= cm
5 △ABC의 한 변의 길이 구하기 40%
27 △ABC의 넓이 구하기 30%
∴ PQ”=15-2_
5
21 21
= (cm) cm
5 5
1
0310 BE”=EC”=CF”=
2
_8=4(cm)
0306 AC”="√(2'3)¤ +2¤ ='ß16=4(cm) △GEC는 한 변의 길이가 4 cm인 정삼각형이므로
AD”_CD”=AC”_DF”이므로 '3
△GEC= _4¤ =4'3(cm¤ )
2'3_2=4_DF” ∴ DF”='3(cm) 4
△DFC에서 CF”="√2¤ -('3)¤ =1(cm) '3
이때 △ABC= _8¤ =16'3(cm¤ )이므로
이때 △ABE™△CDF(RHA 합동)이므로 4

AE”=CF”=1 cm (색칠한 부분의 넓이)=2_(△ABC-△GEC)

∴ EF”=4-(1+1)=2(cm) =2_(16'3-4'3)

1 =24'3(cm¤ ) 24'3 cm¤


∴ △DEF= _2_'3
2 다른풀이
='3(cm¤ ) '3 cm¤ 색칠한 부분의 넓이는 오른쪽 그림 A D
과 같이 한 변의 길이가 4 cm인 정 G
8 cm
0307 △ABC의 한 변의 길이를 a cm라고 하면 삼각형 6개의 넓이와 같다.
'3 따라서 색칠한 부분의 넓이는 B F
a¤ =18'3, a¤ =72 4 cm E C
4 '3
6_{ _4¤ }=24'3(cm¤ )
∴ a=6'2(cm)(∵ a>0) 4
'3
AD”= _6'2=3'6(cm)이므로
2
'3 0311 주어진 정육각형은 한 변의 길이가 8 cm인 정삼각형 6
△ADE= _(3'6)¤
4 개로 이루어져 있으므로 구하는 넓이는
27'3 27'3 '3
= (cm¤ ) cm¤ 6_{ _8¤ }=96'3(cm¤ ) ③
2 2 4

03. 피타고라스 정리의 활용 29


(001~042)개념RPM3-2(정답) 2014.11.24 02:54 PM 페이지30 다민 2540DPI 175LPI

RPM 알피엠

0312 정육각형의 한 변의 길이를 a cm라고 하면 0316 꼭짓점 A에서 BC”에 내린 A


'3 수선의 발을 H라고 하면 10 cm 10 cm
6_ a¤ =24'3, a¤ =16
4
1
BH”=CH”= _14=7(cm) H
∴ a=4(cm)(∵ a>0) 2 B C
14 cm
따라서 정육각형의 둘레의 길이는 △ABH에서
6_4=24(cm) ② A’H=
” "√10¤ -7¤ ='ß51(cm)
1
∴ △ABC= _14_'ß51
2
0313 AC”를 그으면 △ABC와 A
△ACD는 정삼각형이다. =7'ß51(cm¤ ) 7'ß51 cm¤
60˘
yy` B 60˘ D
60˘
이때 마름모의 한 변의 길이를 0317 BC”=2BH”=2_5=10(cm)이고
C
a cm라고 하면 △ABC의 넓이가 15 cm¤ 이므로
1
2_
'3
a¤ =32'3 _10_AH”=15 ∴ AH”=3(cm)
4 2
a¤ =64 ∴ a=8(cm)(∵ a>0) yy` 이때 점 G는 △ABC의 무게중심이므로
따라서 마름모의 둘레의 길이는 1 1
GH”= AH”= _3=1(cm)
3 3
4_8=32(cm) yy`
따라서 △GBH에서
32 cm
BG”="√5¤ +1¤ ='2å6(cm) '2å6 cm
단계 채점요소 배점
△ABC와 △ACD가 정삼각형임을 알기 20%
0318 이등변삼각형 ABC의 둘레의 길이가 32 cm이고
마름모의 한 변의 길이 구하기 60%
BC”=12 cm이므로
마름모의 둘레의 길이 구하기 20%
1
AB”=AC”= _(32-12)=10(cm)
2

0314 △OCD는 높이가 3 cm인 정삼각형이므로 △OCD의 꼭짓점 A에서 BC”에 내린 수선의 발 A

한 변의 길이를 a cm라고 하면 을 H라고 하면


10 cm
1
'3
a=3 ∴ a=2'3(cm) BH”=CH”= _12=6(cm)
2 2 B C
H
'3 △ABH에서 12 cm
∴ △OCD= _(2'3)¤ =3'3(cm¤ )
4 A’H=
” "√10¤ -6¤ ='ß64=8(cm)
따라서 정육각형의 넓이는 1
∴ △ABC= _12_8
6△OCD=6_3'3 2
=18'3(cm¤ ) 18'3 cm¤ =48(cm¤ ) 48 cm¤

0315 ∠B=∠C이므로 △ABC는 A 0319 꼭짓점 A에서 BC”에 내린 A

AB”=AC”인 이등변삼각형이다. 수선의 발을 H라고 하자. 20


13
꼭짓점 A에서 BC”에 내린 수선의 발 BH”=x라고 하면
을 H라고 하면 △ABC의 넓이가 B C CH”=21-x B C
H x H 21-x
8 cm △ABH에서
8'5 cm¤ 이므로
1 A’H” ¤ =13¤ -x¤ yy ㉠
_8_A’H”=8'5 ∴ A’H=
” 2'5(cm)
2 △AHC에서 A’H” ¤ =20¤ -(21-x)¤ yy ㉡
1 ㉠, ㉡에서 13¤ -x¤ =20¤ -(21-x)¤
BH”= _8=4(cm)이므로 △ABH에서
2
42x=210 ∴ x=5
AB”="√4¤ +(2'5)¤ ='ß36=6(cm)
∴ A’H=
” "√13¤ -5¤ ='∂144=12
따라서 △ABC의 둘레의 길이는
1
AB”+BC”+C’A”=6+8+6=20(cm) 20 cm ∴ △ABC= _21_12=126 ⑤
2

30 정답과 풀이
(001~042)개념RPM3-2(정답) 2014.11.24 02:54 PM 페이지31 다민 2540DPI 175LPI

● 본문 046쪽 ~ 048쪽 ●

0320 BH”=x라고 하면 A 0324 △DBC에서 BD” : BC”=2 : '3이므로


CH”=6-x 4 5 BD” : 6=2 : '3 ∴ BD”=4'3(cm)
△ABH에서 △ABD에서 AB” : BD”=1 : '2이므로
A’H” ¤ =4¤ -x¤ yy ㉠ B
x H 6-x
C AB” : 4'3=1 : '2 ∴ AB”=2'6(cm)
△AHC에서 이때 △ABD는 AB”=AD”인 직각이등변삼각형이므로
A’H” ¤ =5¤ -(6-x)¤ yy ㉡ AD”=AB”=2'6 cm
㉠, ㉡에서 4¤ -x¤ =5¤ -(6-x)¤ 1
∴ △ABD= _2'6_2'6
2
9
12x=27 ∴ x=
4 =12(cm¤ ) 12 cm¤
9 ¤ 175 5'7 5'7
∴ A’H”=æ4¤ ≠-{ } =Æ… =
4 16 4 4
0325 △ABC에서 AB” : AC”=2 : 1이므로
12 : AC”=2 : 1 ∴ AC”=6(cm) yy`
0321CH”=x cm라고 하면 BH”=(8-x) cm 1 1
△ADC에서 ∠CAD= ∠A= _60˘=30˘ yy`
△ABH에서 A’H” ¤ =9¤ -(8-x)¤ yy ㉠ 2 2
△AHC에서 A’H” ¤ =7¤ -x¤ yy ㉡ AD” : AC”=2 : '3, AD” : 6=2 : '3
㉠, ㉡에서 9¤ -(8-x)¤ =7¤ -x¤ ∴ AD”=4'3(cm) yy`

16x=32 ∴ x=2(cm) yy` 4'3 cm


따라서 A’H=
” "√7¤ -2¤ ='ß45=3'5(cm)이고
단계 채점요소 배점
H’M”=C’M”-CH”=4-2=2(cm)이므로 yy`
AC”의 길이 구하기 40%
△AMH에서 ∠CAD의 크기 구하기 20%
A’M=
” "√(3'5 )¤ +2¤ ='ß49=7(cm) yy` AD”의 길이 구하기 40%
7 cm

단계 채점요소 배점 0326 AC”가 정사각형 ABCD의 대각선이므로


CH”의 길이 구하기 50% ∠BCA=45˘ ∴ ∠ACE=90˘
A’H,” H’M”의 길이 구하기 30%
정삼각형 AEF에서 AC”는 높이이므로
A’M의
” 길이 구하기 20%
'3
AC”= _12=6'3(cm)
2
△ABC에서 AB” : AC”=1 : '2이므로
0322 꼭짓점 A에서 BC”의 연장선 A
AB” : 6'3=1 : '2
위에 내린 수선의 발을 H라고 하자.
'∂13
h ∴ AB”=3'6(cm) 3'6 cm
” h라고 하면
CH”=x, A’H= '5
△ABH에서 B H
h¤ =('ß13 )¤ -(2+x)¤ yy ㉠
2 C x
0327 꼭짓점 A에서 BC”에 내린 A
수선의 발을 H라고 하면 12
△ACH에서
△ABH에서 30˘ 45˘
h¤ =('5 )¤ -x¤ yy ㉡ B
H
C
㉠, ㉡에서 ('ß13)¤ -(2+x)¤ =('5)¤ -x¤ AB” : A’H=
” 2 : 1이므로
4x=4 ∴ x=1 12 : A’H=
” 2:1 ∴ A’H=
” 6
∴ h="√('5 )¤ -1¤ ='4=2 △AHC에서 A’H” : AC”=1 : '2이므로
6 : AC”=1 : '2
1
∴ △ABC= _2_2=2 2
2 ∴ AC”=6'2 ①

0323 △ABC에서 AB” : BC”=1 : '2이므로 0328 꼭짓점 A에서 BC”에 내린 수 A D


3 : BC”=1 : '2 ∴ BC”=3'2(cm) 선의 발을 H라고 하면 △ABH에서 8
△DBC에서 BC” : CD”='3 : 1이므로 AB” : BH”=2 : 1이므로 60˘
B C
3'2 : CD”='3 : 1 ∴ CD”='6(cm) ④ 8 : BH”=2 : 1 ∴ BH”=4 H

03. 피타고라스 정리의 활용 31


(001~042)개념RPM3-2(정답) 2014.11.24 02:54 PM 페이지32 다민 2540DPI 175LPI

RPM 알피엠

또한 AB” : A’H=
” 2 : '3이므로 1 1
∠BAE= ∠A= _90˘=30˘이므로
8 : A’H=
” 2 : '3 ∴ A’H=
” 4'3 3 3

HC”=AD”=8이므로 BC”=4+8=12 △ABE에서 AB” : BE”='3 : 1

1 9 : BE”='3 : 1 ∴ BE”=3'3(cm)
∴ ABCD= _(8+12)_4'3=40'3 ③
2 ∴ EC”=BC”-BE”=9-3'3(cm) (9-3'3 ) cm

0329 꼭짓점 A에서 BC”의 연장선 A 0333 △ABC에서 AB” : BC”=1 : 2이므로
6 cm AB” : 8=1 : 2 ∴ AB”=4(cm) yy`
위에 내린 수선의 발을 H라고 하면
∠ACH=180˘-120˘=60˘ 또한 AC” : BC”='3 : 2이므로
120˘ 60˘ AC” : 8='3 : 2 ∴ AC”=4'3(cm) yy`
△ACH에서 B
5 cm C H
AC” : A’H=
” 2 : '3이므로 ∴ (색칠한 부분의 넓이)=△ABC
6 : A’H=
” 2 : '3 ∴ A’H=
” 3'3(cm) 1
∴ (색칠한 부분의 넓이)= _4_4'3
2
또한 AC” : CH”=2 : 1이므로
=8'3(cm¤ ) yy
6 : CH”=2 : 1 ∴ CH”=3(cm)
8'3 cm¤
따라서 △ABH에서
AB”="√(5+3)¤ +(3'3)¤ ='ß91(cm) 'ß91 cm 단계 채점요소 배점
AB”의 길이 구하기 30%
AC”의 길이 구하기 30%
0330 두 꼭짓점 A, D에서 BC” A 6 cm D
색칠한 부분의 넓이 구하기 40%
에 내린 수선의 발을 각각 E, F라 4"6 cm
고 하자. 45˘ 60˘
△ABE에서 B E F C 0334 △ABP에서 AP” : AB”=1 : 2이므로
AP” : 2=1 : 2 ∴ AP”=1(cm)
AB” : BE” : AE”='2 : 1 : 1이므로
또한 AB” : BP”=2 : '3이므로 2 : BP”=2 : '3
4'6 : BE” : AE”='2 : 1 : 1
∴ BP”='3(cm)
∴ BE”=AE”=4'3(cm)
이때 BQ”=AP”=1 cm이므로
DF”=AE”=4'3 cm이고
PQ”=BP”-BQ”='3-1(cm)
△DFC에서 CF” : DF”=1 : '3이므로
따라서 PQRS가 정사각형이므로
CF” : 4'3=1 : '3 ∴ CF”=4(cm)
PQRS=PQ” ¤ =('3-1)¤ =4-2'3(cm¤ )
1
∴ ABCD= _{6+(4'3+6+4)}_4'3 (4-2'3 ) cm¤
2
=24+32'3(cm¤ )
(24+32'3) cm¤ 0335 두 점 사이의 거리를 각각 구하면
① "√(2-1)¤ +√(4-2)¤ ='5

0331 정팔각형의 한 외각의 크기는 45˘ ② "√3¤ +2¤ ='ß13


360˘ a cm ③ "√(0-2)¤ +(√-5-0)¤ ='ß29
=45˘이므로 잘라 낸 직각이등 4 cm
8 ④ "√{2-(-2)}¤ √+(-2-1)¤ ='ß25=5
변삼각형에서 직각을 낀 한 변의 길이 4 cm
⑤ "√{-4-(-3)√}¤ +{-7-(-1)}¤ ='ß37
를 a cm라고 하면 따라서 두 점 사이의 거리가 가장 긴 것은 ⑤이다. ⑤
a : 4=1 : '2 ∴ a=2'2(cm)
따라서 처음 정사각형의 한 변의 길이는
0336 ⑴ AB”="√(a-2)¤ +(√-1-2)¤ =3'5이므로
4+2a=4+2_2'2
(a-2)¤ +(-3)¤ =(3'5)¤
=4+4'2(cm) (4+4'2) cm
a¤ -4a-32=0, (a+4)(a-8)=0
∴ a=-4 또는 a=8
0332 ABCD의 한 변의 길이를 a cm라고 하면 그런데 점 B는 제3사분면 위의 점이므로 a<0
a¤ =81 ∴ a=9(cm)(∵ a>0) ∴ a=-4

32 정답과 풀이
(001~042)개념RPM3-2(정답) 2014.11.24 02:54 PM 페이지33 다민 2540DPI 175LPI

● 본문 048쪽 ~ 050쪽 ●

⑵ AC”="√{2-(-1)}¤ +√(3-5)¤ 0340 AB”="√(-2-0)¤ +√(4-2)¤


='ß13 ='8=2'2
BD”="√{(1-a)-1}¤ √+(1-a)¤ BC”="√{-1-(-2)}¤ √+(-4-4)¤
="√2a¤ -2a+1 ='ß65
이때 AC”=BD”이므로 CA”="√{0-(-1)}¤ +√{2-(-4)}¤
13=2a¤ -2a+1, 2a¤ -2a-12=0 ='ß37
a¤ -a-6=0, (a-3)(a+2)=0 따라서 BC”가 가장 긴 변이고 BC” ¤ >AB” ¤ +C’A” ¤ 이므로
∴ a=-2 (∵ a<0) △ABC는 둔각삼각형이다. ③
⑶ 점 P는 x축 위의 점이므로 y
P(a, 0)이라고 하면 AP”=BP”이므 7 B
0341 ① AB”="√{7-(-1)}¤ +√(8-2)¤ ='∂100=10
로 ② BC”="√(5-7)¤ √+(-6-8)¤ ='∂200=10'2
A 1
"√{a-(-2)}¤ +(0-1ç)¤ ③ CA”="√(-1-5)¤ +√{2-(-6)}¤ ='∂100=10
-2 O 4P x
="√(a-4)¤ +(0-7)¤ ④ AB”=C’A이” 고 BC” ¤ =AB” ¤ +C’A” ¤ 이므로 △ABC는
(a+2)¤ +1¤ =(a-4)¤ +7¤ ∠A=90˘인 직각이등변삼각형이다.
12a=60 ∴ a=5 1 1
⑤ △ABC= _AB”_C’A”= _10_10=50
2 2
∴ P(5, 0)
따라서 옳지 않은 것은 ③이다. ③
⑴ -4 ⑵ -2 ⑶ P(5, 0)

0342 △ABC는 ∠A=90˘이므로 BC”가 빗변인 직각삼각형


1
0337 y= x¤ 의 그래프와 y=x+4의 그래프가 두 점 A,
2
이다.
이때 BC” ¤ =AB” ¤ +AC” ¤ 이므로
B에서 만나므로
{3-(-1)}¤ +{7-(-5)}¤
1
x¤ =x+4, x¤ -2x-8=0 ={(-1-a)¤ +(-5-3)¤ }+{(3-a)¤ +(7-3)¤ }
2
(x+2)(x-4)=0 ∴ x=-2 또는 x=4 a¤ -2a-35=0, (a-7)(a+5)=0

x=-2일 때 y=2이므로 A(-2, 2) ∴ a=7 또는 a=-5

x=4일 때 y=8이므로 B(4, 8) 따라서 모든 a의 값의 합은


7+(-5)=2 2
∴ AB”="√{4-(-2)}¤ +√(8-2)¤
='ß72=6'2 6'2
0343 점 C를 AB”에 대하여 대칭
C
D
이동한 점을 C'이라고 하면 5
4
0338 y=x¤ -7x+17의 그래프와 y=x+2의 그래프가 두 CP”=C’'P”이므로 A
P
B
9

점 P, Q에서 만나므로 CP”+DP”=C’'P”+DP”


x¤ -7x+17=x+2, x¤ -8x+15=0 æC’'D” C' 12 D'

(x-3)(x-5)=0 ∴ x=3 또는 x=5 이때 점 C'에서 DB”의 연장선에 내린 수선의 발을 D'이라고


x=3일 때 y=5, x=5일 때 y=7이므로 하면 △DC'D'에서
P(3, 5), Q(5, 7) 또는 P(5, 7), Q(3, 5) C’'D”="√12¤ +9¤ ='∂225=15
∴ PQ”="√(5-3)¤ +√(7-5)¤ 따라서 CP”+DP”의 최솟값은 15이다. 15
='8=2'2 2'2

0344 점 A를 x축에 대하여 대칭이동 y


4 B
0339 AB”="√{3-(-2)}¤ √+(-5-2)¤ ='ß74 한 점을 A'이라고 하면 A
2
BC”="√(7-3)¤ +{√-1-(-5)}¤ ='ß32=4'2 A'(1, -2)
CA”="√(-2-7)¤ +√{2-(-1)}¤ ='ß90=3'ß10 따라서 AP”+BP”의 최솟값은 O 1 P 5 x
-2
따라서 C’A”가 가장 긴 변이고 C’A” ¤ <AB” ¤ +BC” ¤ 이므로 A’'B”="√(5-1)¤ +√{4-(-2)}¤ A'

△ABC는 예각삼각형이다. ⑤ ='ß52=2'ß13 ③

03. 피타고라스 정리의 활용 33


(001~042)개념RPM3-2(정답) 2014.11.24 02:54 PM 페이지34 다민 2540DPI 175LPI

RPM 알피엠

0345 점 A를 y축에 대하여 대칭이 y AB” ¤ +BC” ¤ =AC” ¤ 에서 (x¤ +100)+(x¤ +100)=(2x)¤
동한 점을 A'이라고 하면 4 x¤ =100 ∴ x=10(cm)(∵ x>0)
A' A
A'(-2, 4) ∴ AC”=2x=2_10=20(cm) 20 cm
P 2 5
따라서 AP”+BP”의 최솟값은 -2 O x
-2 B
A’'B”="√{5-(-2)}¤ √+(-2-4)¤ 0351 정육면체의 한 모서리의 길이를 a cm라고 하면
='ß85 'ß85 EG”="√a¤ +a¤ ='2a(cm)이므로
1
△AEG= _AE”_EG”
2
0346 A마을을 직선 도로에 대 A 6 km
B 1
하여 대칭이동한 곳을 A'이라고 9 km = _a_'2a
P 2
하면 A마을에서 버스 정류장(P
15 km '2
지점)을 거쳐 B마을로 가는 최 = a¤
A' 2
단 거리는 20 km
'2
따라서 a¤ =18'2이므로
A’'B”="√20¤ +15¤ ='∂625=25(km) 25 km 2
a¤ =36 ∴ a=6(cm)(∵ a>0)

0347 ” x cm라고 하면
D’H= ∴ AG”='3a=6'3(cm) ①

"√5¤ +3¤ +x¤ =5'2, 34+x¤ =50


x¤ =16 ∴ x=4(cm)(∵ x>0) ④ 0352 정육면체의 한 모서리의 길이를 a cm라고 하면
'3a=3'2 ∴ a='6(cm)
따라서 구하는 정육면체의 부피는
0348 AD”=x cm라고 하면
'6_'6_'6=6'6(cm‹ ) ③
"√x¤ +4¤ +5¤ =5'2, x¤ +41=50
x¤ =9 ∴ x=3(cm)(∵ x>0) yy`

△DGH에서 DG”="√4¤ +5¤ ='ß41(cm) yy` 0353 정육면체의 한 모서리의 길이를 a cm라고 하면
따라서 AFGD는 가로의 길이가 3 cm, 세로의 길이가 '3a=4'3 ∴ a=4(cm)
'ß41 cm인 직사각형이므로 ∴ FH”='2_4=4'2(cm)
AFGD=3_'ß41=3'ß41(cm¤ ) yy` 따라서 BF”⊥FH”이므로
3'ß41 cm¤ 1
△BFH= _BF”_FH”
2
단계 채점요소 배점 1
= _4_4'2
AD”의 길이 구하기 50%
2
DG”의 길이 구하기 20% =8'2(cm¤ ) 8'2 cm¤
AFGD의 넓이 구하기 30%

0354 AMGN에서 A 10 cm
D
A’M=
” MÚG”=G’N”=N’A”
0349 직육면체의 세 모서리의 길이를 k, 2k, 3k(k>0)라고 B
C N
="√5¤ +10¤
하면 M E
='∂125 H
(직육면체의 부피)=k_2k_3k=48
=5'5(cm) F
G
6k‹ =48, k‹ =8
이므로 AMGN은 마름모이다. yy`
∴ k=2
MÚN”=FH”='2_10=10'2(cm)이고 yy`
따라서 직육면체의 세 모서리의 길이는 각각 2 cm, 4 cm,
AG”='3_10=10'3(cm)이므로 yy`
6 cm이므로 대각선의 길이는
1
"√2¤ +4¤ +6¤ ='5å6=2'1å4(cm) 2'1å4 cm AMGN= _MÚN”_AG”
2
1
AMGN= _10'2_10'3
0350 한 직육면체의 가로의 길이를 x cm라고 하면 2
AB”=BC”="√x¤ +6¤ +8¤ ="√x¤ +100 =50'6(cm¤ ) yy`

이때 AC”=x+x=2x이고 ∠ABC=90˘이므로 50'6 cm¤

34 정답과 풀이
(001~042)개념RPM3-2(정답) 2014.11.24 02:54 PM 페이지35 다민 2540DPI 175LPI

● 본문 050쪽 ~ 052쪽 ●

단계 채점요소 배점 0359 정사면체의 한 모서리의 길이를 a cm라고 하면


AMGN이 마름모임을 알기 30% '6
a=6 ∴ a=3'6(cm)
MÚN의
” 길이 구하기 20%
3
AG”의 길이 구하기 20% '2
∴ (부피)= _(3'6)‹ =27'3(cm‹ ) 27'3 cm‹
AMGN의 넓이 구하기 30% 12

0360 ⑴ AH”는 한 모서리의 길이가 A


0355 △AFC는 AF”=FC”=C’A”='2_4=4'2(cm)인 정
9 cm인 정사면체의 높이이므로
삼각형이므로 9 cm
'6
AH”= _9=3'6(cm) D
△AFC=
'3
_(4'2)¤ =8'3(cm¤ ) 3
4 B
이때 DH”의 연장선이 BC”와 만나는 H
E
BI”=h cm라고 하면
점을 E라고 할 때, DE”는 정삼각형 C
1 1
_△AFC_h= _△ABC_BF” BCD의 높이이므로
3 3
'3 9'3
1 1 1 DE”= _9= (cm)
_8'3_h= _{ _4_4}_4 2 2
3 3 2
한편 점 H는 △BCD의 무게중심이므로
8'3 32 4'3 4'3
h= ∴ h= (cm) cm 2 2 9'3
3 3 3 3 DH”= DE”= _ =3'3(cm)
3 3 2

0356 ⑴ BG”=GD”=DB”='2_12=12'2(cm)이므로 ∴ △AHD=


1
_3'3_3'6=
27'2
(cm¤ )
2 2
△BGD는 한 변의 길이가 12'2 cm인 정삼각형이다.
⑵ 정사면체의 부피는
'3
∴ △BGD= _(12'2)¤ =72'3(cm¤ ) 243'2
4 '2
_9‹ = (cm‹ )
12 4
⑵ AE”=øπ(3'3 )¤ +6¤ ='6å3=3'7(cm)
27'2 243'2
AF”="√6¤ +6¤ ='7å2=6'2(cm) ⑴ cm¤ ⑵ cm‹
2 4
이때 (6'2 )¤ =(3'7)¤ +3¤
즉, AF” ¤ =AE” ¤ +EF” ¤ 이므로 △AEF는 ∠AEF=90˘인
0361
'3
MÚB=
” MÚC”= _4 M
직각삼각형이다. 2

1 1 =2'3(cm) 2'3 cm 2'3 cm


∴ △AEF= _AE”_EF”= _3'7_3
2 2 점 M에서 BC”에 내린 수선의 발을
9'7 H라고 하면 B C
= (cm¤ ) 2 cm H
2
1
BH”=CH”= BC”
9'7 2
⑴ 72'3 cm¤ ⑵ cm¤
2 1
= _4=2(cm)
2
0357 AG”="√2¤ +2¤ +(2'2)¤ ='ß16=4(cm) △MBH에서 MÚH”="√(2'3)¤ -2¤ ='8=2'2(cm)
EG”="√2¤ +2¤ ='8=2'2(cm) 1
∴ △MBC= _BC”_MÚH”
△AEG에서 AE”_EG”=AG”_EI”이므로 2
2'2_2'2=4_EI” ∴ EI”=2(cm) 2 cm 1
= _4_2'2=4'2(cm¤ ) 4'2 cm¤
2

0358 C’M”=MÚF”=x cm라고 하면


0362
'6
△ABE에서 AE”="√6¤ +(2x)¤ ="√36+4x¤ ① A’H=
” _2'3=2'2(cm)
3
△AMC에서 A’M=
” "√12¤ +x¤ ="√144+x¤ '3
② D’M”= _2'3=3(cm)
이때 △AEM은 정삼각형이므로 AE”=A’M에
” 서 2

"√36+4x¤ ="√144+x¤ , 36+4x¤ =144+x¤ ③ 점 H는 △BCD의 무게중심이므로

3x¤ =108, x¤ =36 ∴ x=6(cm)(∵ x>0) 1 1


MÚH”= D’M”= _3=1(cm)
3 3
따라서 C’M= ” 6 cm이므로 이 삼각기둥의 높이는
” MÚF=
1 1
C’M”+MÚF”=6+6=12(cm) 12 cm ④ △AMH= _MÚH”_A’H”= _1_2'2='2(cm¤ )
2 2

03. 피타고라스 정리의 활용 35


(001~042)개념RPM3-2(정답) 2014.11.24 02:54 PM 페이지36 다민 2540DPI 175LPI

RPM 알피엠

'2 따라서 MBCN은 오른쪽 그림 M 2 cm N


⑤ (부피)= _(2'3)‹ =2'6(cm‹ ) ③
12 과 같은 등변사다리꼴이므로
2'3 cm 2'3 cm
PQ”=MÚN=
” 2 cm
0363 주어진 전개도로 만들어지는 A 1 cm
1 B C
정사각뿔은 오른쪽 그림과 같다. h cm BP”=CQ”= _(4-2)=1(cm) P Q
9 cm 2 4 cm
BD”='2_6=6'2(cm)이므로 △MBP에서
E D
1 MÚP”="√(2'3)¤ -1¤ ='ß11(cm)
BH”= BD”=3'2(cm) H 6 cm
2
B 6 cm C 1
△ABH에서 ∴ MBCN= _(2+4)_'ß11
2
A’H=
” "√9¤ -(3'2)¤ ='ß63=3'7(cm) =3'ß11(cm¤ ) 3'ß11 cm¤
1
∴ (부피)= _6¤ _3'7
3 0367 밑면의 반지름의 길이를 r cm라고 하면
=36'7(cm‹ ) ④ 2pr=6p ∴ r=3(cm)
∴ (높이)="√6¤ -3¤ ='ß27=3'3(cm)
0364 꼭짓점 A에서 BCDE에 A 1
∴ (부피)= _p_3¤ _3'3=9'3p(cm‹ )
내린 수선의 발을 H라고 하면 3 cm 3
E
BD”='2_3=3'2(cm)이므로 D 9'3p cm‹
1 3'2 B H
H’D”=
2
BD”=
2
(cm) C 0368 ⑴ 밑면의 반지름의 길이를 r cm
라고 하면
△AHD에서 10 cm
pr¤ =36p, r¤ =36
3'2 ¤ 3'2
A’H=
” æ≠3¤ -{ }= (cm) ∴ r=6(cm)(∵ r>0)
2 2
∴ (정팔면체의 부피)=2_(정사각뿔의 부피) 이때 원뿔의 높이는 r cm

1 3'2 "√10¤ -6¤ ='ß64=8(cm)이므로


=2_{ _3¤ _ }
3 2 원뿔의 부피는
=9'2(cm‹ ) 9'2 cm‹ 1
_36p_8=96p(cm‹ )
3
0365 BD”='2_4=4'2이므로 ⑵ ∠HBO=45˘이므로 O

1 BH”:OH”:6=1:1:'2에서 6 cm
BH”= BD”=2'2 45˘
2 BH”=OH”=3'2 cm
45˘
△ABH에서 A’H=
” "√8¤ -(2'2)¤ ='ß56=2'ß14 따라서 원뿔의 부피는 A B
H
1 32'ß14 1
∴ (부피)= _4¤ _2'ß14= _p_(3'2)¤ _3'2=18'2p(cm‹ )
3 3 3
또한 꼭짓점 A에서 BC”에 내린 수선의 발을 A ⑴ 96p cm‹ ⑵ 18'2p cm‹
M이라고 하면
0369 AO”=BO”=5 cm이므로
A’M=
” "√8¤ -2¤ ='ß60=2'ß15
8 OH”=A’H”-AO”=9-5=4(cm) yy`
1
∴ △ABC= _4_2'ß15=4'ß15 △OHB에서
2
∴ (겉넓이)=4_4+4'ß15_4 B C HB”="√5¤ -4¤ ='9=3(cm) yy`
2 M
=16(1+'ß15) 따라서 원뿔의 부피는
32'ß14 1
부피: , 겉넓이:16(1+'ß15) _p_3¤ _9=27p(cm‹ ) yy`
3 3
27p cm‹
0366 △AED에서
단계 채점요소 배점
1 1
MÚN”= ED”= _4=2(cm)이고 OH”의 길이 구하기 30%
2 2
HB”의 길이 구하기 30%
'3
B’M”=CN”= _4=2'3(cm) 원뿔의 부피 구하기 40%
2

36 정답과 풀이
(001~042)개념RPM3-2(정답) 2014.11.24 02:54 PM 페이지37 다민 2540DPI 175LPI

● 본문 052쪽 ~ 054쪽 ●

0370 △ABC에서 AB” : AC”=2 : '3이므로 0374 원뿔의 모선의 길이는


8 cm x˘
8 : AC”=2 : '3 ∴ AC”=4'3(cm) "√2¤ +(2'ß15)¤ ='ß64=8(cm)
또한 AB” : BC”=2 : 1이므로 오른쪽의 전개도에서 부채꼴의 중심각의
8 : BC”=2 : 1 ∴ BC”=4(cm) 크기를 x˘라고 하면 부채꼴의 호의 길이
2 cm
이때 △ABC를 직선 l을 축으로 하여 A 는 밑면인 원의 둘레의 길이와 같으므로
30˘
1회전시킬 때 생기는 입체도형은 오른 4'3 cm x
2p_8_ =2p_2 ∴ x˘=90˘
쪽 그림과 같다. 8 cm 360
따라서 입체도형의 부피는 90˘
B 4 cm C
1 64'3
3
_p_4¤ _4'3=
3
p(cm‹ ) 0375 단면인 원의 반지름의 길이는
"√10¤ -6¤ ='ß64=8(cm)
64'3
p cm‹ 따라서 단면인 원의 넓이는
3
p_8¤ =64p(cm¤ ) ⑤

0371 ⑴ 부채꼴의 호의 길이는 밑면인 원의 둘레의 길이와 0376 단면인 원의 반지름의 길이를 r cm라고 하면
같으므로 부채꼴의 중심각의 크기를 x˘라고 하면 p_r¤ =400p, r¤ =400
x ∴ r=20(cm)(∵ r>0)
2p_6_ =2p_2
360
따라서 △OAH에서
∴ x˘=120˘
OH”="√25¤ -20¤ ='∂225=15(cm)
⑵ 주어진 전개도로 만들어지는 원뿔은 오른

쪽 그림과 같으므로
6 cm
(높이)="√6¤ -2¤ ='ß32=4'2(cm) 0377 OB”를 그으면 OB”=OC”=7 cm이 A B
1 므로 △AOB에서 5 cm 7 cm
(부피)= _p_2¤ _4'2
3 2 cm O
AB”="√7¤ -5¤ ='ß24=2'6(cm)
7 cm
16'2 따라서 단면인 원의 넓이는
= p(cm‹ )
3 C
p_(2'6)¤ =24p(cm¤ )
16'2
⑴ 120˘ ⑵ 높이:4'2 cm, 부피: p cm‹ ④
3

0378 구하는 최단 거리는 D C G H


0372 밑면의 반지름의 길이를 r cm라고 하면 오른쪽 그림에서 A’H”의 길
5 cm
2pr=24p ∴ r=12(cm) 이와 같으므로
주어진 전개도로 만들어지는 원뿔은 A’H” A 3 cm B 6 cm F 3 cm E
오른쪽 그림과 같으므로 20 cm
="√(3+6+3)¤ +5¤
(높이)="√20¤ -12¤ ='∂169=13(cm) 13 cm
='∂256 12 cm
=16(cm) 16 cm 0379 구하는 최단 거리는 오른 D C G
쪽 그림에서 AG”의 길이와 같으
5
므로
120
0373 ⑴ 2p_9_
360
=2pr AG”="√(8+4)¤ +5¤ A 8 B 4 F

∴ r=3 ='∂169=13 13

⑵ (높이)="√9¤ -3¤ ='ß72=6'2(cm) 0380 구하는 최단 거리는 오른쪽 그림에 A 4 B 3 C


1 서 AF”의 길이와 같으므로
⑵ (부피)= _p_3¤ _6'2 9 cm
3
AF”="√(4+3)¤ +7¤ 7
=18'2p(cm‹ )
='ß98=7'2
D E F
3 cm
⑴ 3 ⑵ 높이:6'2 cm, 부피:18'2p cm‹ 7'2

03. 피타고라스 정리의 활용 37


(001~042)개념RPM3-2(정답) 2014.11.24 02:54 PM 페이지38 다민 2540DPI 175LPI

RPM 알피엠

0381 구하는 최단 거리 B A D C B' 0386 원뿔의 전개도에서 부채 O


는 오른쪽 그림에서 B’F'” 꼴의 중심각의 크기를 x˘라고 하면 20 cm x˘ 20 cm
7
의 길이와 같으므로 x
2p_20_ =2p_5 A A'
F 4 E 8 H 4 G 8 F' 360
B’F'”="√(4+8+4+8)¤ +7¤
='∂625=25 25 ∴ x˘=90˘
5 cm
따라서 구하는 최단 거리는 A’A'”의
0382 구하는 가장 짧은 실의 길이 B B' 길이와 같으므로 직각삼각형 OAA'에서
는 오른쪽 그림에서 A’B'”의 길이와 A’A'”="√20¤ +20¤ ='∂800=20'2(cm) 20'2 cm
12p cm
같다.
A’A'”=2p_8=16p(cm)이므로
A 16p cm A' 0387 원뿔의 전개도에서 부채 O
A’B'”="√(16p)¤ +(12p)¤ 2 cm
꼴의 중심각의 크기를 x˘라고 하면 4 cm x˘
M
A’B'”="√400p¤
x
=20p(cm) ③ 2p_4_ =2p_1 A A'
360
∴ x˘=90˘
0383 원기둥의 옆면의 전개도를 그리 A A'
따라서 구하는 최단 거리는 A’M”의
1 cm
면 오른쪽 그림과 같다. 10p`cm
길이와 같으므로 직각삼각형 OAM에서
B’B'”=2p_4=8p(cm)이므로
B 8p`cm B'
A’M=
” "√4¤ +2¤ ='ß20=2'5(cm) 2'5 cm
AB”="√(10p)¤ -(8p)¤
="√36p¤
=6p(cm) 6p cm 0388 △DBC에서 DB” : BC”=2 : '3이므로
4'3 : BC”=2 : '3 ∴ BC”=6(cm)
0384 밑면의 둘레의 길이는 2p_3=6p(cm) yy` △ABC에서 AB” : BC”=1 : '2이므로
이므로 원기둥의 옆면을 두 바퀴 돌 B B' B'' AB” : 6=1 : '2 ∴ AB”=3'2(cm) ③
았을 때의 전개도는 오른쪽 그림과
같다. 16p`cm
6p`cm 0389 정삼각형의 한 변의 길이를 a cm라고 하면
따라서 구하는 최단 거리는 A’B"”
A 6p`cm A' A'' '3
의 길이이므로 yy` a¤ =9'3, a¤ =36 ∴ a=6(cm)(∵ a>0)
4
A’B"”="√(12p)¤ +(16p)¤ '3
∴ A’H=
” _6=3'3(cm)
="√400p¤ 2
=20p(cm) yy` 이때 점 G는 무게중심이므로
20p cm 2 2
AG”= A’H=
” _3'3=2'3(cm) 2'3 cm
3 3
단계 채점요소 배점
밑면의 둘레의 길이 구하기 30%
최단 거리가 A’B"”의 길이임을 알기 40%
0390 정사각형의 한 변의 길이를 a cm라고 하면

최단 거리 구하기 30% (3a)¤ +a¤ =(5'2 )¤ , 10a¤ =50


∴ a¤ =5

0385 오른쪽 그림에서 AC” 와 A


∴ AC”="√(2a)¤ +a¤ ="ç5a¤ ='ƒ5_5=5(cm) ①
6 cm
BD”의 교점을 H라고 하면 60˘ 60˘
30˘ 30˘
B D
ABCD는 마름모이므로 H 0391 △ABH에서 A’H” : BH”=1 : '3이므로
AC”⊥BD” 8 : BH”=1 : '3 ∴ BH”=8'3
C
△ABH에서 AB” : BH”=2 : '3 또한 △AHC는 직각이등변삼각형이므로 HC”=A’H”=8
이므로 6 : BH”=2 : '3 ∴ BC”=BH”+HC”=8'3+8=8('3+1)
BH”=3'3(cm) 1
∴ △ABC= _8('3+1)_8=32('3+1)
따라서 구하는 최단 거리는 BD”의 길이와 같으므로 2
BD”=2 BH”=2_3'3=6'3(cm) ② 32('3+1)

38 정답과 풀이
(001~042)개념RPM3-2(정답) 2014.11.24 02:54 PM 페이지39 다민 2540DPI 175LPI

● 본문 054쪽 ~ 058쪽 ●

0392 AD”=
'3
_8=4'3(cm) 0399 AD”=x라고 하면 AC”=2x
2
AB”=BC”="√3¤ +4¤ +x¤ ="√25+x¤
'3
∴ △ADE= _(4'3)¤ =12'3(cm¤ ) ④ 즉, △ABC는 AB”=BC”이고 ∠ABC=90˘인 직각이등변삼
4
각형이므로 AB” : AC”=1 : '2에서 "√25+x¤ : 2x=1 : '2

0393 AB”="√{3-(a-1)}¤ √+(a-3)¤ ='5이므로 2x="√50+2x¤

(4-a)¤ +(a-3)¤ =5, 2a¤ -14a+20=0 양변을 제곱하면 4x¤ =50+2x¤

a¤ -7a+10=0, (a-2)(a-5)=0 x¤ =25 ∴ x=5 (∵ x>0) 5

∴ a=2 또는 a=5 0400 꼭짓점 A에서 BC”에 내린 수선의 A


따라서 모든 a의 값의 합은 2+5=7 ④ 발을 H라고 하면
13 13
1 1
BH”=CH”= BC”= _10=5
0394 AB”="√(-5-0)¤ +√(5-a)¤ ="√a¤ -10a+50 2 2
△ABH에서 B C
AC”="√(1-0)¤ +√(3-a)¤ ="√a¤ -6a+10 5 H
이때 AB”=AC”이므로 AH”="√13¤ -5¤ ='∂144=12
a¤ -10a+50=a¤ -6a+10 1
∴ △ABC= _10_12=60 60
2
4a=40 ∴ a=10 ③
0401 직육면체의 가로와 세로의 길이, 높이를 각각 a, b, c
0395 정육면체의 대각선의 길이는 '3_12=12'3(cm) 라고 하면
이때 구의 지름의 길이는 정육면체의 대각선의 길이와 같으므로 BD” ¤ =a¤ +b¤ =25 yy ㉠
1 BG” ¤ =a¤ +c¤ =52 yy ㉡
(구의 반지름의 길이)= _12'3=6'3(cm) ④
2 DG” ¤ =b¤ +c¤ =45 yy ㉢
㉠+㉡+㉢을 하면
0396 BD”='2_8=8'2(cm)이므로
2(a¤ +b¤ +c¤ )=122 ∴ a¤ +b¤ +c¤ =61
1 1 따라서 직육면체의 대각선의 길이는
BH”= BD”= _8'2=4'2(cm)
2 2
"√a¤ +b¤ +c¤ ='ß61 ③
△ABH에서 A’H=
” "√9¤ -(4'2)¤ ='ß49=7(cm) 7 cm
0402 원기둥의 옆면의 전개도를 그리 B B'
1 면 오른쪽 그림과 같다.
0397 y=
2
x¤ 의 그래프와 y=2x+6의 그래프가 두 점 A, 8"2p cm
A’A'”=2p_4=8p(cm)이므로
B에서 만나므로 교점의 x좌표는
AB”="√(8'2p)¤ -(8p)¤
1 A A'
x¤ =2x+6, x¤ -4x-12=0 AB="√64p¤ =8p(cm) 8p cm
2
(x+2)(x-6)=0 ∴ x=-2 또는 x=6 8p cm

x=-2일 때 y=2이므로 A(-2, 2) 0403 점 A를 직선 l에 대하여 대 A


B
x=6일 때 y=18이므로 B(6, 18) 칭이동한 점을 A'이라고 하면 구하 6
4
∴ AB”="√{6-(-2)}¤ +(18-√2)¤ ='∂320=8'5 8'5 는 최단 거리는 A’'B”의 길이와 같다. l
P 10
∴ A’'B”="√12¤ +10¤
0398 AB”="√(2-1)¤ +(-2-2)¤ ='1å7 ='∂244 A'
12
BC”="√(-3-2)¤ +{1-(-√2)}¤ ='3å4 =2'ß61
AC”="√(-3-1)¤ +(1-2)¤ ='1å7 ③
③, ④ BC” ¤ =AB” ¤ +AC” ¤ 이고 AB”=AC”이므로
0404 △EAD에서 AD”=6 cm이므로 E
△ABC는 ∠A=90˘인 직각이등변삼각형이다.
DE”:AD”='3:2, DE”:6='3:2
1 A 30˘ D
⑤ △ABC= _AB”_AC” ∴ DE”=3'3(cm) F
2 6 cm
점 E에서 AD”, BC”에 내린 수선의 발을
1 17
= _'1å7_'1å7=
2 2 각각 F, G라고 하면
B G C
따라서 옳지 않은 것은 ③이다. ③ △DEF에서 DF”:DE”='3:2

03. 피타고라스 정리의 활용 39


(001~042)개념RPM3-2(정답) 2014.11.24 02:54 PM 페이지40 다민 2540DPI 175LPI

RPM 알피엠

DF”:3'3='3:2 ∴ DF”=
9
(cm) 0408 정팔면체의 한 모서리의 길이를 a cm라고 하면
2
'3
a¤ =8'3, a¤ =32
1 1 9 4
∴ △ECD= _CD”_CG”= _6_
2 2 2
∴ a=4'2(cm)(∵ a>0)
27 27 따라서 이 정팔면체의 한 모서리의 길이는 4'2 cm이다.
= (cm¤ ) cm¤
2 2
한편 정팔면체를 정사각뿔 2개로 나 A
누면 BCDE는 정사각형이므로 4'2 cm B
CE”='2_4'2=8(cm) E
0405
'6 C H
A’H=
” _12=4'6(cm) 이때
3 4'2 cm
4'2 cm D
1 1
D’M”=
'3
_12=6'3(cm) CH”= CE”= _8=4(cm)
2 2 2

이때 점 H는 △DBC의 무게중심이므로 이므로 △ACH에서

2 2 AH”=øπ(4'2)¤ -4¤ ='1å6=4(cm)


D’H=
” D’M”= _6'3=4'3(cm)
3 3 1 128
∴ (정사각뿔의 부피)= _(4'2)¤ _4= (cm‹ )
1 3 3
∴ △AHD= _4'3_4'6
2 따라서 구하는 정팔면체의 부피는
=24'2(cm¤ ) 24'2 cm¤ 128 256 256
2_ = (cm‹ ) cm‹
3 3 3

0409 구하는 최단 거리는 다음 그림처럼 강의 폭의 길이를


0406 AE”="√6¤ +3¤ ='ß45=3'5(cm)
제외한 그림에서의 최단 거리에 강의 폭을 더한다.
AF”="√6¤ +6¤ ='ß72=6'2(cm)
학교 (11-2)km
EF”="√6¤ -3¤ ='ß27=3'3(cm)
학교
따라서 AF” ¤ =AE” ¤ +EF” ¤ 이므로 △AEF는 ∠AEF=90˘인
11 km ⇨ x km
직각삼각형이다. 다리 2 km
1 1
∴ △AEF= _AE”_EF”= _3'5_3'3
2 2 12 km 도서관 12 km 도서관
9'ß15 9'ß15
= (cm¤ ) cm¤ ∴ (최단 거리)=x+2="√9¤ +12¤ +2='∂225+2
2 2
=15+2=17(km) 17 km

0410 원뿔의 전개도에서 부채꼴의 A


0407 ① △AOC에서 AC” : OC”=2 : 1이므로
중심각의 크기를 x˘라고 하면 20 cm x˘
10 cm
M
6 : OC”=2 : 1 ∴ OC”=3(cm)
x
∴ (밑면의 넓이)=p_3¤ =9p(cm¤ ) 2p_20_ =2p_5 B B'
360
② BC”=2 OC”=2_3=6(cm)이므로 △ABC는 정삼각형 ∴ x˘=90˘
이다. 5 cm
구하는 최단 거리는 B’M”의 길이와
③ △AOC에서 AC” : AO”=2 : '3이므로 같으므로 직각삼각형 ABM에서
6 : AO”=2 : '3 ∴ AO”=3'3(cm) B’M”="√20¤ +10¤ ='∂500=10'5(cm) 10'5 cm
따라서 원뿔의 높이는 3'3 cm이다.

④ (부피)=
1
_p_3¤ _3'3=9'3p(cm‹ ) 0411 AP”, BP”, CP”를 각각 그으면 A
3
△ABC
⑤ 원뿔의 전개도에서 부채꼴의 중심 6 cm D F
=△PAB+△PBC+△PCA P
각의 크기를 x˘라고 하면 x˘
1 1
= _12_PD”+ _12_PE”
x 2 2 B
E
C
2p_6_ =2p_3
360 O 1 12 cm
3 cm =+ _12_PF”
∴ x˘=180˘ 2
따라서 옳은 것은 ②이다. ② =6(PD”+PE”+PF”)

40 정답과 풀이
(001~042)개념RPM3-2(정답) 2014.11.24 02:54 PM 페이지41 다민 2540DPI 175LPI

● 본문 058쪽 ~ 059쪽 ●

'3 따라서 구하는 원뿔의 부피는


그런데 △ABC= _12¤ =36'3(cm¤ )이므로
4
1 128'2
_p_4¤ _8'2= p(cm‹ ) yy`
6(PD”+PE”+PF”)=36'3 3 3
∴ PD”+PE”+PF”=6'3(cm) 6'3 cm 128'2
p cm‹
3
0412 AB” : AC”=BD” : CD”이므로
9 : 6=3 : CD” ∴ CD”=2 yy` 단계 채점요소 배점
밑면의 반지름의 길이 구하기 40%
△ABH에서 A’H” ¤ =9¤ -(3+2+CH” )¤ yy ㉠
원뿔의 높이 구하기 20%
△ACH에서 A’H” ¤ =6¤ -CH” ¤ yy ㉡
원뿔의 부피 구하기 40%
㉠, ㉡에서 9¤ -(3+2+CH”)¤ =6¤ -CH” ¤
10 CH”=20 ∴ CH”=2
∴ AH”="√6¤ -2¤ ='ß32=4'2 yy`

1 0415 △AOC에서 OA”:AC”=1:2이므로


∴ △ABC= _(3+2)_4'2=10'2 yy`
2 OA” : 4=1 : 2 ∴ OA”=2(cm)
10'2 또한 OC” : AC”='3 : 2이므로
OC” : 4='3 : 2 ∴ OC”=2'3(cm)
단계 채점요소 배점
한편 △AOB에서 OB”=OA”=2 cm yy`
CD”의 길이 구하기 30%
△AOC를 직선 l을 축으로 하여 A 2 cm
A’H의
” 길이 구하기 50%
4 cm
△ABC의 넓이 구하기 20% 1회전시켜 얻은 입체도형의 부피는
1 C B
_p_(2'3 )¤ _2=8p(cm‹ ) O
0413 삼각뿔 G-BCD의 부피는 3 2 cm

1 1 1 yy`
_△BCD_CG”= _{ _3'2_3'2}_3'2
3 3 2 △AOB를 직선 l을 축으로 하여 1회전시켜 얻은 입체도형의
=9'2(cm‹ ) yy` 부피는
한편, △BGD는 BG”=GD”=DB”='2_3'2=6(cm)인 정 1 8
_p_2¤ _2= p(cm‹ ) yy`
삼각형이므로 3 3
'3 따라서 구하는 입체도형의 부피는
△BGD= _6¤ =9'3(cm¤ ) yy`
4 8 16
8p- p= p(cm‹ ) yy`
삼각뿔 C-BGD의 부피는 3 3
1 1 16
_△BGD_CI”= _9'3_CI”=3'3 CI” yy` p cm‹
3 3 3
따라서 3'3 CI”=9'2이므로 CI”='6(cm) yy`
단계 채점요소 배점
'6 cm
OA”, OC”, OB”의 길이 구하기 20%
△AOC를 회전시켜 얻은 입체도형의 부피 구하기 30%
단계 채점요소 배점
△AOB를 회전시켜 얻은 입체도형의 부피 구하기 30%
삼각뿔 G-BCD의 부피 구하기 30%
△ABC를 회전시켜 얻은 입체도형의 부피 구하기 20%
△BGD의 넓이 구하기 20%
삼각뿔 C-BGD의 부피를 CI”를 사용하여 나타내기 30%
CI”의 길이 구하기 20%

0416 밑면의 둘레의 길이는 B''' B'' B' B


0414 밑면인 원의 반지름의 길이를 r cm라고 하면
2p_3=6p(cm)이므로 원기둥
120
2p_12_ =2pr ∴ r=4(cm) 의 옆면을 3바퀴 돌았을 때 전개 9p`cm
360
yy` 도는 오른쪽 그림과 같다.
12 cm A
A''' A'' A'
주어진 전개도로 만들어진 원뿔은 오른쪽 이때 최단 거리는 A’'''B”의 길이
6p`cm 6p`cm 6p`cm
그림과 같으므로 와 같으므로
(높이)="√12¤ -4¤ ='∂128=8'2(cm) 4 cm A’'''B”="√(6p+6p+√6p)¤ +(9p)¤
yy` A'''B="√405p¤ =9'5p(cm) 9'5p cm

03. 피타고라스 정리의 활용 41


(001~042)개념RPM3-2(정답) 2014.11.24 02:54 PM 페이지42 다민 2540DPI 175LPI

RPM 알피엠
Ⅲ 삼각비

0417 C’M”=CN”=
'3
2
_12=6'3(cm)
04 삼각비
△ABD에서 삼각형의 두 변의 중점을 연결한 선분의 성질에
의해 BC” 3 1 1
0420 sin A= = =
6 2 2
1 1 AB”
MÚN”= BD”= _12=6(cm)
2 2
오른쪽 그림과 같이 꼭짓점 C에서 MÚN” C AC” 3'3
0421
'3 '3
cos A= = =
에 내린 수선의 발을 H라고 하면 AB” 6 2 2
1 6'3 cm 6'3 cm
MÚH=
” N’H=
” MÚN”=3(cm)
2
BC” 3
0422
'3 '3
tan A= = =
△CMH에서 M
H
N AC” 3'3 3 3
3 cm 6 cm
CH”="√(6'3 )¤ -3¤ ='ß99
=3'ß11(cm) AC” 3'3
0423
'3 '3
sin B= = =
1 AB” 6 2 2
∴ △CMN= _6_3'ß11=9'ß11(cm¤ ) 9'ß11 cm¤
2

BC” 3 1 1
0418 구하는 최단 거리는 오른 12 cm 0424 cos B= = =
6 2 2
A AB”
C C'
쪽 그림에서 CP”의 길이와 같다. 60˘
4 cm
∠CDA=30˘이므로 P 3'3
AC”
∠CDC'=30˘+60˘=90˘ B
30˘
D
2 cm 0425 tan B= =
3
='3 '3
BC”
△CDC'에서
CD”="√12¤ -6¤ ='∂108=6'3(cm) 0426 AC”="√4¤ -3¤ ='7 '7
따라서 △CDP에서
CP”="√(6'3 )¤ +2¤ ='∂112=4'7(cm) 4'7 cm AC”
0427
'7
sin B= =
BC” 4

0419 ∠OBP=30˘이므로 A cos B=


AB”
=
3
BC” 4
△OBP에서 OP” : BP”=1 : '3
2 : BP”=1 : '3 ∴ BP”=2'3 AC” '7 '7 3 '7
30˘ tan B= = , ,
O AB” 3 4 4 3
∴ CQ”=BP”=2'3
B C
또한 PQ”=2 OP”=4이므로 P2 Q
AC” x 2
BC”=2'3+4+2'3=4+4'3 0428 sin B= 이므로 = 에서
9 3
AB”
BC” ¤ =
'3 '3
∴ △ABC= _(4+4'3)¤ 3x=18 ∴ x=6 6
4 4
=16'3+24 16'3+24
AB” x
0429
'3
cos A= 이므로 = 에서
AC” 12 2
2x=12'3 ∴ x=6'3 6'3

BC” 10 '5
0430 tan A= 이므로
x
= 에서
5
AC”
'5x=50 ∴ x=10'5 10'5

1 1
0431 sin 30˘+cos 60˘=
2
+ =1
2
1

2+'2 2+'2
0432
'2
tan 45˘+sin 45˘=1+ =
2 2 2

42 정답과 풀이
(043~065)개념RPM3-2(정답) 2014.11.24 02:56 PM 페이지43 다민 2540DPI 175LPI

● 본문 059쪽 ~ 065쪽 ●

OB” OB”
0433 0448
'3 '3 '3
tan 60˘-sin 60˘='3- = cos x= = =OB” OB”
2 2 2 O’A” 1

'2 ¤ '2 ¤
0434 sin¤ 45˘+cos¤ 45˘={
2
} +{
2
}
0449 tan x=
CD”
=
CD”
=CD” CD”
OD” 1
1 1
= + =1 1
2 2
0450 0.79
1 1
0435
'3 '3
tan 30˘_cos 30˘= _ =
3 2 2 2 0451 0.62

0436 0452
'3
cos 30˘÷tan 60˘= ÷'3 1.28
2
'3 1 1 1
= _ =
2 '3 2 2 0453 tan 0˘+sin 0˘=0+0=0 0

0437 30˘ 0454 sin 0˘+cos 0˘=0+1=1 1

0438 30˘
0455 tan 0˘+cos 90˘+sin 90˘=0+0+1=1 1

0439 60˘
1 1 1
0456 sin 90˘_sin 30˘=1_
2
=
2 2
0440 60˘

0457 cos 90˘+sin 0˘_sin 90˘=0+0_1=0 0


0441 45˘

0442 45˘ 0458 <, <

x 0459 >, >


0443
'3
tan 30˘= = ∴ x=2'3
6 3

cos 30˘=
6
=
'3
∴ y=4'3 x=2'3, y=4'3 0460 <, <
y 2

4
0461 >
0444
'2
cos 45˘= = ∴ x=4'2
x 2
y 0462 <
tan 45˘= =1 ∴ y=4 x=4'2, y=4
4

0463 <
x 1
0445 cos 60˘= =
4 2
∴ x=2

y '3
0464 <
sin 60˘= = ∴ y=2'3 x=2, y=2'3
4 2
0465 =
x 1
0446 sin 30˘=
2'3
=
2
∴ x='3
0466 >
y '3
cos 30˘= = ∴ y=3 x='3, y=3
2'3 2
0467 0.7431

AB” AB”
0447 sin x= =
1
=AB” AB”
0468 0.6293
O’A”

04. 삼각비 43
(043~065)개념RPM3-2(정답) 2014.11.24 02:56 PM 페이지44 다민 2540DPI 175LPI

RPM 알피엠

0469 1.0355 BC” 3


0480 sin A=
20
= 이므로 5BC”=60
5
∴ BC”=12
0470 0.7547
∴ AC”="√20¤ -12¤ ='∂256=16 ④

0471 0.6820
6
0481 tan B=
AB”
=3이므로

0472 1.1918 3AB”=6 ∴ AB”=2


BC”="√6¤ +2¤ ='ß40=2'ß10이므로
0473 50˘
sin C=
AB”
=
2
=
'ß10 'ß10
BC” 2'ß 1 0 10 10

0474 49˘
AB”
0482
'5
cos A= = 이므로 3AB”=12'5
12 3
0475 51˘ ∴ AB”=4'5 yy`

BC”="√12¤ -(4'5)¤ ='ß64=8 yy`


0476 BC”="√2¤ +4¤ ='ß20=2'5이므로 1
∴ △ABC= _4'5_8=16'5 yy`
4 2'5 2
① sin B= =
2'5 5 16'5
2 '5
② cos B= =
2'5 5 단계 채점요소 배점

2 '5 AB”의 길이 구하기 40%


③ sin C= =
2'5 5 BC”의 길이 구하기 40%

4 2'5 △ABC의 넓이 구하기 20%


④ cos C= =
2'5 5
2 1
⑤ tan C=
4
=
2
0483 AB”=x, AC”=y라고 하면
y '5 '5
따라서 옳은 것은 ③이다. ③ sin B= = ∴ y= x
x 3 3
피타고라스 정리에 의해 x¤ =6¤ +y¤ 이므로
0477 AC”="√10¤ -8¤ ='ß36=6이므로 5 4
x¤ =36+ x¤ , x¤ =36, x¤ =81
6 3 8 4 9 9
sin B= = , cos B= =
10 5 10 5 ∴ x=9(∵ x>0) 9
3 4 7 7
∴ sin B+cos B= + =
5 5 5 5
5
0484 sin A=
6
이므로 이를 만족하는 직각 C

0478 △ADC에서 AC”="√('ß34)¤ -3¤ ='ß25=5 삼각형 ABC를 그리면 오른쪽 그림과 같다. 6
5
△ABC에서 BC”="√13¤ -5¤ ='∂144=12 AB”="√6¤ -5¤ ='ß11이므로
BC” 12 12 'ß11 'ß11 11 A B
∴ sin A= = cos A_tan C= _ =
AB” 13 13 6 5 30

0479 △ABC에서 BC”="√15¤ -12¤ ='8å1=9


3
AC”
∴ tan x= =
12
=
4 0485 2 tan A-3=0에서 tan A=
2
이므 C
BC” 9 3
로 이를 만족하는 직각삼각형 ABC를 그리
또한 △ADC에서 DC”="√13¤ -12¤ ='2å5=5 3
면 오른쪽 그림과 같다.
AC” 12
∴ tan y= = AC”="√2¤ +3¤ ='ß13이므로
DC” 5 A B
2
4 12 16 16 3 3'ß13
∴ tan x_tan y= _ = sin A= =
3 5 5 5 'ß13 13

44 정답과 풀이
(043~065)개념RPM3-2(정답) 2014.11.24 02:56 PM 페이지45 다민 2540DPI 175LPI

● 본문 065쪽 ~ 068쪽 ●

cos A=
2
=
2'ß13 0490 BD”="√6¤ +8¤ ='∂100=10이고 A
x
D
'ß13 13
△ABDª△HAD이므로
6
121`+`121
3'ß13 2'ß13 ∠ABD=∠x x H 10
= 11111124 =5
sin A+cos A 13 13
∴ ② AD” 8 4 B C
sin A-cos A
121`-`121
3'ß13 2'ß13 ∴ sin x= = = 8
13 13 BD” 10 5
AB” 6 3
cos x= = =
BD” 10 5
2
0486 3 cos A-2=0에서 cos A=
3
yy`
4 3 1 1
∴ sin x-cos x= - =
5 5 5 5
이므로 이를 만족하는 직각삼각형 ABC를 C
그리면 오른쪽 그림과 같다.
BC”="√3¤ -2¤ ='5이므로 yy`
3
0491 △ABC에서 AB” ¤ =BD”_BC”이므로
AB” ¤ =8_10=80
'5 '5
sin A= , tan A= yy` A B
∴ AB”=4'5 (∵ AB”>0)
3 2 2
'5 '5 이때 △ABCª△DAC(AA 닮음)이므로
∴ 30 sin A_tan A=30_ _ =25 yy`
3 2 ∠B=∠DAC
25 AB” 4'5 2'5 2'5
∴ cos x=cos B= = =
BC” 10 5 5
단계 채점요소 배점
cos A의 값 구하기 20%
BC”의 길이 구하기 30% 0492 △ABC에서 BC”="√8¤ +15¤ ='∂289=17이고
sin A, tan A의 값 구하기 30% △ABCª△EDC(AA 닮음)이므로 ∠x=∠B
30 sin A_tan A의 값 구하기 20% AB” 8 8
∴ cos x=cos B= =
BC” 17 17

0487 ∠C=90˘이고 A
0493 △ABC에서 AB”="√9¤ -6¤ ='4å5=3'5이고
15
sin(90˘-A)=sin B= △ABCª△EDC(AA 닮음)이므로
17 17 15
이므로 이를 만족하는 직각삼각형 ABC를 그 ∠x=∠B
리면 오른쪽 그림과 같다. AB” 3'5 '5 '5
B C ∴ cos x=cos B= = =
BC” 9 3 3
BC”="√17¤ -15¤ ='6å4=8이므로
8 8
tan A=
15 15 0494 △ADE에서 DE”=øπ('7)¤ -2¤ ='3이고
△ABCª△AED(AA 닮음)이므로
0488 BC”="√9¤ +12¤ ='∂225=15이고 ∠B=∠AED
△ABCª△DBAª△DAC (AA 닮음) AD” 2 2'7
∴ sin B= = = ,
AE” '7 7
AB” 9 3
∠x=∠C이므로 sin x= = = DE” '2å1
BC” 15 5 cos B= =
'3
=
AE” '7 7
AB” 9 3
∠y=∠B이므로 cos y= = = 2'7 '2å1 2'3 2'3
BC” 15 5 ∴ sin B_cos B= _ =
7 7 7 7
3 3 6 6
∴ sin x+cos y= + =
5 5 5 5
0495 △EDC에서 CE”="√9¤ -7¤ ='3å2=4'2이고
△ABCª△DEC(AA 닮음)이므로
0489 △ABCª△ACDª△CBD(AA 닮음)이므로
∠A=∠EDC
∠A=∠BCD, ∠B=∠ACD
CE” 4'2 CE” 4'2
AD” ∴ sin x= = , tan x= =
④ △ADC에서 sin B= DC” 9 DE” 7
AC” sin x 4'2 7 7 7
∴ = _ =
따라서 옳지 않은 것은 ④이다. ④ tan x 9 4'2 9 9

04. 삼각비 45
(043~065)개념RPM3-2(정답) 2014.11.24 02:56 PM 페이지46 다민 2540DPI 175LPI

RPM 알피엠

0496 그래프가 x축, y축과 만나는 점을 각각 A, B라고 하


1
1
= 122 =
면 x-2y+8=0에 O’A” 2 '5
∴ cos a=
5
21
AB” '5
y=0을 대입하면 x=-8이므로 A(-8, 0) 2
x=0을 대입하면 y=4이므로 B(0, 4) 2'5 ¤ '5 ¤
∴ sin¤ a-cos¤ a={ } -{ }
직각삼각형 AOB에서 O’A”=8, OB”=4이므로 5 5
AB”="√8¤ +4¤ ='ß80=4'5 4 1 3 3
= - =
5 5 5 5
OB” OB”
∴ sin a_tan a= _
AB” O’A”
4 4 '5 '5
0500 △EGC는 ∠EGC=90˘인 직각삼각형이고
= _ = EG”="√3¤ +3¤ =3'2, CE”="√3¤ +3¤ +3¤ =3'3
4'5 8 10 10
EG” 3'2 '6
∴ cos x= = = ①
CE” 3'3 3
0497 그래프가 x축, y축과 만나는 점을 각각 A, B라고 하
면 4x+3y-6=0에
0501 △BFH는 ∠BFH=90˘인 직각삼각형이고
3 3
y=0을 대입하면 x= 이므로 A { , 0} FH”="√6¤ +6¤ =6'2, BH”="√6¤ +6¤ +6¤ =6'3
2 2
x=0을 대입하면 y=2이므로 B(0, 2) BF” 6 '3
sin x= = =
BH” 6'3 3
3
직각삼각형 BOA에서 O’A”= , OB”=2이므로
2 FH” 6'2 '6
cos x= = =
BH” 6'3 3
= 12 =
OB” 2 4 4
tan a=
1
O’A” 3 3 3 BF” 6 '2
2 tan x= = =
FH” 6'2 2
'3 '6 '2
∴ sin x_cos x-tan x= _ -
0498 그래프가 x축, y축과 만나는 점 y 3 3 2
B
5 5 =
'2
-
'2
을 각각 A, B라고 하면 y= x+5에 3 2
2 5
y=-x+5
2
y=0을 대입하면 x=-2이므로 =-
'2
-
'2
6 6
A(-2, 0) A a
-2 O x
x=0을 대입하면 y=5이므로 B(0, 5)
0502 △DFH는 ∠FHD=90˘인 직각삼각형이고
직각삼각형 AOB에서 O’A”=2, OB”=5이므로
FH”="√8¤ +6¤ ='∂100=10
AB”="√2¤ +5¤ ='ß29
DF”="√8¤ +6¤ +4¤ ='∂116=2'ß29
5 2 7
∴ sin a+cos a= + = D’H” 4 2'ß29
'ß29 'ß29 'ß29 sin x= = =
FD” 2'ß29 29
7'ß29 7'ß29
∴ sin a+cos a= FH” 10 5'ß29
29 29 cos x= = =
DF” 2'ß29 29
2'ß29 5'ß29 10 10
0499 그래프가 x축, y축과 만나는 점을 각 y ∴ sin x_cos x=
29
_
29
=
29 29
각 A, B라고 하면 y=2x-1에 A a
O a x
y=0을 대입하면 x=
1 1
이므로 A { , 0} y=2x-1
0503 △AEG는 ∠AEG=90˘인 직각삼각형이고
2 2 B
EG”="√4¤ +3¤ ='ß25=5
x=0을 대입하면 y=-1이므로 B(0, -1)
AG”="√4¤ +3¤ +5¤ ='ß50=5'2 yy`
1
직각삼각형 OBA에서 O’A”= , OB”=1이므로 AE” 5
2 sin x= = =
'2
AG” 5'2 2
1 ¤ '5
AB”=æ{≠ } +1¤ =Æ;4%;=
2 2 EG” 5 '2
cos x= = =
AG” 5'2 2
= 122 =
OB” 1 2'5
∴ sin a=
5 AE” 5
21
AB” '5 tan x= = =1 yy`
2 EG” 5

46 정답과 풀이
(043~065)개념RPM3-2(정답) 2014.11.24 02:56 PM 페이지47 다민 2540DPI 175LPI

● 본문 068쪽 ~ 070쪽 ●

= 111124 = 12 =4
2 tan x 2_1 2 1
∴ 0508 cos 60˘= 이므로 2x+30˘=60˘ ∴ x=15˘
1
sin x_cos x 1 2
21 _ 21
'2 ` ` '2 yy`
2 2 2
∴ tan 3x-sin 2x=tan 45˘-sin 30˘
4
1 1 1
=1- =
2 2 2
단계 채점요소 배점
EG”, AG”의 길이 구하기 40%
0509
'3
sin x, cos x, tan x의 값 구하기 30% ⑴ sin 60˘= 이므로
2
주어진 식의 값 구하기 30%
2x-20˘=60˘ ∴ x=40˘
'2
⑵ sin 45˘=cos 45˘= 이므로 x=45˘
2
1 1+'3 ∴ tan x=tan 45˘=1
0504
'3
① (좌변)= + =
2 2 2
'2
⑶ cos 45˘= 이므로
'3 '2 3 1 2
② (좌변)='3_ -'2_ = -1=
2 2 2 2
x+15˘=45˘ ∴ x=30˘
'3 1
12+1
③ (좌변)='3_ -2_ =1-1=0 '3
3 2
tan x+1 tan 30˘+1 3
∴ = =
tan x-1 tan 30˘-1
12-1
'3
④ (좌변)='3÷ -1=2-1=1 '3
2 3
'3 1 1 '3 1 '3-3
⑤ (좌변)= _ - = - = =
'3+3
=-2-'3
3 2 2 6 2 6 '3-3
따라서 옳지 않은 것은 ①, ④이다. ①, ④ ⑷ tan 60˘='3이므로
4x-20˘=60˘ ∴ x=20˘
∴ sin 3x-cos(x+10˘)=sin 60˘-cos 30˘
1 1
0505
'2 '2
(주어진 식)={1- - }`{1+ - } '3 '3
2 2 2 2 = - =0
2 2
1 '2 1 '2
⑴ 40˘ ⑵ 1 ⑶ -2-'3 ⑷ 0
={ - }`{ + }
2 2 2 2
1 1 1 1
=
4
- =-
2 4
-
4 0510 tan 45˘=1이므로 60˘-3x=45˘ ∴ x=5˘
∴ (주어진 식)=cos 45˘_sin 30˘
'2 1 '2 '2
= _ =
2 2 4 4
'2
(주어진 식)='3_'3+ 111111124
'2_`12`-2_1
2
0506 1 1
'3
'3_`12`+2_1
3 2
0511 (2x-1)¤ =0이므로 x=
2
(중근)

1-2 1 5 5 1
=3+ =3- = 따라서 cos A= 이므로 A=60˘ 60˘
1+1 2 2 2 2

AD”
0512
'2
△ABD에서 sin 45˘= =
6'2 2
0507 세 내각의 크기의 비가 3:4:5이고 삼각형의 내각의
크기의 합은 180˘이므로 2AD”=12 ∴ AD”=6

3 6 '3
∠A=180˘_ =45˘ △ADC에서 sin 60˘= =
3+4+5 AC” 2

'2 '3 AC”=12 ∴ AC”=4'3 4'3


∴ sin A=sin 45˘=
2
BC”
cos A=cos 45˘=
'2 0513 tan 45˘=
6
=1 ∴ BC”=6
2
tan A=tan 45˘=1 6 '3
sin 60˘= = 이므로
AC” 2
'2 '2 1 1
∴ sin A_cos A_tan A= _ _1= '3 AC”=12 ∴ AC”=4'3 ⑤
2 2 2 2

04. 삼각비 47
(043~065)개념RPM3-2(정답) 2014.11.24 02:56 PM 페이지48 다민 2540DPI 175LPI

RPM 알피엠

0514 ∠BAD=60˘-30˘=30˘이므로 △ABD는 0='3_(-2)+b ∴ b=2'3


AD”=BD”인 이등변삼각형이다. yy` ∴ ab='3_2'3=6 6
△ADC에서 AD”=x이므로

cos 60˘=
2'3
=
1
∴ x=4'3 yy` 0519 (기울기)=tan 60˘='3 y
x 2 4
yy`
y
tan 60˘= ='3 ∴ y=6 yy` 직선 y='3x+b가 점 ('3, 7)을 지나므
2'3 -
4"3
로 3 60˘
∴ xy=4'3_6=24'3 yy` O x
7='3_'3+b ∴ b=4
24'3
즉, 직선의 방정식은 y='3x+4이므로 yy`

단계 채점요소 배점 4'3
x절편은 - , y절편은 4이다.
3
△ABD가 이등변삼각형임을 알기 20%
x의 값 구하기 30% 따라서 구하는 삼각형의 넓이는
y의 값 구하기 30% 1 4'3 8'3
_ _4= yy`
xy의 값 구하기 20% 2 3 3
8'3
3
0515 △ABD에서 22.5˘ A
단계 채점요소 배점
AD”=BD”=6이므로 22.5˘
기울기 구하기 20%
∠DAB=∠B=22.5˘이고 45˘
B C 직선의 방정식 구하기 40%
6 D
∠ADC=22.5˘+22.5˘=45˘ 삼각형의 넓이 구하기 40%
△ADC에서
AC” '2
sin 45˘= = 이므로
6 2 0520 ⑤ AB”∥CD”이므로 ∠z=∠y(동위각)
2AC”=6'2 ∴ AC”=3'2 OB” OB”
∴ sin z=sin y= = =OB”
O’A” 1
DC” '2
cos 45˘= = 이므로
6 2 따라서 옳지 않은 것은 ⑤이다. ⑤
2DC”=6'2 ∴ DC”=3'2
따라서 △ABC에서 0.66 1.15
0521 cos 49˘=
1
=0.66, tan 49˘=
1
=1.15
AC” 3'2
tan 22.5˘= = ='2-1 '2-1
BC” 6+3'2 ∴ cos 49˘+tan 49˘=0.66+1.15=1.81 1.81

0516 (기울기)=tan 45˘=1 0522 AB”∥CD”이므로 ∠OAB=∠y


직선 y=x+b가 점 (0, 2)를 지나므로 2=0+b 따라서 점 A의 좌표는
∴ y=x+2 ② (cos x, sin x), (cos x, cos y),
(sin y, sin x), (sin y, cos y)
의 4가지가 될 수 있다. ③
0517
'3
(기울기)=tan 30˘=
3

0523
'3
직선 y= x+b가 점 (-3, 0)을 지나므로 △ABC에서 ∠A=180˘-(90˘+36˘)=54˘이고
3
cos 54˘=0.59이므로
'3
0= _(-3)+b ∴ b='3
3 AC” AC”
cos 54˘= = =0.59
AB” 10
따라서 구하는 직선의 방정식은
'3 '3 ∴ AC”=5.9 5.9
y= x+'3 y= x+'3
3 3
0524 ㄱ. sin 0˘+tan 0˘+cos 90˘=0+0+0=0 (참)
0518 a=tan 60˘='3 ㄴ. tan 90˘의 값은 정할 수 없다. (거짓)
직선 y='3x+b가 점 (-2, 0)을 지나므로 ㄷ. sin 90˘+cos 0˘=1+1=2 (참)

48 정답과 풀이
(043~065)개념RPM3-2(정답) 2014.11.24 02:56 PM 페이지49 다민 2540DPI 175LPI

● 본문 070쪽 ~ 073쪽 ●

'3 '3 sin 15˘<cos 45˘<sin 80˘<cos 0˘<tan 46˘


ㄹ. sin 60˘= , cos 30˘= 이므로
2 2 sin 15˘, cos 45˘, sin 80˘, cos 0˘, tan 46˘
sin 60˘=cos 30˘ (참)
ㅁ. cos 90˘-sin 90˘=0-1=-1 (거짓)
0532 0˘<A<90˘일 때, 0<sin A<1이므로
따라서 옳은 것은 ㄱ, ㄷ, ㄹ이다. ㄱ, ㄷ, ㄹ sin A+1>0, sin A-1<0
∴ (주어진 식)=sin A+1-(sin A-1)
0525
'3
(주어진 식)=1_'3-1_0+ =2 ③
2
'3 3'3 3'3
='3+ =
2 2 2 0533 ⑴ 0˘<A<45˘일 때, 0<sin A<cos A이므로
sin A+cos A>0, sin A-cos A<0
1
0526
'3 '3
(주어진 식)= _1- _1+2_ _1 ∴ (주어진 식)=(sin A+cos A)+(sin A-cos A)
2 2 2
=2 sin A
1 '3 1+'3 1+'3
= - +'3= ⑵ 0˘<A<45˘일 때, 0<tan A<1이므로
2 2 2 2
tan A-1<0, tan A+1>0
0527 ①, ②, ④, ⑤ 1 ∴ (주어진 식)=-(tan A-1)+(tan A+1)=2
1 1 ⑴ 2 sin A ⑵ 2
③ (주어진 식)= =
1+1 2
따라서 계산한 결과가 다른 하나는 ③이다. ③
0534 45˘<x<90˘일 때, cos x<sin x이고

0528
'2
① 0˘…A<45˘일 때, sin A<cos A이므로 0<cos x< 이므로
2
④ sin 23˘<cos 23˘
sin x-cos x>0, 1-cos x>0
② 45˘<A…90˘일 때, cos A<sin A이므로
∴ (주어진 식)=(sin x-cos x)-(1-cos x)
sin 75˘>cos 75˘
=sin x-1 sin x-1
③ 0˘…A…90˘일 때, A의 값이 증가하면 cos A의 값은 감소
하므로 cos 48˘>cos 50˘
0535 45˘<A<90˘일 때, 0<cos A<sin A이므로
④ 0˘…A…90˘일 때, A의 값이 증가하면 tan A의 값은 한없
cos A-sin A<0, sin A+cos A>0
이 커지므로 tan 20˘<tan 40˘
∴ "√(cos A-sin A)¤ +"√(sin A+cos A)¤
⑤ tan 45˘=1에서 tan 50˘>1이고 0<cos 70˘<1이므로
=-(cos A-sin A)+(sin A+cos A)
tan 50˘>cos 70˘
=2 sin A
따라서 옳은 것은 ⑤이다. ⑤
'3
이때 2 sin A='3이므로 sin A=
2
0529 ① 0˘<x<45˘일 때, sin x<cos x ∴ ∠A=60˘
따라서 옳지 않은 것은 ①이다. ① ∴ tan A=tan 60˘='3 '3

0530 45˘<A<90˘일 때, cos A<sin A<1이고


7.547
tan 45˘=1이므로 1<tan A 0536 cos A=
10
=0.7547이므로

∴ cos A<sin A<tan A ② ∠A=41˘ 41˘

0531 0˘…x…90˘일 때, sin x의 값은 0에서 1로 증가하고, 0537 x=37, y=39이므로


cos x의 값은 1에서 0으로 감소하므로 x+y=37+39=76 76
sin 15˘<sin 45˘=cos 45˘<sin 80˘<1 yy ㉠
cos 0˘=1 yy ㉡
0538 ∠A=180˘-(90˘+50˘)=40˘이므로
또한 tan 45˘=1이고 tan x의 값은 0에서 한없이 증가하므로
AC”
cos 40˘= =0.7660
tan 45˘<tan 46˘ ∴ 1<tan 46˘ yy ㉢ 10
㉠, ㉡, ㉢에서 ∴ AC”=7.660 7.660

04. 삼각비 49
(043~065)개념RPM3-2(정답) 2014.11.24 02:56 PM 페이지50 다민 2540DPI 175LPI

RPM 알피엠

0539 ∠B=180˘-(90˘+55˘)=35˘이므로
=
'3 4
_ -
'3
2 3 2
AC”
sin 35˘= =0.5736
100 '3
=
6
∴ AC”=57.36 yy`
'3 '3 '2 '2
BC” ⑵ (주어진 식)={1- }{ +2_ _ }
cos 35˘= =0.8192 2 2 2 2
100
'3 '3
∴ BC”=81.92 yy` ={1- }{1+ }
2 2
∴ AC”+BC”=57.36+81.92=139.28 yy`
3
139.28 =1-
4

단계 채점요소 배점 1
=
4
AC”의 길이 구하기 40%
⑶ (주어진 식)= 1112 - 1112
BC”의 길이 구하기 40%
1 1
1+`21 1-`21
'2 '2
AC”+BC”의 길이 구하기 20%
2 2
2 2
(주어진 식)= -
4 2+'2 2-'2
0540
'3
sin A= = 이므로
AB” 3 2(2-'2)-2(2+'2)
(주어진 식)=
'3 AB”=12 ∴ AB”=4'3 (2+'2)(2-'2)
∴ AC”="√(4'3)¤ -4¤ ='ß32=4'2 -4'2
(주어진 식)=
2
AC” 4'2 '6
③ sin B= = =
AB” 4'3 3 =-2'2
따라서 옳지 않은 것은 ③이다. ③ '3 1
⑴ ⑵ ⑶ -2'2
6 4

0541 AB” : AC”=2 : 3이므로


1
0545
'3 '3-1
AB”=2k, AC”=3k(단, k>0)라고 하면 cos 30˘-cos 60˘= - = 이므로
2 2 2
BC”="√(3k)¤ -(2k)¤ ='5k '3-1
이차방정식 2x¤ -ax+1=0에 x= 을 대입하면
BC” '5k '5 '5 2
∴ tan A= = =
AB” 2k 2 2 '3-1 ¤ '3-1
2{ } -a_ +1=0
2 2

8.192 (1-'3)a=2('3-3)
0542 sin A=
10
=0.8192이고
2('3-3) 2('3-3)(1+'3)
∴ a= =
sin 55˘=0.8192이므로 ∠A=55˘ 1-'3 (1-'3)(1+'3)
AC” =2'3 ⑤
cos 55˘= =0.5736이므로
10
AC”=5.736 5.736 0546 ② cos x=0.75
따라서 옳지 않은 것은 ②이다. ②
12 3
0543 sin A=
AC”
=
5
∴ AC”=20
AD”
0547
'2
∴ AB”="√20¤ -12¤ ='∂256=16 △ADC에서 sin 45˘= =
8 2
16 12 2AD”=8'2 ∴ AD”=4'2
∴ cos A-tan A= -
20 16
4'2
1 1 △ABD에서 tan 60˘= ='3
= BD”
20 20
4'6
'3 BD”=4'2 ∴ BD”= ⑤
3
'3 ¤ 1
0544
'3
⑴ (주어진 식)= ÷{ } -'3_
2 2 2
'3 3 '3 0548 ⑴ tan 45˘=1이므로
= ÷ -
2 4 2 x+15˘=45˘ ∴ x=30˘

50 정답과 풀이
(043~065)개념RPM3-2(정답) 2014.11.24 02:56 PM 페이지51 다민 2540DPI 175LPI

● 본문 073쪽 ~ 076쪽 ●

∴ sin x+cos 2x=sin 30˘+cos 60˘


0553
'2
tan 45˘=1, sin 45˘=
2
1 1
= + =1
2 2 또한 45˘<x<90˘일 때, cos x<sin x<tan x이고
'3 '2 '2
⑵ sin 60˘= 이므로 <sin x<1, 0<cos x< 이므로
2 2 2
2x-30˘=60˘ ∴ x=45˘ cos 50˘<sin 45˘<sin 50˘<tan 45˘<tan 50˘
∴ tan x-cos 2x=tan 45˘-cos 90˘ ㄹ, ㅁ, ㄴ, ㄷ, ㄱ
=1-0=1
'2 0554 오른쪽 그림에서 sin x=AB”, C
⑶ sin 45˘=cos 45˘= 이므로 x=45˘
2 cos x=OB”, tan x=CD”이다.
A
∴ tan (x+15˘)+tan(75˘-x)=tan 60˘+tan 30˘ ① x의 값이 커지면 OB”의 길이는 짧아
'3 4'3 지므로 cos x의 값은 작아진다. 1 1
='3+ =
3 3
② sin x의 최솟값은 x=0˘일 때 0이고, x
4'3 O D
⑴1 ⑵1 ⑶ 최댓값은 x=90˘일 때 1이다. B
3
③ 0˘…x<45˘일 때 sin x<cos x이고
45˘<x…90˘일 때 sin x>cos x이므로
0549 2x-3y+6=0의 그래프가 y
B 0˘…x…90˘일 때 sin x>cos x라고 할 수 없다.
x축, y축과 만나는 점을 각각 A, B 2x-3y+6=0 2
④ tan 45˘=1이고 tan x의 값은 0에서 한없이 증가한다.
라고 하면 A(-3, 0), B(0, 2) a
A 또한 sin x의 최댓값은 1이므로 45˘…x<90˘일 때
즉, O’A”=3, OB”=2이므로 -3 O x
sin x<tan x이다.
AB”="√3¤ +2¤ ='ß13
'2
2 3 ⑤ x=45˘일 때 sin x=cos x= 이다.
∴ sin a+cos a= + 2
'ß13 'ß13
따라서 옳은 것은 ②, ④이다. ②, ④
5 5'ß13 5'ß13
∴ sin a+cos a= =
'ß13 13 13
0555 AB”="√13¤ -5¤ ='∂144=12
△ABCª△DBAª△DAC이므로
0550 OC”=OA”=12이고
∠ABC=∠x, ∠BCA=∠y
∠COD=180˘-120˘=60˘
AC” AB”
△COD에서 ∴ sin x+sin y= +
BC” BC”
CD” '3
5 12 17 17
sin 60˘= = 이므로 2CD”=12'3 = + =
12 2 13 13 13 13
∴ CD”=6'3 6'3
AC” 1
0556 △ABC에서 sin 30˘=
6
=
2
0551 (기울기)=tan 45˘=1
2AC”=6 ∴ AC”=3
직선 y=x+b가 점 (-4, 0)을 지나므로
1
0=-4+b ∴ b=4 ∠BAC=60˘이므로 ∠DAC= ∠A=30˘
2
∴ y=x+4 y=x+4 △ADC에서
y '3
tan 30˘= =
5 3 3
0552 sin A=
13
이므로 이를 만족하는 A
3y=3'3 ∴ y='3
직각삼각형 ABC를 그리면 오른쪽 그림과 '3 1
13 sin 30˘= =
같다. AD” 2

AC”="√13¤ -5¤ ='∂144=12이므로 90˘-A ∴ AD”=2'3

tan (90˘-A)=tan B B C ∠B=∠BAD=30˘이므로 △ABD는 이등변삼각형이다.


5
BD”=AD”=2'3 ∴ x=2'3
AC” 12
= = ④
BC” 5 ∴ x-y=2'3-'3='3 '3

04. 삼각비 51
(043~065)개념RPM3-2(정답) 2014.11.24 02:56 PM 페이지52 다민 2540DPI 175LPI

RPM 알피엠

0557 45˘<x<90˘일 때, sin x<tan x이므로


△DBC에서 cos 45˘=
10
=
'2
BD” 2
sin x-tan x<0, tan x-sin x>0
∴ (주어진 식)=-(sin x-tan x)-(tan x-sin x) 이므로
=0 ① '2 BD”=20 ∴ BD”=10'2
AB” 1
△ABD에서 cos 60˘= = 이므로
0558 AD”=BD”이고 ∠BDC=30˘이므로 10'2 2
1 2AB”=10'2 ∴ AB”=5'2
∠ABD= _30˘=15˘
2
AD” '3
∴ ∠ABC=15˘+60˘=75˘ sin 60˘= = 이므로
10'2 2
2 1 2AD”=10'6 ∴ AD”=5'6
△DBC에서 cos 60˘= =
BD” 2
∴ ABCD=△ABD+△DBC
∴ BD”=AD”=4
1 1
= _5'2_5'6+ _10_10
CD” 2 2
tan 60˘= ='3 ∴ CD”=2'3
2
=25'3+50
∴ AC”=4+2'3
⑵ 꼭짓점 A, D에서 변 BC에 내린 A 8 D
AC” 4+2'3 수선의 발을 각각 H, H'이라 하
∴ tan 75˘= = 6
BC” 2
자. △ABH에서 B C
=2+'3 2+'3 60˘ H 8 H'
AH” '3
sin 60˘= = 이므로
6 2
0559 4x¤ -2(1+'3)x+'3=0에서
2AH”=6'3 ∴ AH”=3'3
(2x-1)(2x-'3)=0
BH” 1
1 cos 60˘= = 이므로
∴ x= 또는 x=
'3 6 2
2 2
2BH”=6 ∴ BH”=3
이때 0˘<A<B<90˘이므로
CH'”=BH”=3이므로
sin A<sin B
AD”=HH'”=14-3-3=8
1 '3
즉, sin A= , sin B= 이므로 1
2 2 ∴ ABCD= _(8+14)_3'3
2
A=30˘, B=60˘
=33'3
'3 '3
∴ cos (B-A)=cos 30˘= ⑴ 25'3+50 ⑵ 33'3
2 2

0560 4x-3y+12=0에서 0562 0˘<x<45˘에서 0<tan x<1이므로


4 tan x+1>0, tan x-1<0
y= x+4
3
"√(tan x+1)¤ -"√(tan x-1)¤
△ABOª△AOH이므로
5
∠ABO=∠a =(tan x+1)+(tan x-1)=
6
4 5 5
(기울기)=tan B= 2 tan x= ∴ tan x=
3 6 12
A(0, 4), B(-3, 0), 즉 O’A”=4, OB”=3이므로 5
tan x= 인 직각삼각형 ABC를 그 A
12
AB”="√3¤ +4¤ ='ß25=5
5
리면 오른쪽 그림과 같으므로 x
O’A” 4 4 B
∴ sin a=sin B= = AB”="√12¤ +5¤ ='∂169=13 12 C
AB” 5 5
12 12
∴ cos x=
0561 ⑴ BD”를 그으면 D 13 13
∠DBC=∠BDC=45˘이므로 A 30˘
45˘
∠ABD=105˘-45˘=60˘ 0563 AD” ¤ =9_4=36

∠BDA=75˘-45˘=30˘ 60˘ ∴ AD”=6(∵ AD”>0)


45˘
B C
10 △ABD에서 AB”="√9¤ +6¤ ='∂117=3'ß13이고

52 정답과 풀이
(043~065)개념RPM3-2(정답) 2014.11.24 02:56 PM 페이지53 다민 2540DPI 175LPI

● 본문 076쪽 ~ 077쪽 ●

∠ABD=∠y이므로 △ABD에서 0566 △AEG는 ∠AEG=90˘인 직각삼각형이고


BD” 9 3 EG”="√4¤ +4¤ =4'2
tan x= = =
AD” 6 2
AG”="√4¤ +4¤ +4¤ =4'3 yy`
AD” 6 2'ß13
cos x= = = AE” 4 '3
AB” 3'ß13 13 ∴ sin x= = =
AG” 4'3 3
AD” 6 2'ß13
sin y= = = AE” 4 '2
AB” 3'ß13 13 tan x= = = yy`
EG” 4'2 2
3 2'ß13 2'ß13
∴ (주어진 식)= _{ + } ∴ '3 sin x+'2 tan x='3_
'3
+'2_
'2
2 13 13 3 2
3 4'ß13 =2 yy`
= _
2 13
2
6'ß13 6'ß13
=
13 13 단계 채점요소 배점
EG”, AG”의 길이 구하기 30%
sin x, tan x의 값 구하기 40%
1 1
0564 x¤ -x+ =0에서 {x- }¤ =0 '3 sin x+'2 tan x의 값 구하기 30%
4 2
1
∴ x= (중근) yy`
2 0567 △ABC는 ∠A=90˘인 직각삼각형이고
1 1 BC”=2_6=12이므로
이때 sin A= 이고 sin 30˘= 이므로 A=30˘ yy`
2 2
AC” AC” '3
tan 60˘+1 cos 30˘= = =
∴ (주어진 식)= -2 sin 90˘ BC” 12 2
tan 60˘-1
∴ AC”=6'3 yy`
'3+1
= -2_1 △ADC에서
'3-1
('3+1)¤ CD” CD” '3
= -2 cos 30˘= = =
('3-1)('3+1) AC” 6'3 2
=2+'3-2='3 yy` ∴ CD”=9
'3 AD” AD” 1
sin 30˘= = =
AC” 6'3 2
단계 채점요소 배점
∴ AD”=3'3 yy`
이차방정식의 해 구하기 20%
1
A의 값 구하기 30% ∴ △ADC= _CD”_AD”
2
주어진 식의 값 구하기 50%
1
= _9_3'3
2
27'3
= yy`
0565 △ABC에서 A 2
AB”="√3¤ -2¤ ='5이고 yy` E 2 27'3
'5
x 2
△ABCª△DEC (AA 닮음)이므 x y
B D C
로 3 단계 채점요소 배점
∠B=∠x yy` AC”의 길이 구하기 30%

'5 '5 5 CD”, AD”의 길이 구하기 40%


∴ cos x_tan y= _ = yy`
3 2 6 △ADC의 넓이 구하기 30%

5
6
BC”
0568
'2
△ABC에서 sin 45˘= = 이므로
1 2
단계 채점요소 배점
'2
AB”의 길이 구하기 20% 2BC”='2 ∴ BC”=
2
∠B=∠x임을 알기 30%
AB” '2
cos x_tan y의 값 구하기 50% cos 45˘= = 이므로
1 2

04. 삼각비 53
(043~065)개념RPM3-2(정답) 2014.11.24 02:56 PM 페이지54 다민 2540DPI 175LPI

RPM 알피엠

2AB”='2 ∴ AB”=
'2 0571 점 F에서 AD”에 내린 수선 A 6 E D
2 H x
의 발을 H라 하면 x 6
4
ED” ∠CEF=∠AEF(접은 각), x
tan 45˘= =1 ∴ ED”=1
1 B F C
∠AEF=∠CFE(엇각)
∴ BDEC=△ADE-△ABC
이므로 ∠CEF=∠CFE G
1 1 '2 '2
= _1_1- _ _ 즉, △EFC는 이등변삼각형이다.
2 2 2 2
1 1 FC”=EC”=AE”=6, CG”=AB”=4이므로
=
4 4 △CFG에서 FG”="√6¤ -4¤ ='2å0=2'5
AH”=BF”=FG”=2'5이므로
EH”=AE”-AH”=6-2'5
0569 점 E에서 BC”에 내린 수선의
A
D 따라서 △HFE에서
발을 H라 하고 EH”=CH”=x로 놓 E 60˘ HF” 4 3+'5 3+'5
12 tan x= = =
자. x EH” 6-2'5 2 2
30˘ 45˘
△EBH에서 ∠DBC=30˘, B
H x C
∠BEH=60˘이므로
BH”:x='3:1에서 BH”='3x
BC”=BH”+HC”='3x+x=('3+1)x
△BCD에서
('3+1)x
tan 60˘= ='3이므로
12
12'3
x= =18-6'3
'3+1
∴ BC”=('3+1)x=('3+1)(18-6'3)=12'3
1
∴ △EBC= _BC”_EH”
2
1
= _12'3_(18-6'3)
2
=108('3-1) 108('3-1)

0570 △ABC는 한 변의 길이가 6인 A


정삼각형이므로 6
'3
AM”= _6=3'3 B D
2 x
DM”=AM”=3'3이고 M H

꼭짓점 A에서 △BCD에 내린 수선의 C

발을 H라 하면 점 H는 △BCD의 무게중심이므로
1 1
MH”= DM”= _3'3='3
3 3
△AMH에서
AH”=øπ(3'3 )¤ -('3)¤ ='2å4=2'6이므로
AH” 2'6 2'2
sin x= = =
AM” 3'3 3
AH” 2'6
tan x= = =2'2
MH” '3
2'2 8 8
∴ sin x_tan x= _2'2=
3 3 3

54 정답과 풀이
(043~065)개념RPM3-2(정답) 2014.11.24 02:56 PM 페이지55 다민 2540DPI 175LPI

● 본문 077쪽 ~ 081쪽 ●

Ⅲ 삼각비

05 삼각비의활용 0589 BH”+CH”=100이므로


h(tan 30˘+tan 45˘)=100

= 11413
100 100
∴ h=
0572 6, 4'3, 6, 2'3 tan 30˘+tan 45˘
21 +1
'3 `
3
∴ h=50(3-'3) 50(3-'3 )
0573 4, 2'2, 4, 2'2

0574 9, 6'3, 9, 3'3 0590 ∠BAH=60˘, ∠CAH=30˘

0575 x=10 sin 50˘=10_0.77=7.7


y=10 cos 50˘=10_0.64=6.4 x=7.7, y=6.4
0591 BH”=h tan 60˘, CH”=h tan 30˘

0576 ∠C=180˘-(90˘+55˘)=35˘이므로 0592 BH”-CH”=100이므로


x=5 sin 35˘=5_0.57=2.85 h(tan 60˘-tan 30˘)=100

= 114123 =50'3
y=5 cos 35˘=5_0.82=4.1 x=2.85, y=4.1 100 100
∴ h= 50'3
tan 60˘-tan 30˘
'3-`21
'3
3
1
0577 A’H=
” 8 sin 30˘=8_
2
=4 4

1
0593 △ABC=
2
_4_6_sin 60˘
0578
'3
BH”=8 cos 30˘=8_ =4'3 4'3
2 1 '3
= _4_6_ =6'3 6'3
2 2

0579 CH”=BC”-BH”=6'3-4'3=2'3 2'3

1
0594 △ABC= _3'2_8_sin 45˘
0580 ’ ” ¤ +CH” ¤ ="√4¤ +(2'3)¤
AC”=øπAH 2
='ß28=2'7 2'7 1 '2
= _3'2_8_ =12 12
2 2

0581 ∠C=180˘-(30˘+105˘)=45˘ 45˘


1
0595 △ABC=
2
_4_8_sin 30˘
1
0582 BH”=4 sin 30˘=4_
2
=2 2
1 1
= _4_8_ =8 8
2 2

2
0583
'2
BC”= =2÷ =2'2 2'2
sin 45˘ 2
1
0596 △ABC=
2
_10_6_sin(180˘-135˘)

0584 ∠C=180˘-(60˘+75˘)=45˘ 45˘ 1


= _10_6_sin 45˘
2
1
0585
'2 '2
A’H=
” 9'2 sin 45˘=9'2_ =9 9 = _10_6_ =15'2 15'2
2 2 2

9
0586
'3
AB”= =9÷ =6'3 6'3 1
sin 60˘ 2 0597 △ABC=
2
_6_6'3_sin (180˘-150˘)

1
= _6_6'3_sin 30˘
0587 ∠BAH=30˘, ∠CAH=45˘ 2
1 1
= _6_6'3_ =9'3 9'3
0588 BH”=h tan 30˘, CH”=h tan 45˘ 2 2

05. 삼각비의 활용 55
(043~065)개념RPM3-2(정답) 2014.11.24 02:56 PM 페이지56 다민 2540DPI 175LPI

RPM 알피엠

1 1
0598 △ABC=
2
_3_10_sin(180˘-120˘) 0606 ABCD=
2
_12_10_sin(180˘-120˘)

1 1
= _3_10_sin 60˘ = _12_10_sin 60˘
2 2
1 '3 1 '3
= _3_10_ = _12_10_ =30'3 30'3
2 2 2 2
15'3 15'3
=
2 2 1
0607 ABCD=
2
_(3+6)_2_sin 60˘

1 9'3 9'3
0599 ABCD=8_6_sin 45˘ =
2
_9_2_
'3
2
=
2 2
'2
=8_6_ =24'2 24'2
2
1
0608 ABCD=
2
_(4+8)_3'2_sin 45˘
0600 ABCD=4_3'3_sin 60˘
1 '2
= _12_3'2_ =18 18
'3 2 2
=4_3'3_
2
=18 18 y x
0609 AB”=
sin 34˘
또는 AB”=
cos 34˘

0601 ABCD=5_4_sin(180˘-120˘) 또한 ∠A=180˘-(90˘+34˘)=56˘이므로


=5_4_sin 60˘ x y
AB”= 또는 AB”= ①, ②
sin 56˘ cos 56˘
'3
=5_4_ =10'3 10'3
2
c
0610 ⑤ sin C=
b
이므로 c=b sin C
0602 ABCD=6_3_sin(180˘-135˘)
따라서 옳지 않은 것은 ⑤이다. ⑤
=6_3_sin 45˘

0611
'2
=6_3_ =9'2 9'2 AB”=10 cos 43˘=10_0.73=7.3
2
AC”=10 sin 43˘=10_0.68=6.8
따라서 AB”와 AC”의 길이의 차는
1
0603 ABCD=
2
_16_14_sin 45˘ 7.3-6.8=0.5 0.5
1 '2
= _16_14_
2 2 0612 BC”='6 tan 60˘='6_'3=3'2
=56'2 56'2 BC” '2
∴ BD”= =3'2÷ =6 6
sin 45˘ 2

1
0604 ABCD=
2
_8_10_sin 60˘
0613 EG”="√3¤ +3¤ ='1å8=3'2(cm)
1 '3 △CEG에서
= _8_10_
2 2 CG”=3'2 tan 60˘=3'2_'3=3'6(cm)
=20'3 20'3 따라서 직육면체의 부피는
3_3_3'6=27'6(cm‹ ) ⑤
1
0605 ABCD=
2
_8_8_sin(180˘-135˘)
1
1
0614 GH”=6 cos 60˘=6_
2
=3(cm)
= _8_8_sin 45˘
2
'3
DH”=6 sin 60˘=6_ =3'3(cm)
1 '2 2
= _8_8_
2 2 따라서 직육면체의 부피는
=16'2 16'2 5_3_3'3=45'3(cm‹ ) 45'3 cm‹

56 정답과 풀이
(043~065)개념RPM3-2(정답) 2014.11.24 02:56 PM 페이지57 다민 2540DPI 175LPI

● 본문 081쪽 ~ 083쪽 ●

0615 AB”=8'2 cos 45˘=8'2_


'2
=8(cm) 0621 꼭짓점 A에서 BC”에 내린 수 A
2
선의 발을 H라고 하면
'2 10 cm
AC”=8'2 sin 45˘=8'2_ =8(cm) AH”=10 sin 60˘=10_
'3
2 2 60˘
B C
따라서 삼각기둥의 부피는 H
=5'3(cm) 12 cm
1
{ _8_8}_6=192(cm‹ ) ② 1
2 BH”=10 cos 60˘=10_ =5(cm)
2
CH”=12-5=7(cm)이므로 △AHC에서
0616
'3
AH”=6 sin 60˘=6_ =3'3(cm) AC”=øπ(5'3)¤ +7¤ ='∂124=2'3å1(cm) ③
2
1
BH”=6 cos 60˘=6_
2
=3(cm) 0622 꼭짓점 A에서 BC”에 내린 수 A

따라서 원뿔의 부피는 선의 발을 H라고 하면 6"2


45˘
1 A’H=
” 6'2 sin 45˘=6'2_
'2
=6 B
H
C
_p_3¤ _3'3=9'3p(cm‹ ) 9'3p cm‹ 2
3 14
'2
BH”=6'2 cos 45˘=6'2_ =6
2
0617 OH”=QR”=30 m이므로
CH”=14-6=8이므로 △AHC에서
'3
PH”=30 tan 30˘=30_ AC”="√6¤ +8¤ ='∂100=10 10
3
=10'3(m)
HQ”=30 tan 45˘=30_1 0623 꼭짓점 A에서 BC”에 내린 수선의 A
=30(m) 발을 H라고 하면
10
∴ PQ”=PH”+HQ” 3
CH”=10 cos C=10_ =6
=10'3+30(m) (10'3+30) m 5
B C
H
A’H”="√10¤ -6¤ ='ß64=8 10

0618 점 B에서 OA”에 내린 수선의 O BH”=10-6=4이므로 △ABH에서


30˘ AB”="√4¤ +8¤ ='ß80=4'5 4'5
발을 H라고 하면
'3 12 cm
OH”=12 cos 30˘=12_
2
=6'3(cm) H 0624 꼭짓점 A에서 BC”의 연장선에 내 A
B 린 수선의 발을 H라고 하면
∴ AH”=OA”-OH” A
∠ACH=180˘-120˘=60˘이므로 6
=12-6'3 120æ
yy` B
=6(2-'3)(cm) ③ 3 C H
'3
A’H=
” 6 sin 60˘=6_
2
0619 =3'3
'3
BH”=20 cos 30˘=20_ =10'3(m)
2
1
∴ CH”=10'3 tan 45˘=10'3_1=10'3(m) 10'3 m CH”=6 cos 60˘=6_
2
=3 yy`

0620 DB”=15 tan 60˘=15_'3=15'3(m) yy` BH”=BC”+CH”=3+3=6이므로


CB”=15 tan 45˘=15_1=15(m) yy` △ABH에서
∴ CD”=DB”-CB”=15('3-1)(m) yy` AB”="√6¤ +(3'3)¤ ='ß63=3'7 yy`
15('3-1) m 3'7

단계 채점요소 배점 단계 채점요소 배점
DB”의 길이 구하기 40% ∠ACH의 크기 구하기 20%
CB”의 길이 구하기 40% AH”, CH”의 길이 구하기 40%
CD”의 길이 구하기 20% AB”의 길이 구하기 40%

05. 삼각비의 활용 57
(043~065)개념RPM3-2(정답) 2014.11.24 02:56 PM 페이지58 다민 2540DPI 175LPI

RPM 알피엠

0625 꼭짓점 A에서 BC” 에 내린 A 0629 ” h라고 하면


A’H= A
45æ
수선의 발을 H라고 하면 ∠BAH=30˘, ∠CAH=45˘이므로 30æ
9 '2 cm h
'2 '3 60æ
CH”=9'2 cos 45˘=9'2_ BH”=h tan 30˘= h B 45æ C
2 60˘ 45˘ 3 H
B C 10
=9(cm) H CH”=h tan 45˘=h
'2 '3+3
AH”=9'2 sin 45˘=9'2_ =9(cm) BC”=BH”+CH”={ }h=10
2 3
△ABH에서 ∴ h=5(3-'3) 5(3-'3 )
9 9
BH”= = =3'3(cm)
tan 60˘ '3
∴ BC”=3('3+3)(cm) 3('3+3) cm 0630 ” h라고 하면
A’H= A
45æ 40æ
∠BAH=45˘, ∠CAH=40˘이므로
h
BH”=h tan 45˘=h
0626 꼭짓점 A에서 BC”에 내린 수 A B
45æ
H
50æ C
75˘ CH”=h tan 40˘ 13
선의 발을 H라고 하면
12 BC”=BH”+CH”
'3
A’H=
” 12 sin 60˘=12_ =(1+tan 40˘)h=13
2 45˘ 60˘
B C
H
=6'3 13
∴ h= ④
1+tan 40˘
∠B=180˘-(75˘+60˘)=45˘이므로
6'3 '2
AB”= =6'3÷
sin 45˘ 2 0631 A지점에서 BC”에 내린 수 A
=6'6 6'6 45˘ 60˘
선의 발을 H라고 하자.
hm
” h m라고 하면
A’H= B
45˘ 30˘ C
H
0627 꼭짓점 B에서 AC”에 내린 수선 A 45˘ ∠BAH=45˘, ∠CAH=60˘이 60 m

의 발을 H라고 하면 H 므로
1 BH”=h tan 45˘=h(m)
BH”=12 sin 30˘=12_ =6 105˘ 30˘
2 B C CH”=h tan 60˘='3h(m)
12
∠A=180˘-(105˘+30˘)=45˘이므로 BC”=BH”+CH”
6 '2 =(1+'3)h=60
AB”= =6÷
sin 45˘ 2
60
=6'2 6'2 ∴ h= =30('3-1)(m)
1+'3
30('3-1) m
0628 꼭짓점 A에서 BC”에 내린 수선 A
의 발을 H라 하고 A’H=
” x라고 하면
△ABH에서 BH”=A’H”=x
x 0632 꼭짓점 A에서 BC”에 내린 수 A
45˘ 60˘ 45˘
B H C 선의 발을 H라고 하자. 30˘
또한 △AHC에서 h cm
("3+1) cm ” h cm라고 하면
A’H=
x x '3 60˘
CH”= = = x ∠BAH=30˘, ∠CAH=45˘ B
45˘
C
tan 60˘ '3 3 H
6 cm
그런데 BC”=BH”+CH”이므로 이므로
'3
'3
x BH”=h tan 30˘= h(cm)
'3+1=x+
3 3
3+'3 CH”=h tan 45˘=h(cm)
={ }x
3 '3+3
BC”=BH”+CH”={ }h=6
3('3+1) 3
∴ x= ='3(cm)
3+'3 ∴ h=3(3-'3)(cm)
'3 '2 1
∴ AB”= ='3÷ ∴ △ABC= _6_3(3-'3)
sin 45˘ 2 2
='6(cm) '6 cm =9(3-'3)(cm¤ ) 9(3-'3 ) cm¤

58 정답과 풀이
(043~065)개념RPM3-2(정답) 2014.11.24 02:56 PM 페이지59 다민 2540DPI 175LPI

● 본문 084쪽 ~ 086쪽 ●

0633 AH”=h m라고 하면


60˘
A
0637 △ABC=
1
_6_10_sin B=15'2이므로
2
∠BAH=60˘, ∠CAH=30˘
이므로 sin B=
'2
∴ ∠B=45˘ 45˘
30˘ 2
BH”=h tan 60˘='3h(m) 30˘ 60˘
B 300 m C H

0638
'3
CH”=h tan 30˘= h(m) ∠B=∠C=75˘이므로
3
∠A=180˘-(75˘+75˘)=30˘
2'3
BC”=BH”-CH”이므로 h=300
3 1
∴ △ABC= _8_8_sin 30˘
2
∴ h=150'3(m) 150'3 m
1 1
= _8_8_ =16 ③
2 2

0634 ” h라고 하면
A’H= A
∠BAH=58˘, ∠CAH=34˘이므로
58˘
34˘ 0639 tan B='3이므로 ∠B=60˘
h 1
BH”=h tan 58˘ ∴ △ABC= _9_12_sin 60˘
32˘ 56˘ 2
CH”=h tan 34˘ B
15 C H 1 '3
= _9_12_ =27'3 27'3
BC”=BH”-CH” 2 2
=(tan 58˘-tan 34˘)h=15
15 1
∴ h=
tan 58˘-tan 34˘
① 0640 △ABC=
2
_8_12_sin 45˘

1 '2
= _8_12_ =24'2
2 2
0635 CH”=h m라고 하면 60˘
C 1 1
∴ △ABG= △ABC= _24'2=8'2
∠ACH=60˘, ∠BCH=45˘이므로 45˘ 3 3
hm
A’H=
” h tan 60˘='3h(m) 8'2
45˘
A 30˘
BH”=h tan 45˘=h(m) 10 m B H

AB”=A’H”-BH”=('3-1)h=10 0641 ∠A=180˘-(32˘+28˘)=120˘이므로


10 1
∴ h= =5('3+1)(m) 5('3+1) m △ABC= _8_10_sin (180˘-120˘)
'3-1 2
1
= _8_10_sin 60˘
2
0636 ” h cm라고 하면
A’H=
45˘
A
=
1
_8_10_
'3
=20'3 ④
∠BAH=45˘, ∠CAH=30˘이므로 2 2

BH”=h tan 45˘=h(cm) 30˘ h cm


1
CH”=h tan 30˘=
'3
h(cm) 45˘ 60˘ 0642 △ABC=
2
_6_4_sin (180˘-B)=6'2이므로
3 B H
20 cm C
'2
yy` sin(180˘-B)=
2
3-'3
BC”=BH”-CH”={ }h=20 즉, 180˘-B=45˘이므로
3
B=135˘ 135˘
∴ h=10(3+'3)(cm) yy`

1
∴ △ABC= _20_10(3+'3) 0643 △ABD+△ADC=△ABC이므로
2
=100(3+'3)(cm¤ ) yy` 1 1
_10_AD”_sin 60˘+ _AD”_6_sin 60˘
2 2
100(3+'3) cm¤
1
= _10_6_sin (180˘-120˘)
2
단계 채점요소 배점
A’H”=h cm로 놓고 BH”, CH”의 길이를 h로 나타내기 40%
5'3 3'3
AD”+ AD”=15'3, 4'3 AD”=15'3
2 2
A’H의
” 길이 구하기 40%
15 15
△ABC의 넓이 구하기 20% ∴ AD”=
4 4

05. 삼각비의 활용 59
(043~065)개념RPM3-2(정답) 2014.11.24 02:56 PM 페이지60 다민 2540DPI 175LPI

RPM 알피엠

0644 BC”=BD”=8이므로 ∴ (정육각형의 넓이)=6△AOB=6_4'3


1 =24'3(cm¤ ) 24'3 cm¤
AB”=8 sin 30˘=8_ =4 yy`
2
∠ABC=180˘-(90˘+30˘)=60˘이므로 0648 꼭짓점 A에서 BC”에 내린 A D
∠ABD=60˘+90˘=150˘ yy`
수선의 발을 H라고 하면 6"2 6
1 BH”=
” 6'2 cos 45˘ 45˘ 30˘
∴ △ABD= _4_8_sin (180˘-150˘) B C
2 H
'2 14
1 =6'2_ =6
= _4_8_sin 30˘ 2
2
이므로 AH”=BH”=6 yy`
1 1
= _4_8_ CH”=14-6=8이므로
2 2
=8 yy` AC”="√6¤ +8¤ ='∂100=10 yy`
8 ∴ ABCD
=△ABC+△ACD
단계 채점요소 배점
1 1
AB”의 길이 구하기 40% ∴= _6'2_14_sin 45˘+ _10_6_sin 30˘
2 2
∠ABD의 크기 구하기 20%
1 '2 1 1
△ABD의 넓이 구하기 40% = _6'2_14_ + _10_6_
2 2 2 2
∴ =42+15=57 yy`

0645 대각선 AC를 그으면 A


5 cm
57
ABCD D
단계 채점요소 배점
5"3 cm
=△ABC+△ACD 120˘
5 cm A’H”, BH”의 길이 구하기 30%
1 60˘ AC”의 길이 구하기 30%
= _5'3_5'3_sin 60˘ B
2 5"3 cm C
ABCD의 넓이 구하기 40%
1
+ _5_5_sin (180˘-120˘)
2

=
1
_5'3_5'3_
'3 1
+ _5_5_
'3 0649 마름모 ABCD의 한 변의 길이를 x cm라고 하면
2 2 2 2
ABCD=x_x_sin (180˘-135˘)=18'2
75'3 25'3
= + =25'3(cm¤ ) ⑤ x_x_sin 45˘=18'2
4 4
'2
x_x_ =18'2
2
2'3 1
0646 AC”=
cos 60˘
=2'3÷ =4'3
2
x¤ =36 ∴ x=6(cm) (∵ x>0) ③

∴ ABCD
=△ABC+△ACD 0650 ⑴ DC”=AB”=6이므로

1 1 ABCD=8_6_sin(180˘-135˘)
= _2'3_4'3_sin 60˘+ _4'3_4'2_sin 45˘
2 2 =8_6_sin 45˘
1 '3 1 '2
= _2'3_4'3_ + _4'3_4'2_ =8_6_
'2
=24'2
2 2 2 2 2
∴ =6'3+8'3 ⑵ ∠B+∠C=180˘이므로
=14'3 14'3 ∠DBC=180˘-(35˘+100˘)=45˘
1
△DBC= _14_8_sin 45˘
2
0647 오른쪽 그림과 같이 정육각형은 A F
합동인 6개의 정삼각형으로 나눌 수 있다. 1 '2
4 cm
= _14_8_
2 2
1 B E
△AOB= _4_4_sin 60˘ O =28'2
2
∴ ABCD=2△DBC
1 '3 C D
= _4_4_ =2_28'2=56'2
2 2
△AOB=4'3(cm¤ ) ⑴ 24'2 ⑵ 56'2

60 정답과 풀이
(043~065)개념RPM3-2(정답) 2014.11.24 02:56 PM 페이지61 다민 2540DPI 175LPI

● 본문 086쪽 ~ 088쪽 ●

0651 ∠B=∠D=120˘이므로 0656 두 대각선이 이루는 각의 크기를 x(0˘<x…90˘)라고


ABCD=5_8_sin(180˘-120˘) 하면
=5_8_sin 60˘ 1
ABCD= _8_10_sin x=40 sin x
'3 2
=5_8_
2 따라서 0˘<x…90˘에서 sin x의 값은 x=90˘일 때, 최댓값 1
=20'3(cm¤ ) 을 가지므로 ABCD의 넓이의 최댓값은 40이다. 40
1 1
∴ △ABO= ABCD= _20'3=5'3(cm¤ )
4 4
5'3 cm¤ 15 15
0657 cos 40˘=
x
이므로 x=
cos 40˘
y
tan 40˘= 이므로
0652 ∠A+∠B=180˘이므로 15
1 y=15 tan 40˘ ⑤
∠B=180˘_ =45˘
3+1
∴ ABCD=10_12_sin 45˘
1
0658
'2
=10_12_ △ABC= _4_7_sin(180˘-∠C)=7'2이므로
2 2
=60'2 '2
sin(180˘-∠C)=
2
1 1
∴ △AMC= △ABC= ABCD
2 4 180˘-∠C=45˘ ∴ ∠C=135˘ 135˘
1
= _60'2=15'2 15'2
4

0659
'2
cos C= 이므로 ∠C=45˘
2
1
0653 ABCD=
2
_6_BD”_sin (180˘-135˘) ∴ ∠B=180˘-(15˘+45˘)=120˘
1 1
= _6_BD”_sin 45˘ ∴ △ABC= _4_2('3-1)_sin (180˘-120˘)
2 2

1 1
= _6_BD”_
'2
=12'2 = _4_2('3-1)_sin 60˘
2 2 2
1 '3
∴ BD”=8 8 = _4_2('3-1)_
2 2
∴ △ABC=6-2'3 6-2'3
0654 등변사다리꼴의 두 대각선의 길이는 같으므로
BD”=AC”=8
1 0660
'3
∴ ABCD= _8_8_sin(180˘-120˘) FG”=8 cos 30˘=8_ =4'3
2 2
1 1
= _8_8_sin 60˘ GH”=8 sin 30˘=8_ =4
2 2
1 '3 ∴ (직육면체의 부피)=4'3_4_3=48'3 ②
= _8_8_ =16'3 ⑤
2 2

0655 두 대각선의 교점을 O라고 하면 △OBC에서 0661 △ABH에서


∠BOC=180˘-(26˘+34˘)=120˘ 1
BH”=4 cos 60˘=4_ =2(cm)
2
1
∴ ABCD= _16_12_sin(180˘-120˘) BC”=2+5=7(cm)이므로
2
1 1
= _16_12_sin 60˘ △ABC= _7_4_sin 60˘
2 2
1 '3 1 '3
= _16_12_ =48'3 = _7_4_
2 2 2 2
48'3 =7'3(cm¤ ) ③

05. 삼각비의 활용 61
(043~065)개념RPM3-2(정답) 2014.11.24 02:56 PM 페이지62 다민 2540DPI 175LPI

RPM 알피엠

0662 BC”=10 m이므로 0667 두 대각선이 이루는 각의 크기를 x(0˘<x…90˘)라고


AC”=10 tan 60˘=10'3(m) 하면
∴ A’H”=AC”+CH”=10'3+1.5(m) (10'3+1.5) m 1
ABCD= _8_9_sin x=36 sin x
2
따라서 0˘<x…90˘에서 sin x의 값은 x=90˘일 때, 최댓값 1
18
0663
'3
AB”= =18÷ =12'3(m) 을 가지므로 ABCD의 넓이의 최댓값은 36 cm¤ 이다. ②
cos 30˘ 2
'3
AC”=18 tan 30˘=18_
3
=6'3(m) 0668 등변사다리꼴의 두 대각선의 길이는 같으므로
따라서 나무의 처음 높이는 AC”=BD”=x cm라고 하면
12'3+6'3=18'3(m) ③ 1
ABCD= _x_x_sin (180˘-120˘)
2
1 '3
= _x_x_ =9'3
2 2
0664 ABCD=8_12_sin(180˘-120˘)
=8_12_sin 60˘ x¤ =36 ∴ x=6(cm) (∵ x>0) ③

'3
=8_12_ =48'3(cm¤ )
2 0669 PH”=N’Q=
” 50 m이므로 R
1 1 RH”=50 tan 45˘=50(m)
∴ △BMD= △BCD= ABCD 45˘
2 4 P H
H’Q”=50 tan 60˘=50'3(m)
1 60˘
= _48'3=12'3(cm¤ ) 12'3 cm¤ ∴ RQ”=RH”+H’Q”
4 50 m
=50+50'3
Q
=50(1+'3)(m)
0665 꼭짓점 D에서 BC”의 연장 A D
50(1+'3 ) m
선에 내린 수선의 발을 H라고 하 4 cm 135˘
45˘
면 ∠DCH=45˘이므로 B H
4"2 cm C 0670 ⑴ ∠BAH=60˘, A
CH”=CD” cos 45˘ 60˘
∠CAH=45˘이므로 45˘
'2 x
=4_ BH”=x tan 60˘='3x
2 30˘ 45˘
CH”=x tan 45˘=x B
=2'2(cm) 10 C H
BC”=BH”-CH”=('3-1)x=10
D’H”=CH”=2'2 cm
10
BH”=BC”+CH”=4'2+2'2=6'2(cm) ∴ x= =5('3+1)
'3-1
따라서 △BHD에서
⑵ ∠BAH=60˘, 60˘
A
BD”="√(6'2 )¤ +(2'2)¤
∠CAH=30˘이므로
='ß80=4'5(cm) 4'5 cm 30˘ x
BH”=x tan 60˘='3x
30˘ 60˘
B
CH”=x tan 30˘=
'3
x 80 C H
3
0666 대각선 BD를 그으면
3
D
2'3
ABCD A BC”=BH”-CH”= x=80
3"7
3
=△ABD+△BCD 150˘
3"3 ∴ x=40'3
1 60˘
= _3_3'3_sin(180˘-150˘) B C ⑴ 5('3+1) ⑵ 40'3
2 3"7
1
+ _3'7_3'7_sin 60˘
2 0671 꼭짓점 A에서 BC”에 내린 60˘
A
45˘
1 1 수선의 발을 H라고 하자.
= _3_3'3_sin 30˘+ _3'7_3'7_sin 60˘ h
2 2 30˘ 45˘
” h라고 하면
A’H= B C
1 1 1 H
= _3_3'3_ + _3'7_3'7_
'3 10
2 2 2 2 ∠BAH=60˘, ∠CAH=45˘이므로

9'3 63'3 BH”=h tan 60˘='3h


= + =18'3 ⑤
4 4 CH”=h tan 45˘=h

62 정답과 풀이
(043~065)개념RPM3-2(정답) 2014.11.24 02:56 PM 페이지63 다민 2540DPI 175LPI

● 본문 088쪽 ~ 090쪽 ●

BC”=BH”+CH”=('3+1)h=10 0676 △ABC에서


10
∴ h= =5('3-1) AC”=8 sin 60˘=8_
'3
=4'3(cm)
'3+1 2
h '2 ∠ACE=30˘+90˘=120˘이므로
∴ AC”= =5('3-1)÷
cos 45˘ 2 1
△AEC= _4'3_8_sin(180˘-120˘)
=5('6-'2) ② 2
1
= _4'3_8_sin 60˘
2
0672 두 꼭짓점 A, D에서 BC”에 A 6 cm D
1 '3
내린 수선의 발을 각각 H, H'이라 60˘ = _4'3_8_ =24(cm¤ ) 24 cm¤
2 2
6 cm
고 하면 B C
2 cmH H' 2 cm
H’H'”=AD”=6 cm이므로 0677 ∠COD=180˘-120˘=60˘이므로
1 △COD에서
BH”= _(10-6)=2(cm)
2 1
OD”=8 cos 60˘=8_ =4(cm)
A’H=
” 2 tan 60˘=2'3(cm)이므로 2
1 CD”=8 sin 60˘=8_
'3
=4'3(cm)
ABCD= _(6+10)_2'3 2
2
=16'3(cm¤ ) ④ 따라서 AD”=8+4=12(cm)이므로
1
△CAD= _12_4'3=24'3(cm¤ ) 24'3 cm¤
2
0673 꼭짓점 A에서 BC”에 내린 45æ A
다른풀이
수선의 발을 H라고 하면 30`m 60æ
1
45æ △CAO= _8_8_sin(180˘-120˘)
∠BAH=45˘, ∠CAH=60˘ B C 2
H
이므로 1
= _8_8_sin 60˘
'2 2
A’H=
” 30 sin 45˘=30_ =15'2(m)
2 1 '3
= _8_8_ =16'3(cm¤ )
A’H” 1 2 2
∴ AC”= =15'2÷
cos 60˘ 2 △COD에서 ∠COD=180˘-120˘=60˘이고
=30'2(m) ④ 1
OD”=8 cos 60˘=8_ =4(cm)이므로
2
1
0674 점 B에서 O’A”에 내린 수선 O △COD=
2
_8_4_sin 60˘
의 발을 H라고 하면 20 cm 45˘
1 '3
= _8_4_ =8'3(cm¤ )
△OBH에서 2 2
B H B'
'2 ∴ △CAD=16'3+8'3=24'3(cm¤ )
OH”=20 cos 45˘=20_
2 A

=10'2(cm) 0678 ∠BAC=∠DAC=x라고 하면


∴ A’H=
” O’A”-OH”=20-10'2 1
△ABC= _16_AC”_sin x=112이므로
=10(2-'2 )(cm) 2
따라서 점 B는 점 A를 기준으로 10(2-'2) cm의 높이에 AC” sin x=14
있다. 10(2-'2) cm 1
∴ △ACD= _10_AC”_sin x
2
1
0675 ∠A+∠B=180˘이므로 = _10_14=70(cm¤ ) ⑤
2
1
∠B=180˘_ =30˘
5+1
0679 꼭짓점 C에서 B’A”의 연장 H
1 a
ABCD=4_6_sin 30˘=4_6_ =12 선에 내린 수선의 발을 H라고 하면 A a
2 45æ
∠HAC=30˘+15˘=45˘이므로 30æ 45æ
1 1 B C
∴ △OBC= ABCD= _12=3 3 ∠ACH=45˘
15æ
4 4

05. 삼각비의 활용 63
(043~065)개념RPM3-2(정답) 2014.11.24 02:56 PM 페이지64 다민 2540DPI 175LPI

RPM 알피엠

” HC”=a라고 하면
H’A= 단계 채점요소 배점
a '2 △ABC의 넓이 구하기 40%
AC”= =a÷ ='2a
sin 45˘ 2
AD”에 대한 방정식 세우기 50%
a 1 AD”의 길이 구하기 10%
BC”= =a÷ =2a
sin 30˘ 2
a '3
BH”= =a÷ ='3a
tan 30˘ 3
∴ AB”=BH”-A’H”='3a-a=('3-1)a
0682 △ABC에서
1
△ABC= _('3-1)a_2a_sin 30˘=2('3-1)이므로 '3
2 AC”=12 sin 60˘=12_ =6'3(cm)이고 yy`
2
1 1
_('3-1)a_2a_ =2('3-1) ∠BCA=180˘-(60˘+90˘)=30˘이므로
2 2
a¤ =4 ∴ a=2 (∵ a>0) 1
△ABC= _12_6'3_sin 30˘
2
따라서 AB”=2('3-1), BC”=4, AC”=2'2이므로
1 1
(△ABC의 둘레의 길이)=2('3-1)+4+2'2 = _12_6'3_ =18'3(cm¤ ) yy`
2 2
=2'3+2'2+2 1
△ACD= _6'3_9_sin 30˘
2'3+2'2+2 2
1 1 27'3
0680 정팔각형은 8개의 합동인 이등 H =
2
_6'3_9_ =
2 2
(cm¤ ) yy`
A G
변삼각형으로 나누어지므로 ∴ ABCD=△ABC+△ACD
O
1 x
∠FOE=360˘_ =45˘ B 45æ F 27'3 63'3
8 =18'3+ = (cm¤ ) yy`
x 2 2
△OEF에서 OE”=OF”=x라고 하면 C E
63'3
D cm¤
1 2
△OEF= _x_x_sin 45˘
2
단계 채점요소 배점
1 '2
= _x_x_ AC”의 길이 구하기 20%
2 2
△ABC의 넓이 구하기 30%
'2
△OEF= x¤ △ACD의 넓이 구하기 30%
4
ABCD의 넓이 구하기 20%
(정팔각형의 넓이)=8△OEF=50'2이므로
'2
8_ x¤ =50'2
4
x¤ =25 ∴ x=5(∵ x>0)
0683 점 A에서 BC”에 내린 수선 A
∴ AE”=2 OE”=2_5=10 10
의 발을 H라고 하면 30˘
45˘ 3'2 cm
1 ∠BAH=30˘, ∠CAH=45˘이므로
0681 △ABC= _12_6_sin(180˘-120˘)
2 AH”=3'2 cos 45˘ 60˘
B C
1 H
= _12_6_sin 60˘ =3'2_
'2
=3(cm) yy`
2 2
1 '3
= _12_6_ =18'3(cm¤ ) yy`
HC”=3'2 sin 45˘=3'2_
'2
=3(cm)
2 2 2
이때 △ABC=△ABD+△ADC이므로 '3
BH”=3 tan 30˘=3_ ='3(cm)
AD”=x cm라고 하면 3
1 1 ∴ BC”=BH”+HC”=3+'3(cm) yy`
18'3= _12_x_sin 60˘+ _6_x_sin 60˘이므로
2 2
1
∴ △ABC= _(3+'3)_3
3'3 2
18'3=3'3x+ x yy`
2
3(3+'3)
= (cm¤ ) yy`
9'3 2
x=18'3 ∴ x=4(cm) yy`
2
3(3+'3)
cm¤
4 cm 2

64 정답과 풀이
(043~065)개념RPM3-2(정답) 2014.11.24 02:56 PM 페이지65 다민 2540DPI 175LPI

● 본문 090쪽 ~ 091쪽 ●

단계 채점요소 배점 BC”=BH”+CH”=(1+'3)h=10
AH”의 길이 구하기 20% 10
∴ h= =5('3-1)(cm)
BC”의 길이 구하기 60% 1+'3
△ABC의 넓이 구하기 20% 1
∴ △EBC= _10_5('3-1)
2
=25('3-1)(cm¤ )
0684 점 A에서 BC”의 연장선 A D
25('3-1) cm¤
에 내린 수선의 발을 H라고 하면 3 cm 120˘
C
” 3 cm, ∠ABH=60˘이므로
A’H= H B
3 '3 0687 MN”을 그으면
AB”= =3÷
sin 60˘ 2 △AMN= ABCD-(△ABM+△MCN+△AND)
=2'3(cm) yy` 1 1 1
=4_4-{ _2_4+ _2_2+ _4_2}
2 2 2
∠DAC=∠BAC(접은 각), ∠DAC=∠BCA(엇각)이므로
=16-10
∠BAC=∠BCA
=6(cm¤ ) A D
즉, △ABC는 AB”=BC”인 이등변삼각형이다. yy`
AM”="√4¤ +2¤ ='2å0=2'5(cm), x
1 4 cm
∴ △ABC= _AB”_BC”_sin(180˘-120˘) AN”="√4¤ +2¤ ='2å0=2'5(cm)이므로 N
2
1 1
= _2'3_2'3_sin 60˘ △AMN= _2'5_2'5_sin x=6 B M C
2 2
1 '3 3 3
= _2'3_2'3_ 10 sin x=6 ∴ sin x=
2 2 5 5
∴ △ABC=3'3(cm¤ ) yy`

3'3 cm¤ 0688 오른쪽 그림에서 겹쳐진 3 cm


45˘
부분, 즉 ABCD는 평행사변형 A D
단계 채점요소 배점
이다. 4 cm 45˘
AB”의 길이 구하기 40% B 45˘
3 '2 C
△ABC는 AB”=BC”인 이등변삼각형임을 알기 AD”= =3÷

cm
30%
sin 45˘ 2

3
△ABC의 넓이 구하기 30%
=3'2(cm)
∴ ABCD=3'2_4=12'2(cm¤ ) 12'2 cm¤
0685 OC”를 그으면 O’A”=OC”이므로
150˘ C
∠ACO=15˘
∠AOC=180˘-(15˘+15˘) A B
15˘ O 8 cm
=150˘
∴ (색칠한 부분의 넓이)
=(부채꼴 AOC의 넓이)-△AOC
150 1
∴ =p_8¤ _ - _8_8_sin(180˘-150˘)
360 2
5 1 1
=p_8¤ _ - _8_8_
12 2 2
80 80
∴= p-16(cm¤ ) { p-16} cm¤
3 3

0686 점 E에서 BC”에 내린 수선의 발 A


D
을 H라고 하자. E 60˘
45˘
EH”=h cm라고 하면
45˘ 30˘
∠BEH=45˘, ∠CEH=60˘이므로 B H C
10 cm
BH”=h tan 45˘=h(cm) h cm

CH”=h tan 60˘='3h(cm)

05. 삼각비의 활용 65
(066~103)개념RPM3-2(정답) 2014.11.24 02:58 PM 페이지66 다민 2540DPI 175LPI

Ⅳ 원의 성질

06 원과직선 0700 ∠OTP=90˘이므로 △OPT에서


x="√13¤ -5¤ ='∂144=12 12

0689 ㈎ OB” ㈏ O’M” ㈐ RHS ㈑ B’M”


0701 ∠OTP=90˘이므로 △OTP에서
x="√(8+9)¤ -8¤ ='∂225=15 15
0690 △OAM에서
A’M=
” "√6¤ -3¤ ='ß27=3'3(cm)
0702 PB”=P’A=
” 10 cm 10 cm
A’M”=B’M이
” 므로
AB”=2A’M”=2_3'3=6'3(cm) 0703 ∠PBO=90˘이므로 △OPB에서
∴ x=6'3 6'3 PO”="√10¤ +5¤ ='∂125=5'5(cm) 5'5 cm

0691 △OMB에서 0704 ∠OAP=90˘이므로 △AOP에서


MÚB”="√5¤ -3¤ ='ß16=4(cm) AP” ¤ +4¤ =(4+2)¤ , AP” ¤ =20
A’M”=B’M이
” 므로` x=4 4 AP”=2'5(cm) (∵ AP”>0)
∴ x=AP”=2'5 2'5
1 1
0692 A’M=

2
AB”= _8=4(cm)
2
0705 ㈎ BE” ㈏ CE” ㈐ AF” ㈑ 12 ㈒ 3
△OAM에서
O’A”="√4¤ +2¤ ='ß20=2'5(cm) 0706 AD”+AF”=AB”+BC”+C’A”
∴ x=2'5 2'5 =8+5+7=20
이때 AD”=AF”이므로 x=10 10
1 1
0693 B’M”=
2
AB”= _10=5(cm)
2
0707 BE”=BD”=12-8=4
△OMB에서 AF”=AD”=12이므로
O’M=
” "√6¤ -5¤ ='ß11(cm) CE”=CF”=12-9=3
∴ x='ß11 'ß11 ∴ x=BE”+CE”=4+3=7 7
다른풀이
0694 두 현의 길이가 같으므로 x=3 3 AD”+AF”=AB”+BC”+C’A이
” 므로
24=8+x+9 ∴ x=7
0695 원의 중심에서 같은 거리에 있으므로 x=9 9
0708 BE”=BD”=9-6=3
0696 원의 중심에서 같은 거리에 있으므로 두 현의 길이는 CF”=CE”=5-3=2
같다. AF”=AD”=9이므로
1 x=AF”-CF”=9-2=7 7
∴ x= _6=3 3
2 다른풀이
AD”+AF”=AB”+BC”+C’A이
” 므로
0697 원의 중심에서 같은 거리에 있으므로 두 현의 길이는 18=6+5+x ∴ x=7
같다.
∴ x=2_8=16 16 0709 CE”=CF”=6-4=2
BD”=BE”=3-2=1
0698 ∠PAO=∠PBO=90˘이므로 APBO에서 AD”=AF”=6이므로
∠x=360˘-(90˘+45˘+90˘)=135˘ 135˘ x=AD”-BD”=6-1=5 5
다른풀이
0699 ∠PAO=∠PBO=90˘이므로 APBO에서 AD”+AF”=AB”+BC”+C’A이
” 므로
∠x=360˘-(90˘+130˘+90˘)=50˘ 50˘ 12=x+3+4 ∴ x=5

66 정답과 풀이
(066~103)개념RPM3-2(정답) 2014.11.24 02:58 PM 페이지67 다민 2540DPI 175LPI

● 본문 095쪽 ~ 098쪽 ●

0710 ㈎ 5-x ㈏ 6-x ㈐ 2 0720 원 O의 반지름의 길이를 r cm라 하면


OA”=r cm, OM”=(r-5) cm

0711 BE”=BD”=x이므로 △AOM에서


AF”=AD”=7-x, CF”=CE”=9-x r¤ =7¤ +(r-5)¤ , 10r=74
AC”=AF”+CF”이므로 37 37
∴ r= (cm) cm
5 5
8=(7-x)+(9-x), 2x=8 ∴ x=4 4

0721 O’A를
” 그으면
0712 CF”=CE”=x이므로
O’A”=OD”=3+2=5(cm) O
AD”=AF”=8-x, BD”=BE”=12-x 5 cm
△OAC에서 3 cm
AB”=AD”+BD”이므로
AC”="√5¤ -3¤ ='ß16=4(cm) A C B
10=(8-x)+(12-x), 2x=10 ∴ x=5 5 D 2 cm
OC”⊥AB”이므로
AB”=2AC”=2_4=8(cm) 8 cm
0713 AF”=AD”=3이므로
BE”=BD”=9-3=6, CE”=CF”=10-3=7
0722 △CDB에서 2 cm C
4 cm
BC”=BE”+CE”이므로
DB”="√4¤ -2¤ ='ß12 A B
x=6+7=13 13 D
=2'3(cm) yy`
O
원 O의 반지름의 길이를 r cm라고 하면
0714 △ABC에서
OB”=r cm, OD”=(r-2) cm
AC”="√13¤ -12¤ ='ß25=5
△OBD에서
CE”=CF”=x이므로
r¤ =(r-2)¤ +(2'3)¤ , 4r=16
AD”=AF”=5-x, BD”=BE”=12-x
∴ r=4(cm) yy`
AB”=AD”+BD”이므로
따라서 원 O의 지름의 길이는
13=(5-x)+(12-x), 2x=4 ∴ x=2 2
2_4=8(cm) yy`
다른풀이
8 cm
△ABC에서
AC”="√13¤ -12¤ ='ß25=5 단계 채점요소 배점
DB”의 길이 구하기 30%
1 1
_12_5= _x_(13+12+5) 원 O의 반지름의 길이 구하기 60%
2 2
원 O의 지름의 길이 구하기 10%
30=15x ∴ x=2

0715 x+8=6+9 ∴ x=7 7


0723 AD”=
1 1
AB”= _16
2 2
=8(cm)
0716 4+x=7+5 ∴ x=8 8
CD”의 연장선은 이 원의 중심을 지나 C
므로 원의 중심을 O, 반지름의 길이 8 cm 4 cm
B
0717 7+x=5+8 ∴ x=6 6 를 r cm라고 하면
A
D
r cm (r-4) cm
O’A”=r cm, OD”=(r-4) cm
O
0718 8+11=(3+x)+12 ∴ x=4 4 이므로 △AOD에서
r¤ =(r-4)¤ +8¤ , 8r=80
0719 O’M”⊥AB”이므로 ∴ r=10(cm) 10 cm
1 1
A’M=
” AB”= _12=6(cm)
2 2
1 1
△OAM에서 0724 AD”=
2
AB”= _24
2
O’A”="√6¤ +3¤ ='ß45=3'5(cm) ③ =12(cm)

06. 원과 직선 67
(066~103)개념RPM3-2(정답) 2014.11.24 02:58 PM 페이지68 다민 2540DPI 175LPI

RPM 알피엠

CD”의 연장선은 이 원의 중심을 지나므 △AOM에서


로 원의 중심을 O라고 하면 r ¤
r¤ ={ } +(5'3)¤
△AOD에서 2

OD”="√20¤ -12¤ ='∂256=16(cm) 3


r¤ =75, r¤ =100 ∴ r=10 (∵ r>0) yy`
4
∴ CD”=OC”-OD”
10
=20-16=4(cm) 4 cm
단계 채점요소 배점

0725 CD”의 연장선은 이 원의 중심 C


A’M”의 길이 구하기 20%
3"3 반지름의 길이를 r로 놓고 O’A”, O’M”을 r로 나타내기 20%
을 지나므로 원의 중심을 O, 반지름의 3
A B 원 O의 반지름의 길이 구하기 60%
길이를 r라고 하면 D
r r-3
O’A”=r, OD”=r-3 O
이므로 △AOD에서 0729 점 O에서 AB”에 수직인 직선을
r¤ =(r-3)¤ +(3'3)¤ 그어 접기 전의 원과 만나는 점을 C라고 6 cm O
6r=36 ∴ r=6 ③ 하면 O’A”=OC”=AC”이므로 △OAC는
정삼각형이다. A B

1 1 마찬가지로 △OCB도 정삼각형이므로 C


0726 DB”=
2
AB”= _8
2
C
D 2 cm ∠AOB=2_60˘=120˘
=4(cm) A B
4 cm 120
CD”의 연장선은 이 접시의 중심을 지
(r-2) cm r cm ∴ μ AB=2p_6_ =4p(cm) ②
360
O
나므로 접시의 중심을 O, 반지름의
길이를 r cm라고 하면
0730 O’M”=ON”이므로 AB”=CD”=6 cm
OB”=r cm, OD”=(r-2) cm
1 1
이므로 △DOB에서 B’M”= AB”= _6=3(cm)이므로
2 2
r¤ =(r-2)¤ +4¤ , 4r=20 △MBO에서
∴ r=5(cm) 3 '3
OB”= =3÷ =2'3(cm)
따라서 원래 접시의 둘레의 길이는 cos 30˘ 2
2p_5=10p(cm) 10p cm 따라서 원 O의 넓이는
p_(2'3)¤ =12p(cm¤ ) ②

0727 원의 중심 O에서 AB”에 내린 수


선의 발을 M이라고 하면
8 cm O 0731 △OCN에서
O’A=
” 8 cm 4 cm CN”="√(5'2)¤ -5¤ ='ß25=5
1 1 A M B
O’M=
” O’A”= _8=4(cm) ∴ CD”=2CN”=2_5=10
2 2
이때 O’M”=O’N이
” 므로
△OAM에서
AB”=CD”=10 10
A’M=
” "√8¤ -4¤ ='ß48=4'3(cm)
∴ AB”=2A’M”=2_4'3=8'3(cm) ⑤
0732 원의 중심 O에서 CD”에 내린
MA
수선의 발을 N이라 하면 AB”=CD” D
0728 원의 중심 O에서 AB”에 내린 수 10"3 B
13 cm N
이므로 B 12 cm
선의 발을 M이라고 하면 M O
C
1 1 A ON”=O’M=
” 12 cm
A’M=
” AB”= _10'3=5'3 r O
2 2 △OND에서
yy` DN”="√13¤ -12¤ ='2å5=5(cm)
원 O의 반지름의 길이를 r라고 하면 따라서 CD”=2DN”=2_5=10(cm)이므로
1 r 1
O’A”=r, O’M=
” O’A”= 이므로 yy` △OCD= _10_12=60(cm¤ ) 60 cm¤
2 2 2

68 정답과 풀이
(066~103)개념RPM3-2(정답) 2014.11.24 02:58 PM 페이지69 다민 2540DPI 175LPI

● 본문 098쪽 ~ 100쪽 ●

0733 원의 중심 O에서 AB”, CD”에 내린 6 cm △ADO에서


A B
수선의 발을 각각 M, N이라고 하면 M 4'3 '3
OA”= =4'3÷ =8(cm)
5 cm cos 30˘ 2
AB”=CD”=6 cm이므로 O
O’M”=O’N” 따라서 원 O의 넓이는 p_8¤ =64p(cm¤ ) 64p cm¤
N 다른풀이
1 1 C D
B’M”= AB”= _6=3(cm) 6 cm
2 2 △ABC는 정삼각형이므로
△OBM에서 '3
AE”= _8'3=12(cm)
2
O’M=
” "√5¤ -3¤ ='ß16=4(cm)
점 O는 정삼각형 ABC의 무게중심이므로
따라서 AB”와 CD” 사이의 거리는
2 2
MÚN”=2 O’M”=2_4=8(cm) 8 cm O’A”= AE”= _12=8(cm)
3 3
따라서 원 O의 넓이는 p_8¤ =64p(cm¤ )
0734 O’M”=O’N이
” 므로 AB”=AC”
따라서 △ABC는 이등변삼각형이다.
0738 ∠OAP=90˘이고 OQ”=O’A=
” 3 cm이므로
1
∴ ∠B= (180˘-48˘)=66˘ 66˘ △OAP에서
2
AP”="√(3+4)¤ -3¤ ='ß40=2'ß10(cm) ②
0735 MBLO에서
∠B=360˘-(90˘+110˘+90˘)=70˘ 0739 원 O의 반지름의 길이를
O’M”=O’N이
” 므로 AB”=AC” r cm라고 하면 O
r cm
∴ ∠C=∠B=70˘ OB”=O’A”=r cm, r cm
9 cm
B
∴ ∠A=180˘-(70˘+70˘)=40˘ 40˘ OP”=(9+r) cm P
15 cm A
△OPA에서 ∠OAP=90˘이므로
0736 AMON에서 (9+r)¤ =r¤ +15¤
∠A=360˘-(90˘+120˘+90˘)=60˘ yy` 18r=144 ∴ r=8(cm)
이때 O’M”=O’N에
” 서 AB”=AC”이므로 △ABC는 이등변삼각 1
∴ △OPA= _15_8=60(cm¤ ) 60 cm¤
형이다. 2

1
∴ ∠B=∠C= (180˘-60˘)=60˘
2 0740 원 O의 반지름의 길이를 T
따라서 △ABC는 정삼각형이므로 yy` r cm라 하면 ∠PTO=90˘이므 r cm
BC”=AB”=2A’M”=2_6=12 yy` 30˘ r cm
로 △TPO에서 P
5 cm A O
12 OT” 1 r
sin 30˘= , =
PO” 2 5+r
단계 채점요소 배점
2r=5+r ∴ r=5(cm)
∠A의 크기 구하기 30%
따라서 OP”=10 cm, OT”=5 cm이므로
△ABC가 정삼각형임을 알기 40%
BC”의 길이 구하기 30% PT”="√10¤ -5¤ ='7å5=5'3(cm) 5'3 cm

0737 OD”=OE”=OF”이므로 0741 PT≥가 원 O의 접선이므로


4"3 cm A 30˘
∠PTO=90˘, ∠TPO=180˘-(90˘+60˘)=30˘ yy`
AB”=BC”=C’A”
D F
△POT에서 PT”=6 tan 60˘=6'3(cm) yy`
따라서 △ABC는 정삼각형이므로 O
B C
∴ (색칠한 부분의 넓이)
∠A=∠B=∠C=60˘ E
=△POT-(부채꼴 TOQ의 넓이)
1 1
AD”= AB”= _8'3=4'3(cm)이고 1 60
2 2 = _6'3_6-p_6¤ _
2 360
△ADO™△AFO(RHS 합동)이므로
=18'3-6p(cm¤ ) yy`
1 1
∠DAO= ∠A= _60˘=30˘ (18'3-6p) cm¤
2 2

06. 원과 직선 69
(066~103)개념RPM3-2(정답) 2014.11.24 02:58 PM 페이지70 다민 2540DPI 175LPI

RPM 알피엠

단계 채점요소 배점 0747 ⑴ (△ABC의 둘레의 길이)=2AD”이므로


∠PTO, ∠TPO의 크기 구하기 20% 9+8+x=2_12 ∴ x=7
PT”의 길이 구하기 40% ⑵ BF”=BD”=2이므로
색칠한 부분의 넓이 구하기 40% CF”=6-2=4
AD”=AE”=9+2=11이고 CE”=CF”=4이므로
0742 ∠PAO=∠PBO=90˘이므로 x=11-4=7
∠AOB=360˘-(90˘+45˘+90˘)=135˘ 다른풀이
∴ (색칠한 부채꼴의 중심각의 크기)=360˘-135˘ (△ABC의 둘레의 길이)=2AD”이므로
=225˘ x+6+9=2(9+2) ∴ x=7
225 ⑴7 ⑵7
∴ (색칠한 부분의 넓이)=p_8¤ _
360
=40p(cm¤ ) 40p cm¤
0748 ①, ② 원 밖의 한 점에서 그은 두 접선의 길이는 같으
0743 PA”=PB”에서 △PBA는 이등변삼각형이므로 므로
1 PA”=PB”, EB”=EC”
∠BAP= _(180˘-70˘)=55˘
2
③ △OAD와 △OCD에서 ∠OAD=∠OCD=90˘,
∠OAP=90˘이므로
OA”=OC”, OD”는 공통이므로
∠x=90˘-55˘=35˘ 35˘
△OAD≡△OCD(RHS 합동)
∴ ∠ODA=∠ODC
0744 ∠PAO=∠PBO=90˘이므로
④ OC”⊥DE”이므로
△PAO™△PBO (RHS 합동)
1
△PBO에서 ∠OPB=30˘이므로 △ODC= _CD”_OC”
2
1 1
OB”=8 sin 30˘=8_ =4(cm) △OEC= _CE”_OC”
2 2
PB”=8 cos 30˘=8_
'3
=4'3(cm) 즉, CD”=CE”인 경우에만 △ODC=△OEC이고
2
CD”+CE”이면 △ODC+△OEC
∴ APBO=2△PBO
⑤ PA”=PB”, EB”=EC”, DA”=DC”이므로
1
=2{ _4_4'3} (△PED의 둘레의 길이)=PE”+ED”+DP”
2
=16'3(cm¤ ) 16'3 cm¤ =PE”+(EC”+DC”)+DP”
=(PE”+EB”)+(DA”+DP”)
=PB”+PA”
0745 ∠PAO=∠PBO=90˘이므로 APBO에서
=2PA”
∠AOB=360˘-(90˘+60˘+90˘)=120˘
따라서 항상 옳지 않은 것은 ④이다. ④
PO”를 긋고 AB”와 PO”의 교점을 H A
라고 하면 AB”⊥PO”, AH”=BH”, 6 cm
∠AOH=60˘이므로 P O
H
△AHO에서
60˘ 60˘ 0749 AO”를 그으면
B △EAO™△DAO(RHS 합동)이므로 E 8 cm
A’H=
” 6 sin 60˘=6_
'3 C
1 O
2 ∠EAO= ∠EAD 60˘ F
2
=3'3(cm) A
1 B D
∴ AB”=2A’H”=2_3'3=6'3(cm) 6'3 cm = _60˘=30˘
2
△EAO에서
0746 ∠AEO=90˘이므로 △AOE에서 8 '3
AE”= =8÷ =8'3(cm)
AE”="√8¤ -4¤ ='ß48=4'3(cm) tan 30˘ 3
∴ (△ABC의 둘레의 길이)=2AE”=2_4'3 ∴ (△ABC의 둘레의 길이)=2AE”=2_8'3
=8'3(cm) 8'3 cm =16'3(cm) 16'3 cm

70 정답과 풀이
(066~103)개념RPM3-2(정답) 2014.11.24 02:58 PM 페이지71 다민 2540DPI 175LPI

● 본문 101쪽 ~ 103쪽 ●

0750 점 D에서 C’A”에 내린 수 C 0753 EF”=BE”=x cm라고 하면


선의 발을 H라고 하면 E CE”=(20-x) cm
9 cmH D
H’A=
” DB”=4 cm 4 cm DF”=DA”=20 cm이므로
∴ CH”=9-4=5(cm) A O B DE”=(20+x) cm
CE”=C’A=
” 9 cm, DE”=DB”=4 cm이므로 △DEC에서
CD”=9+4=13(cm) (20+x)¤ =(20-x)¤ +20¤
△CHD에서 80x=400 ∴ x=5(cm) 5 cm
H’D”="√13¤ -5¤ ='∂144=12(cm)
∴ AB”=H’D=
” 12 cm 12 cm 0754 O’A를
” 그으면 10 cm C
O’A=
” OC”=10 cm A H B
AB”⊥OH”이므로 △AOH에서
O
A’H=
” "√10¤ -4¤ ='ß84=2'ß21(cm) 4 cm
0751 점 P, C, D가 반원 O의 접점이므로
∴ AB”=2A’H”=2_2'ß21
AD”=AP”, BC”=BP”
=4'ß21(cm) ⑤
AD”+BC”=AP”+BP”=AB”=6(cm)
또한 CD”=2 OC”=2_2'2=4'2(cm)
1 0755 작은 원과 큰 원의 반지름
a cm
∴ ABCD= (AD”+BC”)_CD”
2 의 길이의 비가 1 : 2이므로 O’A”, 2a cm
O
1 O’H”를 긋고 O’A=
” 2a cm,
= _6_4'2 A B
2 H
OH”=a cm라고 하자.
∴ ABCD=12'2(cm¤ ) 12'2 cm¤ 8"3 cm
AB”⊥OH”이므로
1 1
A’H=
” AB”= _8'3
2 2
=4'3(cm)
0752 DP”=D’A=
” 8 cm, D
P △OAH에서
CP”=CB”=5 cm이므로 H C
8 cm (2a)¤ =(4'3)¤ +a¤
DC”=DP”+CP” 5 cm
B
a¤ =16 ∴ a=4(cm)(∵ a>0)
=8+5=13(cm) yy` A O
따라서 큰 원의 반지름의 길이는
점 C에서 D’A에
” 내린 수선의 발을 H라고 하면
2a=2_4=8(cm) 8 cm
H’A=
” CB”=5 cm
∴ D’H”=8-5=3(cm)
△DHC에서 0756 AB”와 작은 원의 접점을 H라 하고
O’A”와 OH”를 그으면 두 원의 넓이의 차가 O
HC”="√13¤ -3¤ ='∂160=4'ß10(cm)
64p cm¤ 이므로
즉, AB”=HC”=4'ß10 cm이므로 반원 O의 반지름의 길이는
A B
O’A” ¤ p-OH” ¤ p=64p H
2'ß10 cm이다. yy`
∴ O’A” ¤ -OH” ¤ =64
따라서 OP”를 그으면 OP”⊥DC”이고 OP”=2'ß10 cm이므로
AB”⊥OH”이므로 △OAH에서
1
△DOC= _DC”_OP”
2 ” ø πO’A” ¤ -OH” ¤ ='ß64=8(cm)
A’H=
1 ∴ AB”=2A’H”=2_8=16(cm) 16 cm
= _13_2'ß10
2
△DOC=13'ß10(cm¤ ) yy`
0757 CE”=CF”=x cm라고 하면
13'ß10 cm¤
AD”=AF”=(13-x) cm
단계 채점요소 배점 BD”=BE”=(15-x) cm
DC”의 길이 구하기 30% AB”=AD”+BD”이므로
반원 O의 반지름의 길이 구하기 40% 14=(13-x)+(15-x)
△DOC의 넓이 구하기 30% 2x=14 ∴ x=7(cm) ④

06. 원과 직선 71
(066~103)개념RPM3-2(정답) 2014.11.24 02:58 PM 페이지72 다민 2540DPI 175LPI

RPM 알피엠

0758 AD”=AF”=6 cm이므로 0763 OD”, OF”를 그으면 A r cm


r cm F
BE”=BD”=9-6=3(cm) ADOF는 정사각형이므로 원 D
4 cm 6 cm
CE”=CF”=10-6=4(cm) O의 반지름의 길이를 r cm라고 O
∴ BC”=BE”+CE”=3+4=7(cm) 7 cm 하면 B C
4 cm E 6 cm
AD”=AF”=r cm
BD”=BE”=4 cm, CF”=CE”=6 cm이므로
0759 BD”=BE”=5 cm, CE”=CF”=9 cm이므로 AB”=(4+r) cm, AC”=(6+r) cm
AD”=AF”=x cm라고 하면 △ABC에서
AB”+BC”+C’A=
” 32 cm에서 (4+r)¤ +(6+r)¤ =(4+6)¤
(x+5)+(5+9)+(9+x)=32 r¤ +10r-24=0, (r+12)(r-2)=0
2x=4 ∴ x=2(cm) 2 cm ∴ r=2(cm)(∵ r>0)
따라서 원 O의 넓이는
p_2¤ =4p(cm¤ ) ②
0760 AD”=AF”=6 cm이므로
BE”=BD”=13-6=7(cm)
0764 OF”를 그으면 3 cm A
CE”=15-7=8(cm) D
OECF는 정사각형이므로 x cm O
3 cm
∴ (△QPC의 둘레의 길이)=2CE”=2_8 F
CE”=CF”=2 cm yy` 2 cm 2 cm
=16(cm) 16 cm B
BE”=BD”=x cm라고 하면 x cm E C
2 cm
AD”=AF”=5-2=3(cm)이므로
△ABC에서
0761 AB”="√8¤ +6¤ ='∂100 A
=10(cm) (x+3)¤ =(x+2)¤ +5¤
D (6-r) cm
OE”, OF”를 그으면 OECF O 2x=20 ∴ x=10(cm)
F
는 정사각형이므로 원 O의 반
r cm ∴ BC”=10+2=12(cm) yy`
B C
{8-r}`cm E 1
지름의 길이를 r cm라고 하면 r cm
∴ △ABC= _12_5=30(cm¤ ) yy`
2
EC”=CF”=r cm
30 cm¤
BD”=BE”=(8-r) cm, AD”=AF”=(6-r) cm이므로
AB”=AD”+BD”에서 단계 채점요소 배점

10=(6-r)+(8-r), 2r=4 CE”, CF”의 길이 구하기 20%


BC”의 길이 구하기 60%
∴ r=2(cm) 2 cm
△ABC의 넓이 구하기 20%

0762 △ABC에서 A 0765 △BCD에서


1 F 4 cm CD”="√15¤ -12¤ ='ß81=9
AB”=4 sin 30˘=4_
2 O
D ABCD가 원 O에 외접하므로
=2(cm) r cm
30˘
B C AB”+CD”=AD”+BC”
E
BC”=4 cos 30˘=4_
'3 r cm AB”+9=8+12 ∴ AB”=11 ②
2
=2'3(cm)
0766 AB”+CD”=AD”+BC”이므로
OD”, OE”를 그으면 DBEO는 정사각형이므로 원 O의 반지
2x+8=10+(x+5) ∴ x=7 ④
름의 길이를 r cm라고 하면
BD”=BE”=r cm
0767 AB”+CD”=AD”+BC”이므로
AF”=AD”=(2-r) cm, CF”=CE”=(2'3-r) cm이므로
AB”+CD”=8+12=20(cm)
AC”=AF”+CF”에서 4=(2-r)+(2'3-r)
등변사다리꼴 ABCD에서 AB”=CD”이므로
2r=2'3-2
1
∴ r='3-1(cm) ('3-1) cm AB”= _20=10(cm) ②
2

72 정답과 풀이
(066~103)개념RPM3-2(정답) 2014.11.24 02:58 PM 페이지73 다민 2540DPI 175LPI

● 본문 103쪽 ~ 105쪽 ●

0768 CG”=CF”=x cm라고 하면 (4+x)¤ =(4-x)¤ +4¤


A’H=
” AE”=4 cm 16x=16 ∴ x=1
BE”=BF”=5 cm ∴ AB”=4+1=5(cm) 5 cm
D’G”=D’H=
” 3 cm
2(4+5+3+x)=40이므로
0773 △DIC에서
x=8(cm) 8 cm
IC”="√5¤ -4¤ ='9=3(cm)
AD”=x cm라고 하면 BC”=AD”=x cm이므로
0769 AB”+CD”=AD”+BC”이므로 BI”=BC”-IC”=(x-3) cm
AB”+CD”=12+20=32(cm) AB”=DC”=4 cm
3 AB”+DI”=AD”+BI”이므로
∴ AB”=32_ =12(cm) 12 cm
8 4+5=x+(x-3), 2x=12
∴ x=6(cm) ②

0770 OF”를 그으면 OFCG는 A H D


정사각형이므로
E 6 cm 0774 AE”=AF”=BF” A 4 cm 8 cm D
CF”=CG”=6 cm 12 cm O G E
1 4 cm
BE”=BF”=14-6=8(cm)이므로 =BG”= _8
2 F 8 cm
B F
C 4 cm H 8 cm
A’H=
” AE”=12-8 14 cm =4(cm) yy` G x cm
C
B 4 cm I (8-x) cm
=4(cm) 이므로 x cm
4 cm D’H”=DE”=12-4=8(cm) yy`

GI”=HI”=x cm라고 하면
DI”=(8+x) cm, CI”=(8-x) cm yy`
0771 AB”=2OE”=2_4=8(cm) yy`
△DIC에서
AB”+CD”=AD”+BC”이므로
(8+x)¤ =8¤ +(8-x)¤
AD”+BC”=8+10=18(cm) yy`
32x=64 ∴ x=2(cm) yy`
1
∴ ABCD= _(AD”+BC”)_AB” 2 cm
2
1 단계 채점요소 배점
= _18_8=72(cm¤ ) yy`
2
AE”, BG”의 길이 구하기 20%
72 cm¤
D’H의
” 길이 구하기 20%

단계 채점요소 배점 GI”=x cm로 놓고 DI”, CI”를 x로 나타내기 20%


GI”의 길이 구하기 40%
AB”의 길이 구하기 30%
AD”+BC”의 길이 구하기 40%
ABCD의 넓이 구하기 30%
0775 AE”=AF”=BF” 2 cm 8 cm
2 cm A 6 cm
D
1 E
=BG”= _4 6 cm
2
0772 OF”, OG”를 그으면 A H 2 cm D
4 cm F O
H
E =2(cm) 2 cm
1 1 B C
CG”=CF”= CD”= _4 GI
2 2 4 cm O
G
4 cm DH”=DE”=8-2=6(cm) 2 cm
=2(cm) 2 cm
∴ (△DIC의 둘레의 길이)
I F
B C
DH”=DG”=2 cm이고 6 cm =DI”+IC”+DC”
2 cm
BE”=BF”=6-2=4(cm) =(DH”+HI”)+IC”+DC”
AE”=AH”=x cm라 하고 =DE”+(GI”+IC”)+DC”
점 A에서 BC”에 내린 수선의 발을 I라고 하자. =DE”+GC”+DC”
IF”=AH”=x cm이므로 BI”=(4-x) cm =6+6+4
△ABI에서 =16(cm) ③

06. 원과 직선 73
(066~103)개념RPM3-2(정답) 2014.11.24 02:58 PM 페이지74 다민 2540DPI 175LPI

RPM 알피엠

0776 원 O의 반지름의 길이가


A 25 cm D
0780 AF”=AD”=x cm라고 하면
9 cm이므로 원 O'의 반지름의 (9-r) cm
BE”=BD”=(9-x) cm
길이를 r cm라고 하면 O
18 cm 9 cm r cm CE”=CF”=(5-x) cm
O’O'”=(9+r) cm H O' BC”=BE”+CE”이므로
OH”=OE”-HE” B E F C 8=(9-x)+(5-x)
(16-r) cm
=OE”-O’'F” 2x=6 ∴ x=3(cm) 3 cm
=(9-r) cm
O’'H”=FE”=BC”-(9+r) 0781 CE”=CD”=18 cm이므로
=25-(9+r)=(16-r) cm AE”=30-18=12(cm)
△OHO'에서 △AOE에서
(9+r)¤ =(9-r)¤ +(16-r)¤ AO”="√9¤ +12¤ ='∂225=15(cm)
r¤ -68r+256=0, (r-4)(r-64)=0 ∴ AG”=AO”-GO”=15-9=6(cm) 6 cm
∴ r=4(cm)(∵ 0<r<9) 4 cm

0782 BD”=BF”=9 cm
CD”=CE”=5 cm
0777 점 O에서 DC”에 내린 A x cm D
수선의 발을 H라 하고 정사각 AE”=AF”=x cm라고 하면
(x+3) cm
형 ABCD의 한 변의 길이를 (x-3) cm
2(x+9+5)=36 ∴ x=4 4 cm

x cm라고 하면 O E
DO”=(x+3) cm
H 0783 ∠PAO=∠PBO=90˘이므로
B F C
OH”=(x-3) cm 3 cm (x-3) cm ∠x=360˘-(90˘+50˘+90˘)=130˘

D’H”=(x-3) cm 또한 OA”=OB”이므로 △OAB는 이등변삼각형이다.

△DOH에서 1
∴ ∠y= (180˘-130˘)=25˘
2
(x+3)¤ =(x-3)¤ +(x-3)¤
∴ ∠x+∠y=130˘+25˘=155˘ 155˘
x¤ -18x+9=0
∴ x=9—'ß72=9—6'2(cm)
0784 AB”=CD”이므로 OE”=OF”=2 cm
그런데 AD”>3이므로
△OAE에서 AE”="√3¤ -2¤ ='5(cm)이므로
x=9+6'2(cm) (9+6'2) cm
AB”=2AE”=2'5(cm)
1
∴ △OAB= _2'5_2=2'5(cm¤ ) ②
2
0778 원 Q의 반지름의 길이가 r cm
8 cm이므로 원 P의 반지름의 길 8 cm Q

이를 r cm라고 하면 P H 0785 (△ABC의 둘레의 길이)=2AE”이므로

16 cm O 3+4+5=2AE” ∴ AE”=6(cm)
PQ”=(8+r) cm
∴ CF”=CE”=6-4=2(cm) 2 cm
OH”=r cm, QH”=(8-r) cm
PO”=(16-r) cm
PQ” ¤ -QH” ¤ =PO” ¤ -OH” ¤ 이므로 0786 원의 중심 O에서 AB”, CD”에 내린 12 cm
A B
수선의 발을 각각 M, N이라고 하면 M
(8+r)¤ -(8-r)¤ =(16-r)¤ -r¤
AB”=CD”=12 cm이므로
64r=256 ∴ r=4(cm) 4 cm O 10 cm
O’M”=O’N”
N
1 1 C D
N’D”= CD”= _12=6(cm) 12 cm
0779 AB”+CD”=AD”+BC”이므로 2 2
(x+3)+(x+2)=x+(2x-1) △OND에서
∴ x=6 O’N”="√10¤ -6¤ ='ß64=8(cm)
따라서 ABCD의 둘레의 길이는 따라서 두 현 AB와 CD 사이의 거리는
2(AB”+CD”)=2(9+8)=34 ③ MÚN”=2O’N”=2_8=16(cm) ③

74 정답과 풀이
(066~103)개념RPM3-2(정답) 2014.11.24 02:58 PM 페이지75 다민 2540DPI 175LPI

● 본문 105쪽 ~ 107쪽 ●

0787 O’M”=O’N이
” 므로 AB”=AC” 꼭짓점 A에서 BC”에 내린 수선의 발 A 6 cm D
따라서 △ABC는 이등변삼각형이므로 을 H라고 하면
8 cm
∠B=∠C=64˘ 1 O
BH”= (10-6)=2(cm)
2
∴ ∠A=180˘-(64˘+64˘)=52˘ B C
△ABH에서 H
AMON에서 10 cm
∠MON=360˘-(90˘+52˘+90˘) A’H=
” "√8¤ -2¤ ='ß60=2'ß15(cm)

=128˘ 128˘ 따라서 원 O의 지름의 길이가 2'ß15 cm이므로 원 O의 둘레의


길이는 2'ß15p cm이다. ③

0788 OC”⊥AB”이므로 AH”=BH”=2'6 cm


0793 OD”=OE”=OF”이므로 3 cm A
OA”=x cm라고 하면 OH”=(x-2) cm 30˘
AB”=BC”=C’A”
△OAH에서 D F
따라서 △ABC는 정삼각형이므로
x¤ =(2'6 )¤ +(x-2)¤ O
∠A=∠B=∠C=60˘, B C
4x=28 ∴ x=7(cm) 7 cm E
1 1
AD”= AB”= _6=3(cm)
2 2
0789 원의 중심 O에서 AB”, BC”,
E
D 이고 △ADO™△AFO(RHS 합동)이므로
A
CD”에 내린 수선의 발을 각각 F, H 1 1
3 cm 8 cm ∠DAO= ∠A= _60˘=30˘
G, H라고 하면 2 2
F
O
FBGO는 한 변의 길이가 3 cm △ADO에서
B G C 3
3 cm인 정사각형이므로 O’A”= =3÷
'3
=2'3(cm)
3 cm 9 cm cos 30˘ 2
CH”=CG”=9-3=6(cm)
따라서 원 O의 넓이는
∴ DE”=DH”=8-6=2(cm) 2 cm
p_(2'3)¤ =12p(cm¤ ) 12p cm¤
다른풀이
0790 ∠ADO=90˘이므로 △AOD에서
OD”=OE”=OF”에서 AB”=BC”=C’A이
” 므로 △ABC는 정삼
AD”="√11¤ -5¤ ='ß96=4'6(cm)
각형이다.
∴ (△ACB의 둘레의 길이)=AB”+BC”+AC”
AO”를 그으면 AE”는 정삼각형 ABC의 높이이므로
=AD”+AF”
'3
=2AD” AE”= _6=3'3(cm)
2
=2_4'6 점 O는 △ABC의 무게중심이므로
=8'6(cm) ⑤ 2 2
AO”= AE”= _3'3=2'3(cm)
3 3
따라서 원 O의 넓이는
0791 PO”를 긋고 AB”와 PO”의 교 A
p_(2'3)¤ =12p(cm¤ )
점을 H라고 하면 ∠AOB=120˘이 8
H
므로 P O
60˘ 0794 점 O에서 AB”에 수직인 직선을 그 A 3 cm
∠AOH=60˘
B 어 접기 전의 원 O와 만나는 점을 C라고 하면 C
△AHO에서 r cm M
O’M”=MÚC”
'3 O
A’H=
” 8 sin 60˘=8_ =4'3 원 O의 반지름의 길이를 r cm라고 하면 -r cm B
2 2
∴ AB”=2A’H”=2_4'3=8'3 8'3 r
O’A”=r cm, O’M=
” cm
2
1 1
A’M=
” AB”= _6=3(cm)이므로
0792 ABCD가 원 O에 외접하므로 2 2
AB”+CD”=AD”+BC”=6+10=16(cm) △AOM에서
그런데 AB”=CD”이므로 r ¤
r¤ ={ } +3¤ , r¤ =12
2
1
AB”= _16=8(cm) ∴ r=2'3(cm) (∵ r>0) 2'3 cm
2

06. 원과 직선 75
(066~103)개념RPM3-2(정답) 2014.11.24 02:58 PM 페이지76 다민 2540DPI 175LPI

RPM 알피엠

0795 O’A,” OP”를 그으면 0798 AB”=2OF”=2_5=10(cm)


r cm
OP”⊥AB”이므로 2r cm AB”+CD”=AD”+BC”이므로
O
1 AD”+BC”=10+12=22(cm)
AP”=BP”= AB”
2 A B
P 1
∴ ABCD= _(AD”+BC”)_AB”
1 6'3 cm 2
= _6'3=3'3(cm)
2
1
OP”=r cm라고 하면 O’A”=2r cm = _22_10=110(cm¤ ) 110 cm¤
2
이므로 △OAP에서
(2r)¤ =(3'3 )¤ +r¤ , r¤ =9
∴ r=3(cm) (∵ r>0) 0799 ⑴ PT”=RT”, QT”=RT”이므로
따라서 큰 원의 반지름의 길이는 PT”=RT”=QT”
2r=2_3=6(cm) 6 cm 1 1
∴ RT”= PQ”= _12=6(cm)
2 2
0796 O’M”=O’N이
” 므로 △ABC는 A 6'3 cm ⑵ PT”=RT”=QT”이므로 △PTR, △RTQ는 모두 이등변
AB”=AC”인 이등변삼각형이고 30˘
삼각형이다.
M N
∠A=60˘이므로 △ABC는 정삼각형
∠PRT=∠RPT=43˘이므로
O
이다.
B 12'3 cm C ∠TRQ=∠TQR=∠x라고 하면
AO”를 그으면 △AON에서
△RPQ에서 2_43˘+2∠x=180˘
1 1 1 ∴ ∠x=47˘ ⑴ 6 cm ⑵ 47˘
A’N”= AC”= BC”= _12'3=6'3(cm)
2 2 2
∠OAN=30˘이므로
6'3 '3
AO”= =6'3÷ =12(cm) 12 cm
cos 30˘ 2 0800 점 O에서 AB”에 내린 수선
의 발을 H라고 하면
0797 ①, ② △PAO™△PBO(RHS 합동)이므로 O
1 R cm r cm
1 1 A’H=
” BH”= AB”
∠APO=∠BPO= ∠P= _60˘=30˘ 2 A
2 2 H B
1
∠PAO=∠PBO=90˘이므로 △APO에서 = _12=6(cm) 12 cm
2
2 1 큰 원의 반지름의 길이를 R cm, 작은 원의 반지름의 길이를
PO”= =2÷ =4(cm)
sin 30˘ 2
r cm라고 하면 △OAH에서
2 '3
P’A=
” =2÷ =2'3(cm) R¤ -r¤ =6¤ =36
tan 30˘ 3
∴ (색칠한 부분의 넓이)=pR¤ -pr¤
∴ PO”=4 cm, P’A=
” 2'3 cm
=p(R¤ -r¤ )
∴ PB”=P’A”=2'3 cm
=36p(cm¤ ) 36p cm¤
③ PO”⊥AB”이고 ∠AOB=180˘-60˘=120˘이므로
∠AOH=60˘
△AHO에서
1
AH”=2 sin 60˘=2_
'3
='3(cm) 0801 BR”=BQ”=
2
AB” A P E
D
2 F
∴ AB”=2A’H=
” 2'3(cm) 1 6 cm
= _6=3(cm) Q O
2
④ APBO=2△APO
RC”=8-3=5(cm) C
1 B R
=2_{ _2'3_2} ∴ (△DEC의 둘레의 길이) 8 cm
2
=4'3(cm¤ ) =DE”+EC”+CD”

⑤ △OAB에서 ∠AOB=120˘이므로 =DE”+EF”+FC”+CD”


=DE”+EP”+RC”+CD”
1
∠OAB= (180˘-120˘)=30˘
2 =PD”+RC”+CD”
따라서 옳지 않은 것은 ③이다. ③ =5+5+6=16(cm) ③

76 정답과 풀이
(066~103)개념RPM3-2(정답) 2014.11.24 02:58 PM 페이지77 다민 2540DPI 175LPI

● 본문 108쪽 ~ 109쪽 ●

0802 OF”를 긋고 내접원 O의 반 A ∴( ABCD의 둘레의 길이)


지름의 길이를 r cm라고 하면 F 8 cm
=AB”+BC”+CD”+D’A”
7 cm D r cm
1 =12+4+13+9
r(7+8+9)=12'5 O
2 =38(cm) yy`
B C
∴ r='5(cm) E 38 cm
9 cm
CF”=CE”=a cm라고 하면
단계 채점요소 배점
BD”=BE”=(9-a) cm,
DC”의 길이 구하기 30%
AD”=AF”=(8-a) cm
AB”의 길이 구하기 40%
이므로
ABCD의 둘레의 길이 구하기 30%
AB”=(9-a)+(8-a)=7
∴ a=5(cm)
△OCF에서 0805 PO”를 그으면 A
OC”="√5¤ +('5)¤ ='ß30(cm) 6 cm
∠PAO=90˘이고 ∠APO=30˘
P 30˘ O
'ß30 cm 이므로 △APO에서
120˘
'3
O’A=
” 6 tan 30˘=6_ B
0803 C’M”의 연장선은 이 원의 중 C 3
심을 지나므로 원의 중심을 O, 반지 2 cm
B
=2'3(cm) yy`
A
M 4 cm ∠PAO=∠PBO=90˘이므로
름의 길이를 r cm라고 하면 (r-2) cm
r cm
O’A”=r cm, O’M=
” (r-2) cm APBO에서
O
yy` ∠AOB=360˘-(90˘+60˘+90˘)
=120˘ yy`
이므로 △AOM에서
120
r¤ =(r-2)¤ +4¤ , 4r=20 ∴ (색칠한 부분의 넓이)=p_(2'3)¤ _
360
∴ r=5(cm) yy`
=4p(cm¤ ) yy`
따라서 이 원의 넓이는 4p cm¤
p_5¤ =25p(cm¤ ) yy`
단계 채점요소 배점
25p cm¤
O’A의
” 길이 구하기 40%
단계 채점요소 배점 ∠AOB의 크기 구하기 30%
원의 반지름의 길이를 r cm로 놓고 OA”, O’M의
” 길이를 r로 색칠한 부분의 넓이 구하기 30%
40%
나타내기
원의 반지름의 길이 구하기 40%
원의 넓이 구하기 20%
0806 OD”, OF”를 그으면 A
F
ADOF는 정사각형이므로 원 D
r cm
0804 점 C에서 D’A”에 내린 수 D O의 반지름의 길이를 r cm라고 O
선의 발을 H라고 하면 P 하면 B C
9 cm H C 6 cm E 9 cm
DP”=D’A=
” 9 cm AD”=AF”=r cm
4 cm
CP”=CB”=4 cm A B BD”=BE”=6 cm, CF”=CE”=9 cm이므로
O
∴ DC”=DP”+CP” AB”=(r+6) cm, AC”=(r+9) cm yy`
=9+4 △ABC에서
=13(cm) yy` 15¤ =(r+6)¤ +(r+9)¤
H’A=
” CB”=4 cm이므로 r¤ +15r-54=0, (r-3)(r+18)=0
D’H”=9-4=5(cm) ∴ r=3(cm)(∵ r>0) yy`
△DHC에서 따라서 원 O의 넓이는
CH”="√13¤ -5¤ ='∂144=12(cm) p_3¤ =9p(cm¤ ) yy`
∴ AB”=CH”=12 cm yy` 9p cm¤

06. 원과 직선 77
(066~103)개념RPM3-2(정답) 2014.11.24 02:58 PM 페이지78 다민 2540DPI 175LPI

RPM 알피엠

단계 채점요소 배점 0810 원 O의 반지름의 길이 A 10 cm D


원의 반지름의 길이를 r cm로 놓고 AB”, AC”를 r로 나타내기 40% 가 4 cm이므로 원 O'의 반지 (4-r) cm
원의 반지름의 길이 구하기 40% 름의 길이를 r cm라고 하면 8 cm O 4 cm r cm
O'
원의 넓이 구하기 20% O’O'”=(4+r) cm, H
OH”=OE”-H’E” B E F C
(6-r) cm
=OE”-O’'F”
0807 점 E에서 DC”에 내 A D
=(4-r) cm
린 수선의 발을 H라고 하면 10 cm
(10-x) cm O’'H”=FE”=BC”-(4+r)
EH”=BC”=10 cm x cm F
E
=10-(4+r)=(6-r) cm
EF”=x cm라고 하면 O
H
B C △OHO'에서
EB”=HC”=x cm이므로 10 cm
(4+r)¤ =(4-r)¤ +(6-r)¤
D’H”=(10-x) cm
r¤ -28r+36=0
DF”=DC”=10 cm이므로
∴ r=14—4'ß10
DE”=(10+x) cm
그런데 0<r<4이므로
△DEH에서
r=14-4'ß10(cm) (14-4'ß10 ) cm
(10+x)¤ =10¤ +(10-x)¤
5
40x=100 ∴ x= (cm)
2
∴ DE”=DF”+EF”
5 25 25
=10+ = (cm) cm
2 2 2

0808 CE”를 그으면 BF”⊥CE”이고 (10-x) cm


A F x cm D
CE”=8 cm
x cm
△BCE에서 E
8 cm 8 cm
BE”="√10¤ -8¤
='3å6=6(cm) B C
10 cm
DF”=EF”=x cm라고 하면
AF”=(10-x) cm
△ABF에서
(6+x)¤ =(10-x)¤ +8¤
32x=128 ∴ x=4(cm) 4 cm

0809 AP”=AQ”=AR” T
A O¢ F J
=AS”=AT” 3 cm
S E
AP”=x cm라고 하면 R O£ I 7 cm
20 cm
BP”=(20-x) cm O™
P O¡ Q H D
CQ”=CG”=15-(20-x) 11 cm
G
=x-5(cm) B 15 cm C

CH”=CQ”이므로
DR”=DH”=11-(x-5)=16-x(cm)
DI”=DR”이므로 ES”=EI”=7-(16-x)=x-9(cm)
EJ”=ES”이므로
FT”=FJ”=3-(x-9)=12-x(cm)
∴ AF”=x+(12-x)=12(cm) 12 cm

78 정답과 풀이
(066~103)개념RPM3-2(정답) 2014.11.24 02:58 PM 페이지79 다민 2540DPI 175LPI

● 본문 109쪽 ~ 113쪽 ●

Ⅳ 원의 성질

07 원주각 0823
∴ x=4
∠APB=∠CQD이므로 μ AB=μ CD
4

1 1
0811 ∠x=
2
∠AOB= _110˘=55˘
2
55˘
0824 ∠BAD=90˘이므로 △ABD에서
∠ADB=180˘-(90˘+60˘)=30˘
∠ADB=∠DBC이므로
0812 ∠x=2∠APB=2_20˘=40˘ 40˘
μ AB=μ CD ∴ x=7 7

1 1
0813 ∠x= ∠AOB= _60˘=30˘ 30˘
2 2 0825 ∠APB : ∠CQD=50˘ : 20˘=5 : 2이므로
μ AB : μ CD=5 : 2, x : 4=5 : 2
0814 ∠x=2∠APB=2_135˘=270˘ 270˘ 2x=20 ∴ x=10 10

0815 ∠x=∠APB=60˘ 60˘ 0826 μ AB : μ BC=9 : 3=3 : 1이므로


∠APB : ∠BPC=3 : 1, x˘ : 25˘=3 : 1
0816 ∠x=∠PBQ=25˘ 25˘ ∴ x=75 75

0817 반원에 대한 원주각의 크기는 90˘이므로 0827 ∠x=∠BAC=35˘ 35˘


∠APB=90˘
∴ ∠x=180˘-(90˘+35˘)=55˘ 55˘
0828 ∠x=∠ABD=180˘-(45˘+70˘)=65˘ 65˘

0818 ∠ABQ=90˘, ∠AQB=∠APB=70˘이므로


0829 ∠ACD=∠ABD=30˘이므로
∠x=180˘-(90˘+70˘)=20˘ 20˘
∠x=180˘-(80˘+30˘)=70˘ 70˘

0819 QB”를 그으면 Q


P x
R 0830 ∠ACB=∠ADB=40˘, ∠ABD=∠ACD=55˘
∠AQB=∠APB=25˘
이므로 △ABC에서
∠BQC=∠BRC=35˘ O 35˘
25˘ ∠x=180˘-(55˘+30˘+40˘)=55˘ 55˘
∴ ∠x=∠AQB+∠BQC A C
=25˘+35˘ B
=60˘ 60˘ 0831 ∠A+∠C=180˘이므로
∠x+85˘=180˘ ∴ ∠x=95˘ 95˘

0820 OB”를 그으면 P Q


∠AOB=2∠APB 20˘ x 0832 ∠x=∠A=80˘ 80˘
O
=2_20˘=40˘
따라서 ∠BOC=100˘-40˘=60˘이므로 A 100˘ C 0833 ㄱ. ∠A+∠C=180˘이므로 ABCD는 원에 내접
1 1 B 한다.
∠x= ∠BOC= _60˘=30˘
2 2
ㄴ. ∠B=180˘-(45˘+60˘)=75˘이므로 ∠B+∠D+180˘
30˘
따라서 ABCD는 원에 내접하지 않는다.
ㄷ. AD”∥BC”이므로 ∠A+∠B=180˘
0821 μ AB=μ CD이므로 ∠CQD=∠APB=15˘ 이때 ∠B=∠C이므로 ∠A+∠C=180˘
∴ x=15 15 따라서 ABCD는 원에 내접한다.
ㄹ. ∠CDE=∠B이므로 ABCD는 원에 내접한다.
0822 μ AB=μ CD이므로 ∠ACB=∠DBC=40˘ 이상에서 ABCD가 원에 내접하는 것은 ㄱ, ㄷ, ㄹ이다.
∴ x=40 40 ㄱ, ㄷ, ㄹ

07. 원주각 79
(066~103)개념RPM3-2(정답) 2014.11.24 02:58 PM 페이지80 다민 2540DPI 175LPI

RPM 알피엠

0834 ∠B+∠D=180˘이므로 0847 ∠x=∠BAT=65˘


∠x+105˘=180˘ ∴ ∠x=75˘ 75˘ ∠ABT=∠DCT=55˘이므로 △ABT에서
∠y=180˘-(65˘+55˘)=60˘
0835 ∠B=180˘-(50˘+45˘)=85˘이므로 ∠x=65˘, ∠y=60˘
∠x=∠B=85˘ 85˘
0848 `® BAD에 대한 원주각의 크기가 105˘이므로 중심각의
크기는
0836 ∠BAD=180˘-50˘=130˘이므로
2_105˘=210˘
∠x=∠BAD=130˘ 130˘
∠y=360˘-210˘=150˘이고
1 1
0837 ∠B=180˘-(90˘+25˘)=65˘이고 ∠x=
2
∠y= _150˘=75˘
2
∠B+∠D=180˘이므로
∴ ∠x+∠y=75˘+150˘=225˘ ③
65˘+∠x=180˘ ∴ ∠x=115˘ 115˘
0849 ⑴ OB”를 그으면 A
0838 ∠PQC=∠BAP=102˘이고 ∠BOC=2∠A 70˘
PQCD는 원에 내접하므로 =2_70˘=140˘ O
x C
∠PQC+∠x=180˘, 102˘+∠x=180˘ OB”=OC”이므로 B
140˘
∴ ∠x=78˘ 78˘ 1
∠x= (180˘-140˘)=20˘
2
⑵ OB”를 그으면
0839 ∠x=∠ACB=40˘ 40˘
P
Q
∠AOB=2∠APB 35˘
=2_35˘=70˘ O 20˘
0840 ∠ACB=∠ABT=50˘이므로
∠x=180˘-(50˘+60˘)=70˘ 70˘ ∠BOC=2∠BQC x
A C
=2_20˘=40˘ B
∴ ∠x=70˘+40˘
0841 ∠CAB=∠CBT'=55˘, ∠ABC=90˘이므로
=110˘
∠x=180˘-(55˘+90˘)=35˘ 35˘
⑴ 20˘ ⑵ 110˘

0842 ∠CAB=∠CBT'=70˘이므로
0850 ∠BOD=140˘이므로
∠x=2∠CAB=2_70˘=140˘ 140˘
1
다른풀이 ∠BAD= _140˘=70˘
2
1 1 △APD에서
∠CAB= ∠COB= ∠x이므로
2 2
70˘=38˘+∠ADC ∴ ∠ADC=32˘ 32˘
1
∠x=70˘ ∴ ∠x=140˘
2
0851 오른쪽 그림과 같이 점 D를 잡으
A
B
면 ® ADC에 대한 중심각의 크기는 62˘
0843 ∠x=∠BCA=180˘-(80˘+30˘)=70˘ 70˘ 360˘-108˘=252˘이므로 yy` O
108˘ x C

252˘
1
∠ABC= _252˘=126˘ yy`
0844 ∠ABC=90˘이므로 2
D
∠x=∠BAC=180˘-(35˘+90˘)=55˘ 55˘ AOCB에서
∠x=360˘-(62˘+108˘+126˘)=64˘ yy`

0845 ∠x=∠BTQ=∠DTP=∠y=55˘ 64˘


∠x=55˘, ∠y=55˘
단계 채점요소 배점

® ADC에 대한 중심각의 크기 구하기 30%


0846 ∠ABT=∠x=∠y=45˘ ∠ABC의 크기 구하기 30%
∠x=45˘, ∠y=45˘ ∠x의 크기 구하기 40%

80 정답과 풀이
(066~103)개념RPM3-2(정답) 2014.11.24 02:58 PM 페이지81 다민 2540DPI 175LPI

● 본문 113쪽 ~ 115쪽 ●

0852 O’A,” OB”를 그으면 A 0857 ∠x=∠DAC=20˘이고


∠OAP=∠OBP=90˘이므로 △PBC에서
AOBP에서 C x O 130˘ 50˘ P 64˘=20˘+∠y ∴ ∠y=44˘
∠AOB ∴ ∠y-∠x=44˘-20˘=24˘ ①
B
=360˘-(90˘+90˘+50˘)
=130˘ 0858 ⑴ ∠DBC=∠DAC=50˘
1 1 ∠BAC=∠BDC=35˘
∴ ∠x= ∠AOB= _130˘=65˘ ③
2 2
따라서 △ABC에서
∠x=180˘-(35˘+50˘+70˘)
0853 O’A,” OB”를 그으면
A =25˘
∠AOB=2∠ACB
P 96˘ O
⑵ ∠BDC=∠BAC=62˘이고
=2_48˘=96˘
48˘ C ∠ACB=∠ADB=36˘
APBO에서 B
따라서 △DBC에서
∠PAO=∠PBO=90˘이므로
∠x=180˘-(62˘+36˘+25˘)
∠APB=360˘-(90˘+96˘+90˘)
=57˘
=84˘ 84˘
⑴ 25˘ ⑵ 57˘

0854 O’A,” OB”를 긋고 오른쪽 그 A


림과 같이 점 D를 잡으면 0859 μ AB에 대한 원주각의 크기가 같으므로
∠OAP=∠OBP=90˘이므로 D
O 128˘ C 52˘ P ∠ACB=∠ADB=20˘ yy`

AOBP에서 △DPB에서 ∠DBC=20˘+25˘=45˘ yy`


B ∴ ∠x=45˘+20˘=65˘ yy`
∠AOB=360˘-(90˘+52˘+90˘)
=128˘ 65˘
이때 ∠ACB는 ® ADB에 대한 원주각이므로
단계 채점요소 배점
1
∠ACB= (360˘-128˘)=116˘ 116˘ ∠ACB의 크기 구하기 40%
2
∠DBC의 크기 구하기 30%
∠x의 크기 구하기 30%
0855 O’A,” OB”를 그으면 A
∠AOB=360˘-2_112˘
112˘
=136˘ P x Q O 0860 DB”를 그으면 D
36˘
∠PAO=∠PBO=90˘이므로 ∠ADB=90˘이고
B ∠CDB=∠CAB=36˘이므로
APBO에서 A 36˘ O B
∠x=360˘-(90˘+136˘+90˘)=44˘ ② ∠ADC=90˘-36˘=54˘
참고 54˘ C
오른쪽 그림에서 P’A”, PB”가 원 O A
의 접선일 때
P Q a
O
b
0861 ⑴ AD”를 그으면 D
① ∠b=2∠AQB E
∠ADC=90˘이고 48˘ 48˘ x
C
② ∠a=360˘-∠b ∠ADB=∠AEB=48˘이므로
O
B
=360˘-2∠AQB A
∠x=90˘-48˘=42˘
B
0856 QB”를 그으면
P
Q
⑵ AC”를 그으면 D C
∠AQB=∠APB=35˘ 35˘ x R x
∠ACB=90˘이고
∠BQC=∠BRC=22˘ 22˘ ∠ACD=∠ABD=32˘ A
32˘
B
∴ ∠x=35˘+22˘=57˘ O
A C ∴ ∠x=32˘+90˘=122˘

B ⑴ 42˘ ⑵ 122˘

07. 원주각 81
(066~103)개념RPM3-2(정답) 2014.11.24 02:58 PM 페이지82 다민 2540DPI 175LPI

RPM 알피엠

0862 AD”를 그으면 P 0867 반원에 대한 원주각의 크기는 90˘ C


x 12
∠ADB=90˘이고 이므로
C D O x B
1 1 ∠ACB=90˘ A D
∠CAD= ∠COD= _64˘ 32˘ 20
2 2 64˘ ∴ ∠ABC=∠ACD=x yy`
=32˘ A B
O △ABC에서
이므로 △PAD에서 AC”="√20¤ -12¤ ='∂256=16이므로
∠CPD=180˘-(90˘+32˘) AC” 16 4
sin x=sin B= = =
=58˘ ③ AB” 20 5
BC” 12 3
cos x=cos B= = = yy`
AB” 20 5
1 1 4 3 12
0863 ∠ACO=
2
∠AOD= _58˘=29˘
2 ∴ sin x_cos x=
5
_ =
5 25
yy`

∠ACB=90˘이고 CE”는 ∠ACB의 이등분선이므로 12


∠ACE=∠BCE=45˘ 25

∴ ∠x=45˘-29˘=16˘ 16˘ 단계 채점요소 배점


∠ABC=∠ACD=x임을 알기 40%
sin x, cos x의 값 구하기 50%
0864 원의 중심 O를 지나는 선분 A'B A sin x_cos x의 값 구하기 10%
를 그으면
A'
∠A'CB=90˘이고 ∠A=∠A'이므로 O

4'3 B
4'3 cm
C 0868 ⑴ μ AB=μ BC이므로 ∠ADB=∠BDC=40˘이고
tan A=tan A'= =2'3
A’'C” ∠BAC=∠BDC=40˘이므로 △ABD에서
2'3 A’'C”=4'3 ∠x=180˘-(40˘+40˘+45˘)=55˘
∴ A’'C”=2(cm) ⑵ AC”를 그으면 μ BD=μ CD이므로 C D
∴ A’'B”=øπ(4'3)¤ +2¤ ='5å2=2'1å3(cm) ∠CAD=∠DAB=32˘ 32˘
32˘ x
따라서 원 O의 지름의 길이는 2'1å3 cm이다. 2'1å3 cm 이때 AB”가 원 O의 지름이므로 A
O
B

∠ACB=90˘
따라서 △ABC에서
0865 ∠ACB=90˘이므로 ∠x=180˘-(90˘+32˘+32˘)=26˘
BC” 1 ⑶ μ AD=2μ BC이므로
sin 30˘= =
8 2
∠ABD=2∠BAC=2_20˘=40˘
2BC”=8 ∴ BC”=4(cm)
∴ ∠x=∠APB=180˘-(20˘+40˘)
AC” '3
cos 30˘= = =120˘
8 2
2AC”=8'3 ∴ AC”=4'3(cm) 1
⑷ ∠APB= _240˘=120˘
2
따라서 △ABC의 둘레의 길이는
1 1
8+4+4'3=12+4'3(cm) (12+4'3) cm μ PB= μ PA이므로 ∠PAB= ∠PBA, 즉
2 2
∠PBA=2∠PAB=2∠x
△PAB에서 120˘+∠x+2∠x=180˘이므로
0866 BO”의 연장선이 원 O와 만나는 A
3∠x=60˘ ∴ ∠x=20˘
점을 A'이라고 하면 60˘ A'
⑴ 55˘ ⑵ 26˘ ⑶ 120˘ ⑷ 20˘
∠BA'C=∠BAC=60˘ 60˘
O
∠BCA'=90˘이므로 B C
2'3 '3
2'3 cm
0869 △ACP에서 ∠CAP=70˘-25˘=45˘
sin 60˘= =
A’'B” 2 원의 둘레의 길이를 l cm라고 하면
'3 A’'B”=4'3 45˘ : 180˘=6p : l
∴ A’'B”=4(cm) 4 cm ∴ l=24p(cm) 24p cm

82 정답과 풀이
(066~103)개념RPM3-2(정답) 2014.11.24 02:58 PM 페이지83 다민 2540DPI 175LPI

● 본문 115쪽 ~ 118쪽 ●

0870 μAB : μ CD=3 : 2이므로 0874 BC”를 그으면 μ BD의 길이는 원주 A


∠ADB : ∠CBD=3 : 2 yy` 1 D
의 이므로
∠ADB=∠x라고 하면 6 30˘ 40˘
P
2 1 C B
∠CBD= ∠x ∠BCD=180˘_ =30˘
3 6

2 이때 μ AC : μ BD=4 : 3이므로
△DBP에서 ∠x= ∠x+25˘
3 ∠ABC : ∠BCD=4 : 3 ∴ ∠ABC=40˘
1 따라서 △PCB에서
∠x=25˘ ∴ ∠x=75˘ yy`
3 ∠APC=30˘+40˘=70˘ 70˘
75˘

단계 채점요소 배점 0875 ② ∠BAC=180˘-(40˘+60˘+40˘)=40˘


∠ADB : ∠CBD=3 : 2임을 알기 40% ③ ∴ ∠BAC=∠BDC=40˘
∠ADB의 크기 구하기 60% ③ ∠ABD=∠ACD=55˘
④ ∠BAC=∠BDC=90˘
⑤ ∠BDC=110˘-80˘=30˘

0871 μ AB : μ BC : μ CA=2 : 3 : 4이므로 ∴ ∠BAC=∠BDC=30˘

∠C : ∠A : ∠B=2 : 3 : 4 따라서 네 점 A, B, C, D가 한 원 위에 있지 않은 것은 ①

그런데 ∠A+∠B+∠C=180˘이므로 이다. ①

3
∠A=180˘_ =60˘
2+3+4 0876 ⑴ 네 점 A, B, C, D가 한 원 위에 있으므로
4 ∠ACB=∠ADB=32˘
∠B=180˘_ =80˘
2+3+4
따라서 △PBC에서
2
∠C=180˘_ =40˘ ∠DPC=32˘+54˘=86˘
2+3+4
∠A=60˘, ∠B=80˘, ∠C=40˘ ⑵ △ABD에서
∠ADB=180˘-(70˘+50˘)=60˘
따라서 네 점 A, B, C, D가 한 원 위에 있으므로
1 ∠x=∠ADB=60˘
0872 μ AB의 길이는 원주의
9
이므로
⑴ 86˘ ⑵ 60˘
1
∠ACB=180˘_ =20˘ yy`
9
0877 네 점 A, B, C, D가 한 원 위에 있으므로
1
μ CD의 길이는 원주의 이므로 ∠ADB=∠ACB=22˘
5
1 △APC에서 ∠DAC=35˘+22˘=57˘
∠DBC=180˘_ =36˘ yy`
5 ∴ ∠x=22˘+57˘=79˘ 79˘
따라서 △PBC에서
∠x=20˘+36˘=56˘ yy` 0878 ∠ACD=∠x라고 하면 ∠ABD=∠x이고
56˘ △APC에서 ∠PAC=∠x-50˘
△ABQ에서 ∠x+(∠x-50˘)=100˘
단계 채점요소 배점
2∠x=150˘ ∴ ∠x=75˘ 75˘
∠ACB의 크기 구하기 40%
∠DBC의 크기 구하기 40%
∠x의 크기 구하기 20% 0879 OB”를 그으면 A
D
∠OBA=∠OAB=25˘ x
25˘
∠OBC=∠OCB=40˘ O y
25˘
∴ ∠ABC=25˘+40˘=65˘
0873 ∠ADC는 ® ABC에 대한 원주각이므로 40˘ 40˘
C
ABCD가 원에 내접하므로 B
1+2
∠ADC=180˘_ =60˘ 60˘
1+2+3+3 ∠x=180˘-65˘=115˘

07. 원주각 83
(066~103)개념RPM3-2(정답) 2014.11.24 02:58 PM 페이지84 다민 2540DPI 175LPI

RPM 알피엠

∠y=2∠ABC=2_65˘=130˘ 0884 ∠BDC=∠BAC=55˘이므로


∴ ∠y-∠x=130˘-115˘=15˘ ① ∠ADC=45˘+55˘=100˘
ABCD가 원에 내접하므로
∠ABE=∠ADC=100˘ 100˘
0880 ⑴ AD”=BD”이므로 △ABD에서
1
∠DAB=∠DBA=
2
(180˘-40˘)=70˘ 0885 △DCE에서 ∠DCE=100˘-35˘=65˘
ABCD가 원에 내접하므로 ABCD가 원에 내접하므로
∠x=180˘-70˘=110˘ ∠BAD=∠DCE=65˘ 65˘
⑵ ∠ADB=90˘이므로 △DAB에서
2
∠DAB=180˘-(90˘+30˘)=60˘ 0886 ®ADC의 길이는 원주의
3
이므로
ABCD가 원에 내접하므로
2
∠x=180˘-60˘=120˘ ∠ABC=180˘_ =120˘
3
⑴ 110˘ ⑵ 120˘ ABCD가 원에 내접하므로
∠ADC=180˘-∠ABC=180˘-120˘=60˘ yy`

0881 ∠BCE=∠BDE=62˘이므로 △BCF에서 `® BCD의 길이는 원주의


3
5
이므로
∠x=20˘+62˘=82˘
3
ABDE가 원에 내접하므로 ∠DAB=180˘_ =108˘
5
∠y=180˘-62˘=118˘ ABCD가 원에 내접하므로
∴ ∠x+∠y=82˘+118˘=200˘ 200˘ ∠DCE=∠DAB=108˘ yy`

∴ ∠ADC+∠DCE=60˘+108˘=168˘ yy`

168˘
0882 BC”=CD”에서
x 단계 채점요소 배점
μ BC=μ CD이므로 80˘ D
A 120˘ ∠ADC의 크기 구하기 40%
∠BAC=∠BDC=∠CAD=∠CBD
y ∠DCE의 크기 구하기 40%
∠BAC+∠CAD=80˘이므로 B
∠ADC와 ∠DCE의 크기의 합 구하기 20%
C
1
∠BAC= _80˘=40˘
2
△ABD에서 ∠ABD=120˘-40˘=80˘이므로
0887 ① ∠ADC=180˘-(45˘+15˘)=120˘
∠x=180˘-(80˘+80˘)=20˘
② ∴ ∠ABC+∠ADC=180˘
∠y=∠ABD=80˘
② ∠DCE+∠BAD
∴ ∠y-∠x=80˘-20˘=60˘ 60˘
③ ∠BDC, ∠ACD의 크기를 알 수 없다.
④ ∠ADC=∠ABE=100˘
0883 ⑴ ABCD가 원에 내접하므로 ⑤ ∠A+∠C=180˘ ②, ③
∠x=∠BAD=100˘
이때 ∠BCD=180˘-100˘=80˘이므로 0888 ABCD가 원에 내접하려면
∠y=2∠BCD=2_80˘=160˘ ∠BDC=∠BAC=54˘이어야 하므로 △PCD에서
⑵ ABCD가 원에 내접하므로 ∠ABC=∠ADE=70˘ ∠x=180˘-(54˘+96˘)=30˘ 30˘
∠x+30˘=70˘
∴ ∠x=40˘ 0889 ABCD가 원에 내접하려면
또한 BC”가 원 O의 지름이므로 ∠BDC=90˘ ∠ABC=∠ADE=180˘-130˘=50˘ yy`
△DBC에서 △ABF에서 ∠DAE=50˘+35˘=85˘ yy`
∠BCD=180˘-(30˘+90˘)=60˘ 따라서 △ADE에서
∠y+60˘=180˘ ∴ ∠y=120˘ ∠x=180˘-(85˘+50˘)=45˘ yy`
⑴ ∠x=100˘, ∠y=160˘ ⑵ ∠x=40˘, ∠y=120˘ 45˘

84 정답과 풀이
(066~103)개념RPM3-2(정답) 2014.11.24 02:58 PM 페이지85 다민 2540DPI 175LPI

● 본문 118쪽 ~ 120쪽 ●

단계 채점요소 배점 ∴ ∠B+∠D+∠F
∠ABC의 크기 구하기 40% =(∠ABE+∠CBE)+∠D+∠F
∠DAE의 크기 구하기 30% =(∠ABE+∠F)+(∠CBE+∠D)
∠x의 크기 구하기 30% =180˘+180˘
=360˘ yy`
다른풀이
360˘
ABCD가 원에 내접하려면
∠ABC+130˘=180˘ ∴ ∠ABC=50˘ 단계 채점요소 배점

△ABF에서 ∠DAE=50˘+35˘=85˘ 보조선을 그어 2개의 사각형으로 나누기 30%

따라서 △ADE에서 원에 내접하는 사각형의 대각의 크기의 합이 180˘임을 이용하기 40%


∠B+∠D+∠F의 크기 구하기 30%
130˘=∠x+85˘ ∴ ∠x=45˘

0890 항상 원에 내접하는 사각형은 한 쌍의 대각의 크기의 0895 ABCD가 원에 내접하므로


합이 180˘이다. ∠CDQ=∠B=56˘
등변사다리꼴, 직사각형, 정사각형은 대각의 크기의 합이 180˘ △PBC에서 ∠DCQ=56˘+24˘=80˘
이므로 항상 원에 내접한다. ㄴ, ㄹ, ㅂ 따라서 △DCQ에서
∠x=180˘-(56˘+80˘)=44˘ 44˘
0891 BD”를 그으면 ABDE가 원 E
에 내접하므로 O 138˘ D 0896 ABCD가 원에 내접하므로
∠A+∠BDE=180˘에서 A 76˘
∠QDC=∠ABC=∠x
∠BDE=180˘-76˘=104˘ C △PBC에서 ∠DCQ=∠x+43˘
B
∴ ∠BDC=138˘-104˘=34˘ 따라서 △DCQ에서
∴ ∠BOC=2∠BDC=2_34˘=68˘ 68˘ ∠x+(∠x+43˘)+33˘=180˘
2∠x=104˘ ∴ ∠x=52˘ 52˘
0892 BD”를 그으면 ABDE가 원에 A
내접하므로 74˘
O 0897 ∠ADP=∠QDC=∠a라고 하면
∠ABD=180˘-102˘=78˘ 102˘ E
98˘ △PAD에서 ∠DAB=∠a+42˘
1 B D
∠CBD= ∠COD C △DCQ에서 ∠x=∠a+40˘
2
1 ABCD가 원에 내접하므로
= _98˘=49˘
2 (∠a+42˘)+(∠a+40˘)=180˘
∴ ∠B=78˘+49˘=127˘ 127˘ 2∠a=98˘ ∴ ∠a=49˘
∴ ∠x=49˘+40˘=89˘ 89˘
0893 AD”를 그으면 ABCD가 원에 F
E
내접하므로 A 50˘ 0898 ABCD가 원에 내접하므로
∠BAD=180˘-110˘=70˘ 70˘
∠ABC=180˘-130˘=50˘
D
∠FAD=120˘-70˘=50˘ △BCP에서 ∠DCQ=∠x+50˘
B 110˘
따라서 ADEF가 원에 내접하므로 C 따라서 △DCQ에서
∠E=180˘-50˘ (∠x+50˘)+42˘=130˘
=130˘ 130˘ ∴ ∠x=38˘ 38˘

0894 BE”를 그으면 yy` A


0899 PQ”를 그으면 ABQP, D
P
ABEF가 원에 내접하므로 F
PQCD가 각각 두 원 O, O'에 A 100˘
B
∠ABE+∠F=180˘ 내접하므로 100˘ O O'
B C
또한 BCDE가 원에 내접하므로 C E ∠DPQ=∠ABQ=100˘ Q

∠CBE+∠D=180˘ yy`
D ∴ ∠QCD=180˘-100˘=80˘ ③

07. 원주각 85
(066~103)개념RPM3-2(정답) 2014.11.24 02:58 PM 페이지86 다민 2540DPI 175LPI

RPM 알피엠

0900 ∠PAB=
1 1
∠POB= _150˘=75˘ 0906 P’A”=PB”이므로 △PBA에서
2 2
1
∠PBA=∠PAB= (180˘-62˘)=59˘ yy`
PQ”를 그으면 ABQP, P D 2
PQCD가 각각 두 원 O, O'에 A
75˘ O O' PT≥가 원 O의 접선이므로
75˘ C
내접하므로 150˘ ∠ABC=∠TAC=72˘ yy`
Q
∠PQC=∠PAB=75˘ B ∴ ∠x=180˘-(59˘+72˘)=49˘ yy`

∴ ∠PDC=180˘-75˘=105˘ 105˘ 49˘

0901 DBQP가 원에 내접하므로 단계 채점요소 배점


∠PBA의 크기 구하기 40%
∠BQP=∠ADP=85˘ yy`
∠ABC의 크기 구하기 40%
PQCE가 원에 내접하므로
∠x의 크기 구하기 20%
∠CEP=∠BQP=85˘ yy`

85˘
0907 △ABD에서
단계 채점요소 배점
∠BAD=180˘-(34˘+58˘)=88˘
∠BQP의 크기 구하기 50%
ABCD는 원에 내접하므로
∠CEP의 크기 구하기 50%
∠y=180˘-∠BAD=180˘-88˘=92˘
또한 직선 CT는 원의 접선이므로
0902 ABCH가 원에 내접하므로
∠DBC=∠DCT=46˘
∠HCD=∠HAB=95˘
△BCD에서
HCDG가 원에 내접하므로
∠x=180˘-(46˘+92˘)=42˘
∠FGD=∠HCD=95˘
∴ ∠y-∠x=92˘-42˘=50˘ 50˘
GDEF가 원에 내접하므로
∠DEF=180˘-95˘=85˘ 85˘
0908 ∠ADB=∠ACB=32˘
0903 ∠ACB=∠ABT=58˘이므로 이때 ABCD가 원에 내접하므로
∠AOB=2_58˘=116˘ ∠CBA=180˘-∠CDA
이때 △OAB는 O’A=
” OB”인 이등변삼각형이므로 =180˘-(46˘+32˘)=102˘
1 ∴ ∠CAT=∠CBA=102˘ 102˘
∠OAB= (180˘-116˘)=32˘ ⑤
2 다른풀이
∠CAB=∠CDB=46˘이므로 △ABC에서
0904 ∠ACB=∠ABT=80˘
∠CBA=180˘-(32˘+46˘)=102˘
μ AB=2μ BC이므로
∴ ∠CAT=∠CBA=102˘
∠ACB : ∠CAB=2 : 1
80˘ : ∠CAB=2 : 1
∴ ∠CAB=40˘ 40˘ 0909 μ AB=μ BC이므로
1
∠ACB=∠BAC= (180˘-106˘)=37˘
2
1
0905 ⑴ ∠CAB=
2
_150˘=75˘
이때 AD”∥BC”이므로
∠ACB=∠BAT'=74˘ ∠CAD=∠ACB=37˘ (엇각)
따라서 △ABC에서 ∴ ∠DCT=∠CAD=37˘ 37˘
∠x=180˘-(74˘+75˘)=31˘
⑵ AP”=AT”이므로 ∠ATP=∠APT=36˘ 0910 ABCD는 원에 내접하므로
∠ABT=∠ATP=36˘ ∠ADC=180˘-110˘=70˘
따라서 △BPT에서 △DCP에서
∠x=180˘-(36˘+36˘+36˘)=72˘ 70˘=∠DCP+46˘ ∴ ∠DCP=24˘
⑴ 31˘ ⑵ 72˘ ∴ ∠CAD=∠DCP=24˘ 24˘

86 정답과 풀이
(066~103)개념RPM3-2(정답) 2014.11.24 02:58 PM 페이지87 다민 2540DPI 175LPI

● 본문 120쪽 ~ 123쪽 ●

0911 BT”를 그으면 AB”는 원 O의 지름 A 0915 △BED는 BD”=BE”인 이등변삼각형이므로


x
이므로 ∠ATB=90˘ 1
O ∠BED= (180˘-50˘)=65˘
또한 PT≥는 원 O의 접선이므로 2
68˘ 68˘ y B 이때 BC”가 원 O의 접선이므로
∠ABT=∠ATC=68˘ P
C T
△ATB에서 ∠DFE=∠BED=65˘

∠x=180˘-(90˘+68˘)=22˘ 따라서 △DEF에서

△ATP에서 ∠EDF=180˘-(65˘+48˘)=67˘ 67˘

68˘=22˘+∠y ∴ ∠y=46˘
∴ ∠y-∠x=46˘-22˘=24˘ ④ 0916 △PBA는 P’A”=PB”인 이등변삼각형이므로
1
∠PBA= (180˘-52˘)=64˘
0912 A’T를
” 그으면 AB”는 원 O의 지
25˘
B 2
름이므로 ∠ATB=90˘ O 또한 P’A≥가 원 O의 접선이므로
△ATB에서 ∠ABC=∠CAD=75˘
A 65˘
∠BAT=180˘-(90˘+25˘) P ∴ ∠CBE=180˘-(64˘+75˘)=41˘ 41˘
25˘ T
=65˘
또한 PT≥는 원 O의 접선이므로
0917 △BED는 BD”=BE”인 이등변삼각형이므로
∠PTA=∠ABT=25˘
1
따라서 △APT에서 ∠BED= (180˘-54˘)=63˘
2
65˘=∠APT+25˘ ∴ ∠APT=40˘ 40˘ ∴ ∠x=∠BED=63˘
BC”는 원 O의 접선이므로
0913 BT”를 그으면 BC”는 A
∠CEF=∠EDF=62˘
원 O의 지름이므로 55˘
B △CFE는 CF”=CE”인 이등변삼각형이므로
∠BTC=90˘ P x 55˘ O 35˘
C
∠CFE=∠CEF=62˘
∠TBC=∠TAC=55˘
35˘ T ∴ ∠y=180˘-(62˘+62˘)=56˘
△BTC에서
∴ ∠x+∠y=63˘+56˘=119˘ 119˘
∠BCT=180˘-(55˘+90˘)=35˘
∠BTP=∠BCT=35˘이므로 △BPT에서
55˘=∠x+35˘ ∴ ∠x=20˘ 20˘ 0918 △PCD에서 ∠DCQ=38˘+24˘=62˘
△CQD는 QC”=QD”인 이등변삼각형이므로
0914 A’T”를 긋고 ∠PBT=∠x
B ∠CDQ=∠DCQ=62˘
라고 하면 PT”=BT”이므로 O x ∴ ∠x=180˘-(62˘+62˘)=56˘
∠BPT=∠PBT=∠x yy` A 2x BC”를 그으면 A
x 2x y 24˘ D
△BPT에서 P ∠BCP=∠BDC=24˘이므로
T C 62˘
∠BTC=∠x+∠x=2∠x ∠BCD=180˘-(24˘+62˘)
접선과 현이 이루는 각의 성질에 의해 B 24˘
=94˘ P
x
Q
38˘ C 62˘
∠BAT=∠BTC=2∠x yy` 이때 ABCD는 원에 내접하므로
이때 AB”가 원 O의 지름이므로 ∠ATB=90˘ ∠y+94˘=180˘ ∴ ∠y=86˘
따라서 △ATB에서 ∴ ∠x+∠y=56˘+86˘=142˘ 142˘
2∠x+∠x+90˘=180˘, 3∠x=90˘ ∴ ∠x=30˘
∴ ∠BTC=2∠x=2_30˘=60˘ yy`
0919 두 원에 공통인 접선 PT를 A C
60˘
그으면 65˘ P 70˘
단계 채점요소 배점 ∠BPT=∠BAP=65˘ 65˘ 70˘
D
B T
∠BPT=∠PBT=∠x로 놓기 30% ∠DPT=∠DCP=70˘
∠BAT=∠BTC=2∠x임을 알기 40% ∴ ∠APB=180˘-(65˘+70˘)
∠BTC의 크기 구하기 30%
=45˘ 45˘

07. 원주각 87
(066~103)개념RPM3-2(정답) 2014.11.24 02:58 PM 페이지88 다민 2540DPI 175LPI

RPM 알피엠

0920 ∠x=∠QTB=∠PTA=∠ADT=70˘ yy` ④ ∠BAC=∠CDT=∠CTP A


P
C
△TCB에서 따라서 엇각의 크기가 같으므로
T
∠y=180˘-(45˘+70˘)=65˘ yy` AB”∥PQ”이지만 AB”와 CD”는 평
D Q
∴ ∠x-∠y=70˘-65˘=5˘ yy` 행하지 않다. B


⑤ ∠BAT=∠BTQ=∠DCT A P
단계 채점요소 배점 C
따라서 동위각의 크기가 같으므로
∠x의 크기 구하기 40% T
AB”∥CD”이다.
∠y의 크기 구하기 40%
B D Q
∠x-∠y의 크기 구하기 20%

0921 AB”를 그으면 C 0925 ∠DCT=180˘-100˘=80˘이므로


P A
∠ATP=∠ABT 66˘ ∠DTP=∠DCT=80˘
O'
∠ABT=∠C=66˘ T O 또한 ∠BTQ=∠BAT=70˘이므로
72˘
∴ ∠ATP=66˘ B ∠x=180˘-(70˘+80˘)=30˘ 30˘
D
66˘
0926 ∠y=2∠ABC=2_70˘=140˘
ABCD가 원 O에 내접하므로
0922 직선 PQ가 두 원의 공통인 접선이므로
∠x+70˘=180˘ ∴ ∠x=110˘
∠y=∠ABT=70˘
∴ ∠x+∠y=110˘+140˘=250˘ ④
∠x=∠y=70˘
∴ ∠x+∠y=70˘+70˘=140˘ 140˘
0927 AB”는 원 O의 지름이므로
∠ACB=90˘
0923 ① ∠TAB=∠QTD=∠ACD 또한 ABCD는 원 O에 내접하므로
② 큰 원에서 접선과 현이 이루는 각의 성질에 의해 ∠ABC+110˘=180˘ ∴ ∠ABC=70˘
∠PTA=∠BDC △ABC에서 ∠x=180˘-(90˘+70˘)=20˘ ②
③ ①에서 ∠TAB=∠ACD(동위각)이므로 AB”∥CD”
④, ⑤ △TAB와 △TCD에서 0928 OC”를 그으면 P Q
④ ∠TAB=∠TCD, ∠ATB는 공통이므로 30˘
∠AOC=2∠APC=2_30˘=60˘ O
△TABª△TCD (AA 닮음) ∠COB=110˘-60˘=50˘
∴ T’A” : TC”=AB” : CD” 1 1 A 110˘ B
∴ ∠CQB= ∠COB= _50˘
따라서 옳지 않은 것은 ⑤이다. ⑤ 2 2 C
=25˘ ③

0924 ① ∠PAB=∠PQD
A P
E
1
=∠PCE C 0929 μ AB의 길이가 원주의
6
이므로

② 따라서 엇각의 크기가 같으므로 D 1


B Q ∠ACB=180˘_ =30˘
AB”∥CD”이다. 6
② ∠BAT=∠BTQ=∠CTP A P
μ AB=μ CD이므로
C
=∠CDT ∠DBC=∠ACB=30˘
② 따라서 엇각의 크기가 같으므로 T ∴ ∠x=30˘+30˘=60˘ 60˘
Q D
AB”∥CD”이다. B
0930 ∠ACD=∠ABD=60˘이므로 △DPC에서
③ ∠BAQ=∠BPQ=∠QDC P C
A ∠CDP=180˘-(70˘+60˘)=50˘
따라서 엇각의 크기가 같으므로
AC”가 원 O의 지름이므로 ∠ADC=90˘
AB”∥CD”이다. B
Q ∴ ∠x=90˘-50˘=40˘ 40˘
D

88 정답과 풀이
(066~103)개념RPM3-2(정답) 2014.11.24 02:58 PM 페이지89 다민 2540DPI 175LPI

● 본문 123쪽 ~ 125쪽 ●

0931 ① ∠ADB=∠ACB이므로 네 점 A, B, C, D는 한 0936 OP”를 그으면 P


원 위에 있다. ∠APO=∠PAO=35˘ 35˘45˘
A 35˘ 45˘
② ∠BAC의 크기를 알 수 없으므로 네 점 A, B, C, D가 한 ∠BPO=∠PBO=45˘ 3
O
원 위에 있다고 할 수 없다. ∴ ∠APB=35˘+45˘=80˘ B
③ ∠BAC=∠BDC이므로 네 점 A, B, C, D는 한 원 위에 ∠AOB=2∠APB=2_80˘=160˘
있다. 160
∴ (색칠한 부분의 넓이)=p_3¤ _ =4p ④
④ ∠ADB=∠ACB이므로 네 점 A, B, C, D는 한 원 위에 360

있다.
⑤ ∠BDC=120˘-80˘=40˘이므로 ∠BAC=∠BDC
0937 ABCD가 원에 내접하므로 E
따라서 네 점 A, B, C, D는 한 원 위에 있다. ② ∠EAD=∠BCD에서 A 65˘
D
y 30˘
65˘=∠x+30˘ ∴ ∠x=35˘
x
0932 μ AB=3μ CD이므로 ∠ADB : ∠DBC=3 : 1 BD”를 그으면 ∠BDC=90˘이고 B
O
C

1 ∠BDA=∠x=35˘
∠x : ∠DBC=3 : 1 ∴ ∠DBC= ∠x
3 ∴ ∠y=35˘+90˘=125˘
△DBP에서 ∠x=35˘, ∠y=125˘
1 2
∠x+20˘=∠x, ∠x=20˘
3 3
∴ ∠x=30˘ 30˘ 0938 ∠ABT=∠ATP=30˘이고
AB”는 원 O의 지름이므로 ∠ATB=90˘
0933 μ AB : μ BC : μ CA=1 : 2 : 3에서 △ATB에서 AB”=8 cm
∠ACB : ∠CAB : ∠ABC=1 : 2 : 3 1
AT”=8 sin 30˘=8_ =4(cm)
2
2
① ∠CAB=180˘_ =60˘
1+2+3 '3
BT”=8 cos 30˘=8_ =4'3(cm)
3 2
② ∠ABC=180˘_ =90˘
1+2+3 1
∴ △ATB= _4_4'3=8'3(cm¤ ) 8'3 cm¤
1 2
③ ∠ACB=180˘_ =30˘
1+2+3
④ △ABC는 직각삼각형이다.
⑤ μ BC : μ CA=2 : 3이므로 6p : μ CA=2 : 3 0939 ABQP와 PQCD는 원에 내접하므로
① ∠PQC=∠PAB=100˘
∴ μ CA=9p(cm) ⑤
② ∠ABQ의 크기는 알 수 없다.
③ ∠CDP+∠PQC=180˘이므로
0934 △DAE에서 ∠DAE+15˘=80˘
∠CDP+100˘=180˘ ∴ ∠CDP=80˘
∴ ∠DAE=65˘
④ ∠PAB+∠CDP=100˘+80˘=180˘이므로
네 점 A, B, C, D가 한 원 위에 있으므로
AB”∥DC”
∠ACB=∠ADB=15˘
⑤ ∠B=∠DPQ이므로 ∠DPQ+∠C=180˘에서
△APC에서
∠B+∠C=180˘ ②
∠x+15˘=65˘ ∴ ∠x=50˘ 50˘

0935 AC”를 그으면 EACD는 원 E 0940 BC”를 그으면 AB”는 반원 O의 P


66˘
에 내접하므로 지름이므로 ∠ACB=90˘ C D
D
40˘ y
∠EAC+∠CDE=180˘ A x △PCB에서
또한 ∠BAC=∠BEC=40˘이므로 40˘ ∠PBC=180˘-(66˘+90˘) A B
O
∠x+∠y B C =24˘
=∠BAC+∠EAC+∠CDE ∴ ∠COD=2∠PBC
=40˘+180˘=220˘ 220˘ =2_24˘=48˘ ②

07. 원주각 89
(066~103)개념RPM3-2(정답) 2014.11.24 02:58 PM 페이지90 다민 2540DPI 175LPI

RPM 알피엠

0941 △PAC는 P’A”=PC”인 이등변삼각형이므로


0947 ∠B=∠CQP=
1
∠COP
2
1
∠PCA= (180˘-54˘)=63˘
2 1
= _176˘=88˘ ⑤
2
∠ABC=∠PCA=63˘
또한 μ AB : μ BC=2 : 1이므로
∠ACB : ∠BAC=2 : 1 0948 AB”를 그으면 직선 TT' T D
1 은 원 O의 접선이므로 A
∴ ∠BAC= ∠ACB 70˘
2 ∠x=∠ABP x
P O
△ABC에서 O'
그런데 ABCD가 원 O'에 x
1 내접하므로 B 62˘
63˘+ ∠ACB+∠ACB=180˘ T'
2
∠ABP=∠ADC=70˘ C
3
∠ACB=117˘ ∴ ∠ACB=78˘ 78˘ ∴ ∠x=70˘ 70˘
2

0942 ABCD가 원에 내접하므로 0949 △PAC에서 C


∠QDC=∠x ∠CAB=∠x+36˘ x
D
△PBC에서 ∠DCQ=∠x+22˘ BC”, BD”를 그으면 x+36˘
P 36˘
△DCQ에서 μ AB=μ BC=μ CD이므로 A x B
∠x+(∠x+22˘)+52˘=180˘ ∠ACB=∠BAC=∠CBD x+36˘
2∠x=106˘ ∴ ∠x=53˘ ④ =∠x+36˘
∠ABD=∠ACD=∠x이므로 △ABC에서
0943 △ABC에서 (∠x+36˘)+(∠x+∠x+36˘)+(∠x+36˘)=180˘
∠B=180˘-(65˘+55˘)=60˘ 4∠x=72˘ ∴ ∠x=18˘ 18˘
△BED에서 BD”=BE”이므로

∠BED=
1
(180˘-60˘)=60˘ 0950 CD”를 그으면 반직선 TC가 접 A
2
선이므로 35˘
∴ ∠DFE=∠BED=60˘ 60˘ B 100˘
∠DCT=∠CAD=35˘ yy` D
ABCD가 원에 내접하므로
0944 ∠BTQ=∠BAT=55˘
∠CDT=∠ABC=100˘ yy`
C
T

∠DTQ=∠DCT=75˘
따라서 △DCT에서
∴ ∠x=180˘-(55˘+75˘)=50˘ ③
∠DTC=180˘-(35˘+100˘)
=45˘ yy`
0945 직선 PQ가 두 원의 공통인 접선이
A P 45˘
므로 C

∠BAT=∠BTQ=∠DCT (③) T 단계 채점요소 배점

∠ABT=∠ATP=∠CDT (⑤) D ∠DCT의 크기 구하기 40%


B Q
∠CDT의 크기 구하기 40%
∴ △ABTª△CDT (AA 닮음) (④)
∠DTC의 크기 구하기 20%
또한 동위각의 크기가 같으므로
AB”∥CD” (①) ②
0951 ∠BCD=∠BAD=∠x yy`

0946 HC”, GD”를 그으면


A H G F △BPC에서 35˘+∠PBC=∠x이므로
ABCH, HCDG, 85˘
∠PBC=∠x-35˘ yy`

GDEF는 각각 원에 내접하 B 88˘ △AQB에서


C D E
므로 ∠x+(∠x-35˘)=75˘
∠ABC=∠CHG=∠GDE 2∠x=110˘ ∴ ∠x=55˘ yy`

=180˘-85˘=95˘ 95˘ 55˘

90 정답과 풀이
(066~103)개념RPM3-2(정답) 2014.11.24 02:58 PM 페이지91 다민 2540DPI 175LPI

● 본문 125쪽 ~ 127쪽 ●

단계 채점요소 배점 ∠ABT=180˘-(56˘+90˘)
∠BCD=∠x임을 알기 30% =34˘
∠PBC를 ∠x로 나타내기 30% ∴ ∠ATP=∠ABT=34˘
∠x의 크기 구하기 40% △APT에서
56˘=∠x+34˘ ∴ ∠x=22˘ 22˘

0952 BC”를 그으면 AC”는 원 O의 A


지름이므로
x 0955 ∠DBC=∠DAC=∠x이므로 △DBE에서
O
∠ADB=∠x+40˘ yy ㉠
∠ABC=90˘ C
62˘
y ∠ADC=90˘이고 μ BC=μ CD이므로
∠ACB=∠ABT=62˘ yy` P
T 62˘ B ∠BDC=∠DBC=∠x에서
△ABC에서
∠ADB=90˘-∠x yy ㉡
∠x=180˘-(90˘+62˘)=28˘ yy`
㉠, ㉡에서
△ABP에서
∠x+40˘=90˘-∠x
62˘=28˘+∠y ∴ ∠y=34˘ yy`
2∠x=50˘ ∴ ∠x=25˘
∴ ∠y-∠x=34˘-28˘=6˘ yy`
삼각형의 외각의 성질에 의해

∠y=∠x+(∠x+40˘)
단계 채점요소 배점 =2∠x+40˘
∠ACB의 크기 구하기 30% =2_25˘+40˘
∠x의 크기 구하기 30%
=90˘ ∠x=25˘, ∠y=90˘
∠y의 크기 구하기 30%
∠y-∠x의 크기 구하기 10%
0956 PC”를 그으면 ∠APC=90˘
Q 55˘
P 35˘
이므로
0953 ∠BAC=∠x라 하고 A ∠CPB=180˘-(55˘+90˘) A 55˘ x B
C
BC”를 그으면 x =35˘
∠PCB=∠BAC=∠x B D 또한 ∠PCA=∠APQ=55˘이므로 △PCB에서
이므로 △BPC에서 40˘
55˘=35˘+∠x
P 43˘
∠ABC=43˘+∠x yy` x C
∴ ∠x=20˘ 20˘
μ CD=μ DA이므로
∠CAD=∠ACD=40˘ 0957 PQ”의 연장선과 AB”가 만나 P
△ACD에서 는 점을 C라 하자.
O
∠QAC=∠APC=∠a, O'
∠CDA=180˘-(40˘+40˘)=100˘ yy` 40˘ Q 52˘
ABCD가 원에 내접하므로 ∠QBC=∠BPC=∠b라 하면
A C B
(43˘+∠x)+100˘=180˘ ∠APB=∠a+∠b
∴ ∠x=37˘ yy` △PAB에서
37˘ (∠a+∠b)+(40˘+∠a)+(∠b+52˘)=180˘이므로
2(∠a+∠b)=88˘ ∴ ∠a+∠b=44˘
단계 채점요소 배점
∴ ∠APB=∠a+∠b=44˘ 44˘
∠BAC=∠x라 하고 ∠ABC의 크기를 ∠x로 나타내기 40%
∠CDA의 크기 구하기 20%
∠BAC의 크기 구하기 40%

0954 A’T를
” 그으면 C
∠ATB=90˘이고 A 56˘
P x B
∠BAT=∠BCT=56˘ 56˘ O 34˘

△ATB에서 34˘
T

07. 원주각 91
(066~103)개념RPM3-2(정답) 2014.11.24 02:58 PM 페이지92 다민 2540DPI 175LPI

Ⅳ 원의 성질

08 원주각의활용 0970 PO”의 연장선이 원 O와 만


나는 점을 D라고 하면 x
4
P C D
6_(6+4)=4(4+2x) O
6
0958 6_5=x_3이므로 3x=30
8x=44 A 4
∴ x=10 10 B
11 11
∴ x=
2 2
0959 3_x=4_6이므로 3x=24
∴ x=8 8 0971 x_3x=3_(3+5), 3x¤ =24, x¤ =8
∴ x=2'2 (∵ x>0) 2'2
0960 x_x=8_2이므로 x¤ =16
∴ x=4 (∵ x>0) 4 0972 P’A”¥PB”=PC”¥PD”이어야 하므로
3_11=6_x, 6x=33

0961 x(12-x)=9_4이므로 ∴ x=
11 11
2 2
x¤ -12x+36=0, (x-6)¤ =0
∴ x=6 (중근) 6
0973 P’A”¥PB”=PC”¥PD”이어야 하므로
3_8=4_x, 4x=24
0962 x(x+3)=4_(4+6)이므로
∴ x=6 6
x¤ +3x-40=0, (x+8)(x-5)=0
∴ x=5 (∵ x>0) 5
0974 P’A”¥PB”=PC”¥PD”이어야 하므로
2_(2+x)=3_(3+3), 2x=14
0963 5_(5+7)=4_(4+x)이므로
∴ x=7 7
4x=44 ∴ x=11 11

0975 P’A”¥PB”=PC”¥PD”이어야 하므로


0964 PC”=PD”=6이므로 9_x=6_6
4_(4+6)=5_(5+x), 5x=15
9x=36 ∴ x=4 4
∴ x=3 3

0965 PD”=PC”=x이고 OP”=4-2=2이므로


0976 P’A”¥PC”=PB”¥PD”이어야 하므로
2_(2+4)=x_x, x¤ =12 2_9=x_3, 3x=18
∴ x=2'3 (∵ x>0) 2'3 ∴ x=6 6

0966 P’A=
” 9-x이므로 (9-x)(9+x)=4_8 0977 P’A”¥PC”=PB”¥PD”이어야 하므로
x¤ =49 ∴ x=7 (∵ x>0) 7 4_x=8_3, 4x=24
∴ x=6 6
0967 OP”=x-2이므로 2_{(x-2)+x}=4_7
4x=32 ∴ x=8 8 0978 P’A”¥PD”=PB”¥PC”이어야 하므로
2_(2+x)=3_(3+5)
0968 P’A=
” 10-x이므로 2x=20
(10-x)(10+x)=7_(7+5), x¤ =16 ∴ x=10 10
∴ x=4 (∵ x>0) 4
0979 P’A”¥PD”=PB”¥PC”이어야 하므로

0969 6_(6+3)=3_(3+2x), 6x=45 x(x+5)=3_(3+9), x¤ +5x-36=0


15 15 (x+9)(x-4)=0
∴ x=
2 2 ∴ x=4 (∵ x>0) 4

92 정답과 풀이
(066~103)개념RPM3-2(정답) 2014.11.24 02:58 PM 페이지93 다민 2540DPI 175LPI

● 본문 129쪽 ~ 132쪽 ●

0980 x¤ =2_(2+6)이므로 x¤ =16 0991 ㈎ ∠ABC ㈏ AB” ㈐ AQ” ㈑ 9 ㈒ 3'6


∴ x=4 (∵ x>0) 4

0992 PB”=x라고 하면 P’A”=17-x이므로


0981 12¤ =x(x+10)이므로 x¤ +10x-144=0 (17-x)_x=6_7, x¤ -17x+42=0
(x-8)(x+18)=0 ∴ x=8 (∵ x>0) 8 (x-3)(x-14)=0
∴ x=3(∵ P’A”>PB”) 3
0982 6¤ =3_(3+2x)이므로 6x=27
9 9
∴ x=
2 2
0993 P’A_
” 6=3_10, 6P’A=
” 30
∴ P’A=
” 5 ②

0983 x¤ =4_(4+6)이므로 x¤ =40


∴ x=2'ß10 (∵ x>0) 2'ß10 0994 P’A”=PB”이므로
1
P’A”=PB”=12_ =6
2
0984 ㄱ. 6¤ =4_(4+5)
PC”=x라고 하면 PD”=15-x이므로
ㄴ. (4'3 )¤ =4_(4+8)
6_6=x(15-x)
ㄷ. 8¤ =4_(4+12)
x¤ -15x+36=0, (x-3)(x-12)=0
ㄹ. 4¤ +3_(3+2)
∴ x=3 또는 x=12
따라서 PT”가 세 점 A, B, T를 지나는 원의 접선인 것은 ㄱ,
그런데 PC”>PD”이므로
ㄴ, ㄷ이다. ㄱ, ㄴ, ㄷ
x=12 12

0985 PT”=P’T'”이므로 x=4


0995 PC” : PD”=2 : 3이므로
원 O'에서 4¤ =2_(2+y)이므로
PC”=2k, PD”=3k(k>0)라고 하면
2y=12 ∴ y=6 x=4, y=6
6_16=2k_3k, 6k¤ =96
k¤ =16
0986 PT”=P’T'”이므로 x=8
∴ k=4(∵ k>0)
원 O'에서 8¤ =4_(4+y)이므로
따라서 PC”=2_4=8, PD”=3_4=12이므로
4y=48 ∴ y=12 x=8, y=12
CD”=PC”+PD”=8+12=20 20

0987 원 O에서 PT” ¤ =3_(3+5)


0996 3_(3+9)=4_(4+x), 4x=20
원 O'에서 PT” ¤ =x(x+2)이므로
∴ x=5 ①
3_(3+5)=x(x+2)
x¤ +2x-24=0, (x+6)(x-4)=0
∴ x=4 (∵ x>0) 4 0997 PC”=x라고 하면 PD”=2x이므로
4_(4+10)=x_2x
2x¤ =56, x¤ =28
0988 원 O에서 PT” ¤ =4_(4+2x)
∴ x=2'7 (∵ x>0) 2'7
원 O'에서 PT” ¤ =5_(5+7)이므로
4_(4+2x)=5_(5+7)
11 11 0998 P’A”=x라고 하면 PB”=x-12이므로
8x=44 ∴ x=
2 2 (x-12)_x=10_(10+6)
x¤ -12x-160=0, (x-20)(x+8)=0
0989 ㈎ ∠BAQ ㈏ Q’A” ㈐ 10 ㈑ 2'ß10 ∴ x=20(∵ x>0) 20

0990 ㈎ ∠AQC ㈏ △AQC ㈐ AQ” ㈑ AP” 0999 6_QB”=4_9이므로 QB”=6 yy`


㈒3 ㈓7 PB”=x라고 하면

08. 원주각의 활용 93
(066~103)개념RPM3-2(정답) 2014.11.24 02:58 PM 페이지94 다민 2540DPI 175LPI

RPM 알피엠

x_(x+6+6)=5_(5+4)이므로 yy` 9_12=x_3x, x¤ =36


x¤ +12x-45=0, (x+15)(x-3)=0 ∴ x=6 (∵ x>0)
∴ x=3 (∵ x>0) yy` 따라서 반원 O의 반지름의 길이는
3 OD”=2x=2_6=12 12

단계 채점요소 배점
QB”의 길이 구하기 40% 1004 PO”의 연장선이 원 O와 만나는 D

PB”=x로 놓고 x에 관한 식 세우기 30% 점을 D라 하고 원 O의 반지름의 길이를 x


O
PB”의 길이 구하기 30% x라고 하면
P’A=
” 13-x, PD”=13+x이므로 A 13
8
P 4 C
(13-x)(13+x)=4_(4+8) B

1000 원 O를 그려 CP”의 연장선과 만 C x¤ =121


나는 점을 D라고 하자. x ∴ x=11(∵ x>0) 11
O
원의 중심에서 현에 내린 수선은 현을 A B
6 P 3
이등분하므로 PC”=PD”=x라고 하면 1005 ⑴ PD”=3+2x이므로
AP”=12-3=9이므로 D 5_(5+4)=3_(3+2x), 6x=36
9_3=x¤ , x¤ =27 ∴ x=6
∴ x=3'3(∵ x>0) 3'3 ⑵ PD”=x+16이므로
5_(5+11)=x(x+16)
x¤ +16x-80=0, (x+20)(x-4)=0
1001 ⑴ P’A=
” 6+x, PB”=6-x이므로 ∴ x=4(∵ x>0) ⑴6 ⑵4
(6+x)(6-x)=4_5, x¤ =16
∴ x=4(∵ x>0)
1006 AQ”_2=4_6이므로 AQ”=12
⑵ P’A=
” 2x-2이므로
PE”=x라고 하면 PD”=2x이므로
(2x-2)_2=8_4, 4x=36
2_(2+2+12)=x_2x
∴ x=9 ⑴4 ⑵9
x¤ =16
∴ x=4(∵ x>0) 4

1002 원 O의 반지름의 길이를 x라고 하면


P’A=
” x-4, PB”=x+4이므로 1007 PO”의 연장선이 원 O와 만나는 P
점을 E라고 하자.
(x-4)(x+4)=(2'3)¤ yy` 7 cm
6 cm
PE”=(6+2r) cm이므로
x¤ =28 ∴ x=2'7 (∵ x>0) yy`
5 cm C D
7_(5+7)=6_(6+2r)
따라서 원 O의 넓이는 r cm
12r=48 A B
p_(2'7 )¤ =28p yy`
O
∴ r=4(cm)
28p
따라서 반원 O의 넓이는 E
단계 채점요소 배점 1
_p_4¤ =8p(cm¤ ) 8p cm¤
원 O의 반지름의 길이를 구하는 식 세우기 40% 2
원 O의 반지름의 길이 구하기 30%
원 O의 넓이 구하기 30%
1008 P’A”¥PB”=PC”¥PD”이므로
(6+3)_5=3_(5+BD”)
3BD”=30 ∴ BD”=10 ④
1003 원 O를 그려 DO”의 연장선과 D 12
C
만나는 점을 E라고 하자.
A
9
P
x
B
1009 P’A”¥PB”=PC”¥PD”이므로
OP”=PD”=x라고 하면 OE”=2x이므 O 2x 12_PB”=4_6

E 12PB”=24 ∴ PB”=2 2

94 정답과 풀이
(066~103)개념RPM3-2(정답) 2014.11.24 02:58 PM 페이지95 다민 2540DPI 175LPI

● 본문 133쪽 ~ 135쪽 ●

1010 P’A”¥PB”=PC”¥PD”이므로 1015 ∠ADC=∠AEC A


5_(5+4)=3_(3+CD”) 이므로 네 점 A, D, E, C는 한
5 cm
3 CD”=36 ∴ CD”=12 12 원 위에 있다. D P
따라서 BD”¥BA”=BE”¥BC” 4 cm
1011 BP”=x라고 하면 이므로 4_9=3(3+EC”) B 3 cm C
E
PC”=6-x yy` 3EC”=27
P’A”¥PC”=PB”¥PD”이므로 ∴ EC”=9(cm) 9 cm
(12+x)(6-x)=x(6-x+12) yy`

24x=72 ∴ x=3 yy`

3 1016 PT” ¤ =P’A”¥PB”이므로


x¤ =3_(3+9)=36
단계 채점요소 배점
∴ x=6(∵ x>0)
BP”=x로 놓고 PC”의 길이를 x로 나타내기 20%
P’A”¥PB”=PC”¥PD”이므로
BP”의 길이를 구하는 식 세우기 40%
BP”의 길이 구하기 40%
3_(3+9)=4_(4+y)
4y=20 ∴ y=5
∴ x+y=6+5=11 ①
1012 ① 6_(6+4)+4_(4+6)
② 3_(3+5)=(6-2)_6
③ 4_4+5_3
1017 △OMB에서 MÚB”="√5¤ -4¤ ='9=3
④ 3_5+3_6
∴ P’A= ” 3, PB”=9
⑤ 3_7+4_6
PT” ¤ =P’A”¥PB”이므로 PT” ¤ =3_9=27
따라서 네 점 A, B, C, D가 한 원 위에 있는 것은 ②이다.
∴ PT”=3'3 (∵ PT”>0) 3'3

1013 ⑴ ABCD가 원에 내접하려면


1018 PB” ¤ =PA”¥PC”이므로
P’A”¥PD”=PB”¥PC”이어야 하므로 PB” ¤ =6_(6+8)=84
6_(6+4)=x(x+7), x¤ +7x-60=0 ∴ PB”=2'2å1(cm)(∵ PB”>0)
(x+12)(x-5)=0 1
∴ △APB= _6_2'2å1_sin 30˘
∴ x=5(∵ x>0) 2
⑵ △APD에서 1 1
= _6_2'2å1_
2 2
AP”="√10¤ -8¤ ='ß36=6
=3'2å1(cm¤ ) 3'2å1 cm¤
△CDP에서 CP”="√17¤ -8¤ ='∂225=15
이때 ABCD가 원에 내접하려면
P’A”¥PC”=PB”¥PD”이어야 하므로
1019 PQ”=PT”=12 cm이므로
45
6_15=x_8 ∴ x= PA”=x cm라 하면
4
12¤ =x_(12+6), 18x=144
45
⑴5 ⑵
4 ∴ x=8(cm) 8 cm

1014 DB”=16이므로 PD”=PB”=8


P’A” : PC”=4 : 1이므로 PC”=x라고 하면 P’A=
” 4x 1020 PO”의 연장선이 원 O와 만나는 B
네 점 A, B, C, D가 한 원 위에 있으므로 점을 B라 하고 원 O의 반지름의 길이 O x
10
4x_x=8_8, x¤ =16 를 x라고 하면
A
∴ x=4 (∵ x>0) PT” ¤ =P’A”¥PB”이므로
P
8 T
따라서 P’A”=4_4=16, PC”=4이므로 8¤ =(10-x)(10+x), x¤ =36
AC”=P’A”+PC”=16+4=20 20 ∴ x=6(∵ x>0) ②

08. 원주각의 활용 95
(066~103)개념RPM3-2(정답) 2014.11.24 02:58 PM 페이지96 다민 2540DPI 175LPI

RPM 알피엠

1021 PT” ¤ =PC”¥PD”이므로 1026 OC”의 연장선이 원 O와 만나는 B


x-4
PT” ¤ =3_(3+6)=27 두 점을 D, E라고 하자. E
4
O
∴ PT”=3'3(∵ PT”>0) PT” ¤ =P’A”¥PB”이므로 C 5
D x A
∠OTP=90˘이므로 △OPT에서 6¤ =3_(3+5+CB”) 3
3CB”=12 ∴ CB”=4 P
OP”="√5¤ +(3'3)¤ ='ß52=2'ß13 T 6
∴ BP”=BO”+OP”=5+2'ß13 5+2'ß13 또한 원 O의 반지름의 길이를 x라고 하면
CD”¥CE”=CB”¥C’A”이므로
1022 PT” ¤ =P’A”¥PB”이므로 (x+4)(x-4)=4_5
9 x¤ =36 ∴ x=6 (∵ x>0) 6
PT” ¤ = _5=9
5
∴ PT”=3 (∵ PT”>0) yy`
1027 QA”¥QB”=QC”¥QT”이므로
∠PTB=90˘이므로 △BPT에서
PA”=QA”=QB”=x cm라고 하면
BT”="√5¤ -3¤ ='ß16=4
x_x=3_9, x¤ =27
1 1
∴ (원 O의 반지름의 길이)= BT”= _4=2 yy` ∴ x=3'3(cm)(∵ x>0)
2 2
또한, PT” ¤ =PA”¥PB”이므로
따라서 원 O의 넓이는
PT” ¤ =3'3_9'3, PT” ¤ =81
p_2¤ =4p yy`
∴ PT”=9(cm)(∵ PT”>0) 9 cm
4p

단계 채점요소 배점
1028 ” x라고 하면 PT” ¤ =P’A”¥PB”이므로
P’A=
PT”의 길이 구하기 30%
6¤ =x(x+9), x¤ +9x-36=0
원 O의 반지름의 길이 구하기 40%
(x+12)(x-3)=0
원 O의 넓이 구하기 30%
∴ x=3(∵ x>0)
△PTA와 △PBT에서
1023 ㄱ. ∠BAT=∠BTQ=60˘ (접선과 현이 이루는 각)
∠PTA=∠PBT (접선과 현이 이루는 각)
ㄴ. AB”가 원 O의 지름이므로 ∠ATB=90˘
∠P는 공통
∴ ∠TBA=180˘-(90˘+60˘)=30˘
따라서 △PTAª△PBT(AA 닮음)이므로
ㄷ. μ AT : ® BT=∠TBA : ∠BAT
P’A” : PT”=AT” : TB”
=30˘ : 60˘=1 : 2
3 : 6=A’T” : 10
따라서 옳은 것은 ㄱ, ㄹ이다. ㄱ, ㄹ
6A’T”=30
∴ A’T”=5 5
1024 Q’A”¥QB”=QC”¥QT”이므로
Q’A”_4=2_6
∴ Q’A”=3 1029 ⑴ PT” ¤ =PA”¥PB”이므로

또한 PT” ¤ =P’A”¥PB”이므로 P’A=


” x라고 하면 PT” ¤ =3_(3+9)=36

(7'2 )¤ =x_(x+7), x¤ +7x-98=0 ∴ PT”=6(∵ PT”>0)


(x-7)(x+14)=0 △PTA와 △PBT에서

∴ x=7 (∵ x>0) 7 ∠PTA=∠PBT (접선과 현이 이루는 각)


∠P는 공통
1025 QA”¥QB”=QC”¥QD”이므로 따라서 △PTAª△PBT(AA 닮음)이므로
4_5=2_QD” PA” : PT”=T’A” : BT”
∴ QD”=10 3 : 6=4 : x, 3x=24
또한, PT” ¤ =PD”¥PC”에서 PQ”=x라고 하면 ∴ x=8
8¤ =(x-10)(x+2), x¤ -8x-84=0 ⑵ PA”=a라 하면 PT” ¤ =PA”¥PB”이므로
(x-14)(x+6)=0 8¤ =a(a+12), a¤ +12a-64=0
∴ x=14 (∵ x>0) 14 (a+16)(a-4)=0 ∴ a=4(∵ a>0)

96 정답과 풀이
(066~103)개념RPM3-2(정답) 2014.11.24 02:58 PM 페이지97 다민 2540DPI 175LPI

● 본문 135쪽 ~ 137쪽 ●

△PTA와 △PBT에서 1035 ∠BAH=∠ACH이므로


A
∠PTA=∠PBT, ∠P는 공통 B’A는
” 세 점 A, H, C를 지나는 원
8
따라서 △PTAª△PBT (AA 닮음)이므로 의 접선이다.
B C
PT” : PB”=TA” : BT” 따라서 B’A” ¤ =BH”¥BC”이므로 H
16
8 : (4+12)=x : 12, 16x=96 8¤ =BH”_16, 16BH”=64
∴ x=6 ∴ BH”=4 4
⑴8 ⑵6

1030 ∠ABT=∠APT이므로 PT”=BT”=6


또한 ∠ATP=∠ABT(접선과 현이 이루는 각)이므로 1036 PT”=P’T'”=6이므로 PB”=x라고 하면
∠ATP=∠APT ¤
PT” =PB”¥P’A에
” 서
즉, △ATP는 이등변삼각형이므로 6¤ =x(x+9), x¤ +9x-36=0
AP”=AT”=4 (x+12)(x-3)=0
PT” ¤ =P’A”¥PB”이므로 6¤ =4_(4+AB”) ∴ x=3 (∵ x>0) ③
4AB”=20 ∴ AB”=5 5

1031 PT” ¤ =PA”¥PB”이므로


(4'3)¤ =4_(4+AB”) 1037 PT”=P’T'”=8이므로 x=8
4AB”=32 ∴ AB”=8(cm) PT” ¤ =P’A”¥PB”이므로
이때 ∠ABT=∠ATP=30˘이고 8¤ =4_(4+y), 4y=48
∠ATB=90˘이므로 △ATB에서 ∴ y=12 x=8, y=12
1
AT”=8 sin 30˘=8_ =4(cm)
2
'3
BT”=8 cos 30˘=8_ =4'3(cm)
2
1038 원 O에서 PT” ¤ =P’A”¥PB” yy`㉠
따라서 △ATB의 둘레의 길이는
원 O'에서 P’A”¥PB”=PC”¥PD” yy`㉡
8+4+4'3=12+4'3(cm) (12+4'3) cm
㉠, ㉡에서 PT” ¤ =PC”¥PD”이므로
PT” ¤ =3_(3+9)=36
1032 8¤ =4_(4+12), 즉 B
PT” ¤ =P’A”¥PB”이므로 PT”는 세 점 ∴ PT”=6(∵ PT”>0) 6
12
A, B, T를 지나는 원의 접선이다. A
4 30˘
∴ ∠PBT=∠ATP=30˘ 35˘
P
8 T
③ 1039 원 O에서 PT” ¤ =P’A”¥PB”이므로
PT” ¤ =2_(2+7)=18
1033 PT”가 세 점 A, B, T를 지나는 원의 접선이 되려면 ∴ PT”=3'2 (∵ PT”>0) yy`

PT” ¤ =P’A”¥PB”가 성립해야 하므로 또한 원 O'에서 P’T'” ¤ =P’A”¥PB”이므로


6¤ =P’A”¥12, 12P’A=
” 36 P’T'”=PT”=3'2 yy`

∴ P’A=
” 3 3 ∴ T’T'”=PT”+P’T'”
=3'2+3'2
1034 PT” ¤ =P’ A ” ¥ PB” 이므로 B
=6'2 yy`
PT”는 세 점 A, B, T를 지나는 A 56˘
6'2
원의 접선이다.
따라서 ∠ABT=∠ATP=56˘ P 30˘ 단계 채점요소 배점
56˘ T
이므로 △BPT에서 PT”의 길이 구하기 40%

∠ATB=180˘-(30˘+56˘+56˘) P’T'”의 길이 구하기 30%


T’T'”의 길이 구하기 30%
=38˘ 38˘

08. 원주각의 활용 97
(066~103)개념RPM3-2(정답) 2014.11.24 02:58 PM 페이지98 다민 2540DPI 175LPI

RPM 알피엠

1040 P’A”¥PB”=PC”¥PD”이므로 AP”=x라고 하면 AB” : AQ”=AP” : AC”이므로


” x라고 하면
P’A= 10 : (x+7)=x : 12
x(x+8)=7_(7+5), x¤ +8x-84=0 x(x+7)=10_12, x¤ +7x-120=0
(x-6)(x+14)=0 (x-8)(x+15)=0
∴ x=6(∵ x>0) ④ ∴ x=8 (∵ x>0) 8

1046 ∠BAQ=∠CAQ이고 A
1041 P’A”¥PB”=PC”¥PD”이므로
∠CBQ=∠CAQ (μ QC에 대한 원주각)
PC”=x라고 하면 6
이므로 ∠BAQ=∠CBQ
4_(4+6)=x(x+3), x¤ +3x-40=0
따라서 BQ”는 세 점 A, B, P를 지나는 B P C
(x-5)(x+8)=0 3
원의 접선이므로 yy`
∴ x=5(∵ x>0) 5 Q
BQ” ¤ =QP”¥Q’A”=3_(3+6)=27
∴ BQ”=3'3 (∵ BQ”>0) yy`

1042 P’A”¥PB”=PC”¥PD”이므로 3'3


CD”=x라고 하면
단계 채점요소 배점
2_(2+7)=(6-x)_6, 6x=18
BQ”가 세 점 A, B, P를 지나는 원의 접선임을 보이기 60%
∴ x=3 3 BQ”의 길이 구하기 40%

1043 PO”의 연장선이 원 O와 만 2 D


1047 ∠BAQ=∠CAQ이고 A
2
나는 점을 D라 하고 PB”=x라고 A O
∠BAQ=∠BCQ (μ BQ에 대한 원주각)
6 T
하면
7 이므로 12
P’A”¥PD”=PB”¥PC”이므로 P C
x B O' ∠CAQ=∠BCQ
B 3 P C
6_(6+4)=x(x+7)
따라서 QC”는 세 점 A, P, C를 지나는 Q
x¤ +7x-60=0
원의 접선이므로
(x-5)(x+12)=0
QC” ¤ =QP”¥Q’A”=3_(3+12)=45
∴ x=5 (∵ x>0) 5
∴ QC”=3'5 (∵ QC”>0) 3'5

1044 CQ”를 그으면 A 1048 AB”=AC”이므로 A


△ABP와 △AQC에서 8 ∠ABC=∠ACB
x 4
∠BAP=∠QAC 6
CQ”를 그으면
P
∠ABP=∠AQC (μ AC에 대한 원주각) B P2 C ∠AQC=∠ABC (μ AC에 대한 원주각) B C
2
∴ △ABPª△AQC (AA 닮음) Q ∴ ∠AQC=∠ACB Q
AC”=x라고 하면 AB” : AQ”=AP” : AC”이므로 따라서 AC”는 세 점 C, P, Q를 지나는
8 : (6+2)=6 : x 원의 접선이므로
8_x=6_8 AC” ¤ =AP”¥AQ”=4_(4+2)=24
∴ x=6 ④ ∴ AC”=2'6 (∵ AC”>0) 2'6

1045 CQ”를 그으면 A 1049 AB”=AC”이므로 A

△ABP와 △AQC에서 12 ∠ABC=∠ACB B 6 C


10 x P
∠BAP=∠QAC BQ”를 그으면
B C 9
P
∠ABP=∠AQC`(μ AC에 대한 원주각) 7 ∠AQB=∠ACB (μ AB에 대한 원주각)
∴ △ABPª△AQC (AA 닮음) ∴ ∠ABC=∠AQB Q
Q

98 정답과 풀이
(066~103)개념RPM3-2(정답) 2014.11.24 02:58 PM 페이지99 다민 2540DPI 175LPI

● 본문 138쪽 ~ 140쪽 ●

따라서 AB”는 세 점 B, P, Q를 지나는 원의 접선이므로 따라서 AB” : AD”=A’H” : AC”이므로


AB” ¤ =AP”¥AQ” 3 : 6=A’H” : 4, 6A’H=
” 12
AP”=x라고 하면 ∴ A’H=
” 2 2
6¤ =x(x+9), x¤ +9x-36=0
(x+12)(x-3)=0
1054 △ABH에서 A
∴ x=3(∵ x>0) 3 8
AH”="√8¤ -(2'7)¤ ='ß36=6 10

CD”를 그으면 B C
2"7 H O
1050 μ AC=μ BC이므로 D △ABH와 △ADC에서
∠ABC=∠BDC ∠ABH=∠ADC (μ AC에 대한 원주각) D
5
따라서 BC”는 세 점 B, D, P를 지나는 P ∠AHB=∠ACD=90˘
A B
원의 접선이므로 3 ∴ △ABHª△ADC (AA 닮음)
BC” ¤ =CP”¥CD” C 따라서 AB” : AD”=A’H” : AC”이므로
=3_(3+5)=24 8 : AD”=6 : 10, 6AD”=80
∴ BC”=2'6 (∵ BC”>0) 2'6 40 40
∴ AD”=
3 3

1051 μ AB=μ AC이므로 A 4"3


∠ACB=∠ABC 1055 △ABD와 △AHC에서 A
∠ADB=∠ACH(μ AB에 대한 원주각) x
CQ”를 그으면 4 5
P B C
∠ABC=∠AQC (μ AC에 대한 원주각) B C ∠ABD=∠AHC=90˘ OH
2
5
∴ ∠ACB=∠AQC Q ∴ △ABDª△AHC (AA 닮음) y

따라서 AC”는 세 점 C, P, Q를 지나는 yy` D

원의 접선이므로 따라서 AB” : A’H=


” AD” : AC”이므로
AC” ¤ =AP”¥AQ” x : 4=10 : 5, 5x=40
AP”=x라고 하면 ∴ x=8 yy`
(4'3 )¤ =x(x+2), x¤ +2x-48=0 △ABD에서
(x-6)(x+8)=0
y=øπAD” ¤ -AB” ¤ ="√10¤ -8¤ ='ß36=6 yy`
∴ x=6(∵ x>0) 6
∴ x+y=8+6=14 yy`

14
1052 △ABH와 △ADC에서 10
A
단계 채점요소 배점
∠ABH=∠ADC(μ AC에 대한 원주각) 8
12
△ABDª△AHC임을 알기 40%
∠AHB=∠ACD=90˘ B O
C
H x의 값 구하기 30%
∴ △ABHª△ADC(AA 닮음) y의 값 구하기 20%
AB” : AD”=A’H” : AC”에서 D x+y의 값 구하기 10%
10 : AD”=8 : 12, 8AD”=120
∴ AD”=15
따라서 △ADC에서 1056 AB”⊥CD”이므로 PD”=PC”=6

CD”="√15¤ -12¤ ='ß81=9 9 OP”=x라 하면 PA”=8+x, PB”=8-x이므로


6_6=(8+x)(8-x)
x¤ =28 ∴ x=2'7(∵ x>0) ④
1053 CD”를 그으면 3 A
△ABH와 △ADC에서 4
B C
∠ABH=∠ADC(μ AC에 대한 원주각) H O
1057 Q’A”_6=4_3이므로 Q’A”=2
∠AHB=∠ACD=90˘ 3 PT” ¤ =P’A”¥PB”=2_(2+2+6)=20이므로
∴ △ABHª△ADC(AA 닮음) D PT”=2'5 (∵ PT”>0) ②

08. 원주각의 활용 99
(066~103)개념RPM3-2(정답) 2014.11.24 02:58 PM 페이지100 다민 2540DPI 175LPI

RPM 알피엠

1058 원 O의 반지름의 길이를 x라고 하면 1064BC”가 △ADC의 외접원의 접선 A


P’A=
” x, PB”=3x 이려면 BC” ¤ =BD”¥B’A이
” 어야 하므로
P’A”¥PB”=PC”¥PD”이므로 (3'3)¤ =3_(3+AD”)
D
x_3x=6_(6+4) 3AD”=18 3
3x¤ =60, x¤ =20 ∴ AD”=6 B C
3"3
∴ x=2'5 (∵ x>0) 2'5

1059 CO”의 연장선이 원 O와 만나는 C 1065 원 O에서 x(x+9)=PC”¥PD” yy ㉠


점을 F라고 하자. D P
9 4 E 원 O'에서 PC”¥PD”=4_(4+y) yy ㉡
A B
PO”=PC”=x라고 하면 O ㉠, ㉡에서 x(x+9)=4_(4+y)이고 y=x+2이므로
PF”=3x F x(x+9)=4(x+6), x¤ +5x-24=0
PD”¥PE”=PC”¥PF”이므로 (x+8)(x-3)=0
9_4=x_3x, x¤ =12 ∴ x=3(∵ x>0)
∴ x=2'3(∵ x>0) y=3+2=5
따라서 원 O의 반지름의 길이는 ∴ x+y=3+5=8 8
2_2'3=4'3 4'3

1 1
1066 PT”=P’T'”=
2
T’T'”= _20=10
2
1060 PC”¥PD”=PE”¥PF”이므로
” x라고 하면 PT” ¤ =P’A”¥PB”이므로
P’A=
PC”_4=3_8
10¤ =x(x+15), x¤ +15x-100=0
4PC”=24
(x+20)(x-5)=0 ∴ x=5 (∵ x>0) ③
∴ PC”=6 ③

1067 ①, ②, ④ 원 O에서 P’A”¥PB”=PE”¥PF”이고


1061 AP”=x라고 하면 PC”=10-x
원 O'에서 PE”¥PF”=PC”¥PD”이므로
BP”=PD”=4이므로 네 점 A, B, C, D가 한 원 위에 있으려면
P’A”¥PB”=PC”¥PD”
x(10-x)=4_4, x¤ -10x+16=0
⑤ ∠EAB와 ∠EFB는 원 O의 μ EB에 대한 원주각이므로
(x-2)(x-8)=0
∠EAB=∠EFB
∴ x=8 (∵ AP”>PC”) 8
따라서 옳지 않은 것은 ③이다. ③

1062 ① 5_6=5_6 1068 ① 3_(3+AB”)=4_(4+8)이므로


② 3_(3+4)+2_(2+5) ① 3AB”=39 ∴ AB”=13
③ 4_(4+11)=3_(3+17) ② PT” ¤ =4_(4+8)=48이므로 PT”=4'3 (∵ PT”>0)
④ 6_6=18_2 ③, ④, ⑤ P’A”¥PB”=PC”¥PD”이므로 네 점 A, B, C, D는 한
⑤ 3_(3+6)+5_(5+3) 원 위에 있다.
따라서 점 A, B, C, D를 꼭짓점으로 하는 사각형이 원에 내 따라서 옳지 않은 것은 ③, ⑤이다. ③, ⑤
접하지 않는 것은 ②, ⑤이다. ②, ⑤

1069 OR”⊥AB”이므로 △OBR에서


1063 ∠ABE=∠ADE=90˘이므로 네 점 A, B, D, E는 BR”=øπ(2'3 )¤ -('3 )¤ ='9=3
한 원 위에 있다. ∴ AR”=BR”=3
따라서 4_(4+11)=5_(5+ED”)이므로 따라서 PT” ¤ =PA”¥PB”이므로
5 ED”=35 PT” ¤ =4_(4+6)=40
∴ ED”=7 7 ∴ PT”=2'1å0(∵ PT”>0) 2'1å0

100 정답과 풀이
(066~103)개념RPM3-2(정답) 2014.11.24 02:58 PM 페이지101 다민 2540DPI 175LPI

● 본문 140쪽 ~ 142쪽 ●

1070 왼쪽 원에서 AC”¥CD”=CP”¥CQ” yy`㉠ 1076 AB”=AC”이므로 A 6


오른쪽 원에서 BC”¥CE”=CP”¥CQ” yy`㉡ ∠ABC=∠ACB
㉠, ㉡에서 AC”¥CD”=BC”¥CE” BE”를 그으면 O
6_2=BC”_(2+8) ∠AEB=∠ACB (μAB에 대한 원주각) P
B C
6 6 ∴ ∠ABC=∠AEB E 2
10 BC”=12 ∴ BC”=
5 5 따라서 AB”는 세 점 B, E, P를 지나는 원의 접선이므로
AB” ¤ =AP”¥AE”=(12-2)_12=120
1071 PT” ¤ =3_(3+6)=27
∴ AB”=2'3å0(∵ AB”>0) 2'3å0
∴ PT”=3'3(∵ PT”>0)
△PTA와 △PBT에서
∠PTA=∠PBT, ∠P는 공통 1077 ∠BAQ=∠CAQ A
따라서 △PTAª△PBT(AA 닮음)이므로 ∠CAQ=∠CBQ(μ CQ에 대한 원주각)
7
PA” : PT”=TA” : BT” ∴ ∠BAQ=∠CBQ
3 : 3'3=4 : BT” 따라서 BQ”는 세 점 A, B, P를 지나는 B P C
4
3BT”=12'3 ∴ BT”=4'3 ④ 원의 접선이므로 Q
BQ” ¤ =QP”¥QA”=4_(4+7)=44
1072 P’A”¥PB”=PD”¥PE”에서
∴ BQ”=2'ß11(∵ BQ”>0) 2'ß11
9_(9+AB”)=8_(8+10)
9AB”=63 ∴ AB”=7
또한 PB”¥PC”=PE”¥PF”에서 1078 TB”를 그으면 △ABT에 T
30˘
16_(16+BC”)=18_(18+6) 서 ∠ATB=90˘이므로 30˘
A 30˘ O P
16 BC”=176 ∴ BC”=11 11 1 12 B
BT”=12 sin 30˘=12_ =6
2
1073 AB”가 원 O의 지름이므로 ∠ATB=90˘ ∠BTP=∠BAT=30˘(접선과 현
△ATB에서 이 이루는 각)이고
AB”="√6¤ +(6'3)¤ ='∂144=12 ∠ABT=180˘-(90˘+30˘)=60˘이므로 △BPT에서
” x라고 하면 PT” ¤ =P’A”¥PB”이므로
P’A= 60˘=30˘+∠BPT
(6'3)¤ =x(x+12), x¤ +12x-108=0 ∴ ∠BPT=30˘
(x+18)(x-6)=0 따라서 ∠BTP=∠BPT=30˘이므로 BP”=BT”=6
∴ x=6(∵ x>0) 6 PT” ¤ =PB”¥P’A”=6_(6+12)=108
PT”=6'3 (∵ PT”>0) ⑤
1074 6¤ =4_(4+AB”)이므로 다른풀이
4AB”=20 ∴ AB”=5 AB” : A’T=
” 2 : '3, 12 : A’T=
” 2 : '3
이때 PQ”=PT”=6이므로 AQ”=6-4=2 ∴ A’T”=6'3
∴ QB”=AB”-AQ”=5-2=3 ④ △TAP에서 ∠TAP=∠TPA=30˘이므로
PT”=A’T”=6'3
1075 AQ”를 그으면 ®AM=® BM이 Q
므로
8
∠ABM=∠BAM
B
1079 ” x라고 하면
P’A= B
P
∠AQM=∠ABM x(x+4)=6_(6+10) 4 A
4 x
A
(®AM에 대한 원주각) x¤ +4x-96=0 O
M P
∴ ∠BAM=∠AQM (x+12)(x-8)=0 10 6
C
따라서 A’M”은 세 점 A, P, Q를 지나는 원의 접선이므로 ∴ x=8(∵ x>0) D

A’M” ¤ =MÚP”¥MÚQ”=4_(4+8)=48 BC”를 그으면 BC”⊥DP”이므로


∴ A’M”=4'3 (∵ A’M”>0) △BCP에서 BC”="√12¤ -6¤ ='∂108=6'3
따라서 옳지 않은 것은 ⑤이다. ⑤ △BDC에서 BD”="√10¤ +(6'3)¤ ='∂208=4'ß13

08. 원주각의 활용 101


(066~103)개념RPM3-2(정답) 2014.11.24 02:58 PM 페이지102 다민 2540DPI 175LPI

RPM 알피엠

따라서 원 O의 반지름의 길이는 2'ß13이므로 넓이는 △PDB와 △PAC에서


p_(2'ß13 )¤ =52p 52p ∠P는 공통, ∠BDP=∠CAP
이므로 △PDBª△PAC (AA 닮음) yy`

따라서 BD” : C’A=


” DP” : AP”이므로
1080 PO”의 연장선이 원 O와 만나는 P BD” : 2.5=(3+5) : 4
4 3
점을 E라 하고 반지름의 길이를 x라 하 C D 4BD”=20
면 x ∴ BD”=5 yy`
8
PE”=3+2x이므로 yy` A B 5
O
PC”¥PA”=PD”¥PE” x
단계 채점요소 배점
4_(4+8)=3_(3+2x) yy` E
PC”의 길이 구하기 40%
6x=39
△PDBª△PAC임을 보이기 40%
13 BD”의 길이 구하기
∴ x= yy` 20%
2
13
2

단계 채점요소 배점
반원 O의 반지름의 길이를 x로 놓고, PE”의 길이를 x로 나타내기 30%
1083 AP”가 ∠A의 이등분선이므로
AB” : AC”=BP” : CP”에서 8 : 6=BP” : CP”
원의 비례 관계를 이용하여 식 세우기 50%
반원 O의 반지름의 길이 구하기 20% BC”=7이므로 BP”=4, CP”=3 yy`

△ABQ와 △APC에서
∠BAQ=∠PAC
∠AQB=∠ACP (μ AB에 대한 원주각)
1081 PA”¥PB”=PC”¥PD”이므로 D
7_(7+5)=6_(6+CD”) H ∴ △ABQª△APC (AA 닮음)
4
C 2 AB” : AP”=AQ” : AC”, 8 : AP”=AQ” : 6
6CD”=48 6 O
5 ∴ AP”¥AQ”=48 yy`
∴ CD”=8 yy` P
A B
7
OH”⊥CD”이므로 즉, AP”_(AP”+PQ”)=AP” ¤ +AP”¥PQ”=48 yy ㉠

1 1 이때 AP”¥PQ”=BP”¥CP”=4_3=12이므로
CH”=DH”= CD”= _8=4
2 2 ㉠에서 AP” ¤ +12=48, AP” ¤ =36
원 O의 반지름의 길이를 r라 하면 ∴ AP”=6(∵ AP”>0) yy`

△COH에서 6
r¤ =2¤ +4¤ =20
단계 채점요소 배점
∴ r=2'5 (∵ r>0) yy`
BP”, CP”의 길이 구하기 20%
따라서 원 O의 넓이는 AP”¥AQ”의 값 구하기 20%
p_(2'5 )¤ =20p yy` AP”의 길이 구하기 60%
20p

단계 채점요소 배점
CD”의 길이 구하기 30%
원 O의 반지름의 길이 구하기 50% 1084 O’'P”, BQ”를 그으면 AP”가 원
Q
P
원 O의 넓이 구하기 20% O'의 접선이므로
O O'
AP” ¤ =AO”¥AB” A B
4
=4_(4+4)=32
1082 PC”=x라고 하면 P’A”¥PB”=PC”¥PD”이므로 ∴ AP”=4'2 (∵ AP”>0)
4_(4+6)=x(x+3), x¤ +3x-40=0 또한 △PAO'과 △QAB에서
(x+8)(x-5)=0 ∠A는 공통, ∠APO'=∠AQB=90˘
∴ x=5(∵ x>0) yy` ∴ △PAO'ª△QAB (AA 닮음)

102 정답과 풀이
(066~103)개념RPM3-2(정답) 2014.11.24 02:58 PM 페이지103 다민 2540DPI 175LPI

● 본문 143쪽 ●

즉, AP” : AQ”=A’O'” : AB”이므로 원 O'에서 PT” ¤ =PC”¥PD”=8_(8+8)=128


4'2 : AQ”=6 : 8 ∴ PT”=8'2 (∵ PT”>0)
6AQ”=32'2 ∠PTD=90˘이므로 △PTD에서
16'2 16'2 T’D”="√16¤ -(8'2)¤ ='∂128=8'2
∴ AQ”=
3 3
1 1
∴ T’O'”= T’D”= _8'2=4'2
2 2
1085 BD”를 그으면 A 8 ∴ O’O'”=O’T”+T’O'”
12
△ABD와 △AHC에서 6
=14+4'2 14+4'2
B C
∠ADB=∠ACH(μ AB에 대한 원주각) O H
∠ABD=∠AHC=90˘
∴ △ABDª△AHC (AA 닮음) D
1087 △ABC와 △ACH에서 A
따라서 AB” : A’H=
” AD” : AC”이므로
∠ABC=∠ACH 10
12 : 6=AD” : 8, 6AD”=96 O
(접선과 현이 이루는 각) 8
∴ AD”=16 D
∠ACB=∠AHC=90˘ B
1 1 C H
∴ (원 O의 반지름의 길이)= AD”= _16=8 8 ∴ △ABCª△ACH (AA 닮음)
2 2
따라서 AB” : AC”=AC” : A’H”이므로
1086 P’A”¥PB”=PC”¥PD”이므로 10 : AC”=AC” : 8, AC” ¤ =80
O’A”=OB”=x라고 하면 ∴ AC”=4'5 (∵ AC”>0)
4_(4+2x)=8_(8+8) △ACH에서 CH”="√(4'5)¤ -8¤ ='ß16=4
8x=112 이때 CH” ¤ =H’D”¥H’A이
” 므로
∴ x=14 4¤ =H’D”_8, 8H’D”=16
∴ O’T”=O’A”=14 ∴ H’D”=2 2

08. 원주각의 활용 103


(104~120)개념RPM3-2(정답) 2014.11.24 02:59 PM 페이지104 다민 2540DPI 175LPI

RPM 알 피 엠

내신만점 테스트 06 편차의 합은 항상 0이므로


-4+3+2+x+2=0
01 대푯값과 산포도 ∴ x=-3
(-4)¤ +3¤ +2¤ +(-3)¤ +2¤
01 나머지 한 개의 변량을 x라고 하면 중앙값이 46이므로 x (분산)=
5
는 45와 53 사이에 있다. 42
= =8.4
이때 중앙값은 3번째와 4번째 변량의 평균이므로 5
45+x ∴ (표준편차)='∂8.4점 ④
=46, 45+x=92
2
∴ x=47 ③
07 두 양궁 선수 A, B의 (편차)¤ 의 총합은 각각
10+7+2+4+1+0
02 (평균)= 6
10_6=60, 15_9=135
이므로 전체 15회에 대한 (편차)¤ 의 총합은
24
= =4(권) 60+135=195
6
1 195
∴ (분산)= {(10-4)¤ +(7-4)¤ +(2-4)¤ 따라서 (분산)= =13이므로
6 15
+(4-4)¤ +(1-4)¤ +(0-4)¤ } (표준편차)='ß13점 ①
74 37
= = ②
6 3
34+a+b
08 (평균)= =4
03 a, b, c, d의 평균이 7이고 표준편차가 3이므로 분산은 9 10
이다. ∴ a+b=6 yy ㉠
(a-7)¤ +(b-7)¤ +(c-7)¤ +(d-7)¤ 1
즉, =9 (분산)= {(-2)¤ +(-1)¤ +(a-4)¤ +(-3)¤ +2¤
4 10
∴ (a-7)¤ +(b-7)¤ +(c-7)¤ +(d-7)¤ =36 +5¤ +(b-4)¤ +(-2)¤ +3¤ +0¤ }
④ 1
= {(a-4)¤ +(b-4)¤ +56}
10
3_3+5_5+7_1+9_1 50 =9
04 (평균)= 10
=
10
=5(시간)
∴ (a-4)¤ +(b-4)¤ =34 yy ㉡
1 ㉠에서 b=6-a를 ㉡에 대입하면
(분산)= {(3-5)¤ _3+(5-5)¤ _5
10
(a-4)¤ +(6-a-4)¤ =34
+(7-5)¤ _1+(9-5)¤ _1}
a¤ -6a-7=0, (a+1)(a-7)=0
32
= =3.2 ∴ a=-1 또는 a=7
10
그런데 a<b이므로 a=-1, b=7 ③
∴ (표준편차)='∂3.2시간 ③

05 도수의 총합이 20이므로


5+3+a+b+4=20 09 a, b, c, d, e의 평균이 70이므로
∴ a+b=8 yy ㉠ a+b+c+d+e
=70
5
또한 평균이 80.5점이므로
따라서 a+8, b-3, c+7, d+5, e-2의 평균은
60_5+70_3+80_a+90_b+100_4
=80.5 (a+8)+(b-3)+(c+7)+(d+5)+(e-2)
20
5
80a+90b+910=1610
a+b+c+d+e+15
∴ 8a+9b=70 yy ㉡ =
5
㉠, ㉡을 연립하여 풀면 a=2, b=6
a+b+c+d+e
따라서 최빈값은 도수가 가장 큰 변량이므로 90점이다. = +3
5
④ =70+3=73 ③

104 정답과 풀이
(104~120)개념RPM3-2(정답) 2014.11.24 02:59 PM 페이지105 다민 2540DPI 175LPI

● 본문 146쪽 ~ 147쪽 ●

10 주어진 히스토그램을 도수분포표로 나타내면 다음과 같다. 14 (전체 점수의 총합)=69_7=483(점)이고


(남학생 점수의 총합)=75_4=300(점)이므로
계급(권) 계급값(권) 도수(명) (계급값)_(도수) (편차)¤ _(도수)
1이상~ 3미만 2 3 6 (-5)¤ _3 (여학생 점수의 총합)
3 이상
~ 5 미만
4 5 20 (-3)¤ _5 =(전체 점수의 총합)-(남학생 점수의 총합)
5 이상
~ 7 미만
6 14 84 (-1)¤ _14 =483-300=183(점)
7이상~ 9미만 8 8 64 1¤ _8 183
∴ (여학생의 평균)= =61(점) 61점
9이상~11미만 10 7 70 3¤ _7 3
11이상~13미만 12 3 36 5¤ _3
합계 40 280 280 15 a, b, c의 평균이 4이므로
280 a+b+c
(평균)= =7(권) =4
40 3
∴ a+b+c=12 yy ㉠ yy`
280
∴ (분산)= =7 ⑤
40 또한 a, b, c의 분산이 2이므로
(a-4)¤ +(b-4)¤ +(c-4)¤
=2
3
11 전체 학생 수는 (29+x)명이고 평균이 18권이므로 (a-4)¤ +(b-4)¤ +(c-4)¤ =6
5_7+15_18+25_x+35_4 a¤ +b¤ +c¤ -8(a+b+c)+48=6
=18
29+x ∴ a¤ +b¤ +c¤ =8(a+b+c)-42 yy ㉡ yy`
445+25x ㉠을 ㉡에 대입하면
=18, 445+25x=522+18x
29+x
a¤ +b¤ +c¤ =8_12-42=54 yy`
7x=77 ∴ x=11
54
1
(분산)= {(5-18)¤ _7+(15-18)¤ _18
40 단계 채점요소 배점
+(25-18)¤ _11+(35-18)¤ _4} 평균의 정의를 이용하여 식 세우기 3점

3040 분산의 정의를 이용하여 식 세우기 3점


= =76
40 a¤ +b¤ +c¤ 의 값 구하기 4점

∴ (표준편차)=2'ß19권 ③

16 (A 중학교의 평균)
1
= (1_1+2_4+3_6+4_2+5_2)
12 ① 평균이 낮다고 고득점자가 없는 것은 아니다. 15
② 각 반의 점수대별 학생 수는 알 수 없다. 45
= =3(개) yy`
③ 성적이 평균 이상인 학생 수는 평균과 표준편차만으로 15
는 알 수 없다. (A 중학교의 분산)
④ 표준편차가 작을수록 성적이 고르다. 1
= {(-2)¤ _1+(-1)¤ _4+0¤ _6+1¤ _2+2¤ _2}
15
⑤ A반의 표준편차가 더 크므로 분산도 더 크다.
18 6
따라서 옳은 것은 ④이다. ④ = = yy`
15 5
(B 중학교의 평균)
1
13 이 자료를 작은 값부터 크기순으로 나열하면 =
15
(1_1+2_2+3_2+4_4+5_3+6_3)
3, 4, 4, 4, 5, 5, 7, 8, 8, 9
60
= =4(개) yy`
10 10 15
따라서 중앙값은 =5번째와 +1=6번째 자료의
2 2
(B 중학교의 분산)
5+5
값인 5와 5의 평균이므로 =5(점) 1
2 = {(-3)¤ _1+(-2)¤ _2+(-1)¤ _2+0¤ _4
15
또한 가장 많이 나타난 자료의 값은 4이므로 최빈값은 4
+1¤ _3+2¤ _3}
점이다.
34
= yy`
중앙값: 5점, 최빈값:4점 15

내신만점 테스트 105


(104~120)개념RPM3-2(정답) 2014.11.24 02:59 PM 페이지106 다민 2540DPI 175LPI

RPM 알피엠

따라서 B 중학교의 분산이 A 중학교의 분산보다 더 크다. 07 BQ”=AP”=1이므로 △ABQ에서


yy` AQ”=øπAB” ¤ -BQ” ¤ ="√3¤ -1¤ ='8=2'2
6 34 ∴ PQ”=2'2-1
A 중학교의 분산: , B 중학교의 분산:
5 15 ∴ PQRS=(2'2-1)¤ =9-4'2 ①
B 중학교의 분산이 더 크다.

단계 채점요소 배점 08 가장 긴 변의 길이가 x+1이므로


A 중학교의 평균 구하기 2점 (x+1)¤ =(x-7)¤ +x¤
A 중학교의 분산 구하기 2점
x¤ -16x+48=0, (x-4)(x-12)=0
B 중학교의 평균 구하기 2점
∴ x=4 또는 x=12 yy ㉠
B 중학교의 분산 구하기 2점
분산 비교하기 2점 그런데 변의 길이는 양수이므로
x-7>0 ∴ x>7 yy ㉡
㉠, ㉡에서 x=12 ④

02 피타고라스 정리 09 삼각형의 세 변의 길이 사이의 관계에 의해


12-5<a<12+5
01 △ADC에서 AC”="√13¤ -5¤ ='∂144=12
∴ 7<a<17
△ABC에서 AB”="√9¤ +12¤ ='∂225=15 ④
그런데 a가 가장 긴 변의 길이이므로 a>12
∴ 12<a<17 yy ㉠
02 EH”="√1¤ +2¤ ='5이고 EFGH는 정사각형이므로
또한 둔각삼각형이 되려면 a¤ >5¤ +12¤
EFGH=('5)¤ =5 ②
a¤ >169
∴ a>13 yy ㉡
03 ① 9¤ +40¤ =41¤ 이므로 직각삼각형 ㉠, ㉡에서 13<a<17 ②
② 9¤ +10¤ <(12'2)¤ 이므로 둔각삼각형
③ 3¤ +(5'3)¤ >9¤ 이므로 예각삼각형
10 △ABD에서 x="√5¤ -4¤ ='9=3
④ (2'2 )¤ +6¤ <7¤ 이므로 둔각삼각형
AB” ¤ =BD”_BC”이므로
⑤ 2¤ +5¤ =('ß29)¤ 이므로 직각삼각형 ③
5¤ =4(4+CD”), 4CD”=9
9
∴ CD”=
04 AC”⊥BD”이므로 4
AB” ¤ +CD” ¤ =AD” ¤ +BC” ¤ =5¤ +8¤ =89 ① 9 ¤ 225 15
y=æ{≠ } +3¤ =Æ… =
4 16 4
15 27
05 OB”="√('2)¤ +('2)¤ ='4=2 ∴ x+y=3+
4
=
4

OC”="√2¤ +('2)¤ ='6
OD”="√('6)¤ +('2)¤ ='8=2'2
OE”="√(2'2)¤ +('2)¤ ='ß10 11 AP” ¤ +CP” ¤ =BP” ¤ +DP” ¤ 이므로
('3)¤ +2¤ =('2)¤ +DP” ¤ , DP” ¤ =5
OF”="√('ß10)¤ +('2)¤ ='ß12=2'3 ④
∴ DP”='5 (∵ DP”>0) ③

06 꼭짓점 A에서 BC”에 내린 수선의 A 6 D


발을 H라고 하면 12 ②
10
BH”=12-6=6
△ABH에서 B H C 13 정사각형의 한 변의 길이를 x라고 하면
A’H=
” "√10¤ -6¤ ='ß64=8 12
AB”="√(2x)¤ +x¤ ="ç5x¤ ='5x=4'2
따라서 △BCD에서 DC”=A’H”=8이므로 4'ß10 4'ß10
∴ x=
BD”="√8¤ +12¤ ='∂208=4'ß13 ③ 5 5

106 정답과 풀이
(104~120)개념RPM3-2(정답) 2014.11.24 02:59 PM 페이지107 다민 2540DPI 175LPI

● 본문 148쪽 ~ 150쪽 ●

14 △ABC에서 AB”="√5¤ -3¤ ='ß16=4(cm) 03 피타고라스 정리의 활용


1 1
∴ △ABF= ADEB= AB” ¤
2 2 01 정삼각형의 한 변의 길이를 a cm라고 하면
1 '3
a¤ =36'3, a¤ =144
= _4¤ =8(cm¤ ) 8 cm¤ 4
2
∴ a=12(cm) (∵ a>0)
15 꼭짓점 A, D에서 BC”에 내 A 5 cm D 따라서 한 변의 길이가 12 cm인 정사각형의 넓이는
린 수선의 발을 각각 H, H' 5 cm 5 cm
12_12=144(cm¤ ) ③
이라고 하면
B C
H’H'”=AD”=5 cm
H
11 cm
H' 02 △ABD에서 AB” : BD”=2 : '3이므로
12 : x=2 : '3
yy`
∴ x=6'3
ABCD가 등변사다리꼴이므로
△BCD에서 BC” : BD”=1 : '2이므로
1
BH”=C’H'”= _(11-5)=3(cm) yy`
y : 6'3=1 : '2
2
△ABH에서 ∴ y=3'6
A’H=
” "√5¤ -3¤ ='ß16=4(cm) yy` ∴ xy=6'3_3'6=54'2 ③
1
∴ ABCD= _(5+11)_4=32(cm¤ ) yy`
03 A(-3, 3), B(2, -3)이므로
2
32 cm¤ AB”="√{2-(-3)}¤ +√(-3-3)¤
='ß61 ③
단계 채점요소 배점
H’H'”의 길이 구하기 2점 04 직육면체의 대각선의 길이가 4'6이므로
BH”의 길이 구하기 2점
"√(2a)¤ +a¤ +a¤ =4'6, '6a=4'6
A’H의
” 길이 구하기 3점
∴ a=4 ②
ABCD의 넓이 구하기 3점

05 △ABC의 한 변의 길이를 a cm라고 하면


16 PD”=AD”=15 cm이므로 AG”=
2 2
AD”이므로 _
'3
a=4
△DPC에서 3 3 2

PC”="√15¤ -12¤ ='ß81=9(cm) yy` ∴ a=4'3(cm)

BQ”=x cm라고 하면 ∴ △ABC=


'3
_(4'3)¤ =12'3(cm¤ ) ④
4
PQ”=AQ”=(12-x) cm yy`

BP”=15-9=6(cm)이므로 06 꼭짓점 A에서 BC”에 내린 수선의 A


△BPQ에서 (12-x)¤ =x¤ +6¤ 발을 H라고 하면 6"2 cm
24x=108 △AHC에서 A’H” : AC”=1 : '2 60˘ 45˘
9 B C
∴ x= (cm) yy` 이므로 H
2
A’H” : 6'2=1 : '2
9
cm ∴ A’H”=6(cm)
2
△ABH에서 AB” : A’H=
” 2 : '3이므로
단계 채점요소 배점
AB” : 6=2 : '3
PC”의 길이 구하기 3점
∴ AB”=4'3(cm) ③
BQ”=x cm로 놓고 PQ”의 길이를 x로 나타내기 3점
BQ”의 길이 구하기 4점
07 AB”="√{2-(-1)}¤ +√(3-7)¤ ='ß25=5
BC”="√(-5-2)¤ +√(4-3)¤ ='ß50=5'2
C’A=
” "√{-1-(-5)}¤ +√(7-4)¤ ='ß25=5
따라서 AB”=C’A이 ” 고 BC” ¤ =AB” ¤ +C’A” ¤ 이므로 △ABC
는 ∠A=90˘인 직각이등변삼각형이다. ⑤

내신만점 테스트 107


(104~120)개념RPM3-2(정답) 2014.11.24 02:59 PM 페이지108 다민 2540DPI 175LPI

RPM 알피엠

08 y=x¤ +4x-5=(x+2)¤ -9 △AHC에서


∴ P(-2, -9) A’H” ¤ =(6'5)¤ -(11-x)¤
y축과 만나는 점의 좌표는 Q(0, -5) 13¤ -x¤ =(6'5)¤ -(11-x)¤
∴ PQ”="√{0-(-2)}¤ +{√-5-(-9)}¤ 22x=110
='ß20=2'5 ① ∴ x=5(cm)
∴ A’H=
” "√13¤ -5¤ ='∂144=12(cm)

09 점 A를 BC”에 대하여 대 D ∴ △ABC=


1
_11_12=66(cm¤ ) 66 cm¤
2
칭이동한 점을 A'이라고
A 4 cm
하면 AP”+DP”의 최솟값 1 cm P
C
은 A’'D”의 길이와 같다. B 12 cm 14 주어진 원뿔의 전개도를 그리면 오른쪽 6 cm
A' H
그림과 같다. x˘
점 A'에서 CD”의 연장선
에 내린 수선의 발을 H라고 하면 △DA'H에서 이때 부채꼴의 모선의 길이는
3 cm
A’'D”="√12¤ +5¤ ='∂169=13(cm) "√(3'3)¤ +3¤ ='ß36=6(cm)
따라서 AP”+DP”의 최솟값은 13 cm이다. ② 부채꼴의 중심각의 크기를 x˘라고 하면
x
2p_6_ =2p_3
360
10 정사면체의 한 모서리의 길이를 a라고 하면
∴ x˘=180˘ 180˘
'6
(높이)= a=4'6
3
∴ a=12
15 V’H=
'6
” _6=2'6 yy`
'2 3
∴ (부피)= _12‹ =144'2 ③
12 '3
C’M”= _6=3'3이고 점 H는 △ABC의 무게중심이
2
므로
11 주어진 전개도로 만들어지는 사각뿔은
6 h 1 1
오른쪽 그림과 같은 정사각뿔이다. MÚH”= C’M”= _3'3='3 yy`
3 3
밑면의 대각선의 길이는 "√6¤ +6¤ =6'2 6
1
이므로 6 ∴ △VMH= ” V’H”
_MÚH_
2
h="√6¤ -(3'2)¤ ='ß18=3'2 1
= _'3_2'6=3'2 yy`
1 2
∴ (부피)= _6¤ _3'2=36'2 ③
3 3'2

단계 채점요소 배점
12 BD”="√12¤ +12¤ =12'2 V’H의
” 길이 구하기 4점
1 1 MÚH”의 길이 구하기 4점
BG”= BD”= _12'2=6'2
2 2
△VMH의 넓이 구하기 2점
△OBG에서
OG”=øπOB” ¤ -BG” ¤ ="√12¤ -(6'2)¤
OG='ß72=6'2 16 원뿔의 전개도는 오른쪽 그림과 같 A
4
OG”=O’'G”이므로 다. yy` 8 x˘
M
O’O'”=2 OG”=2_6'2=12'2 ⑤ 부채꼴의 중심각의 크기를 x˘라고
B
하면
2
13 꼭짓점 A에서 BC”에 내린 수선의 A x
2p_8_ =2p_2
360
발을 H라 하고 B’H”=x cm라고
13 cm 6"5 cm ∴ x˘=90˘ yy`
하면
따라서 구하는 최단 거리는 B’M”의 길이와 같으므로
CH”=(11-x) cm
B x cm C B’M”="√8¤ +4¤ ='ß80=4'5 yy`
△ABH에서 A’H” ¤ =13¤ -x¤ H
(11-x) cm 4'5

108 정답과 풀이
(104~120)개념RPM3-2(정답) 2014.11.24 02:59 PM 페이지109 다민 2540DPI 175LPI

● 본문 150쪽 ~ 153쪽 ●

07 (주어진 식)={ 2 + 2 } { 2 - 2 }
단계 채점요소 배점 '3 '2 '3 '2
전개도 그리기 3점
부채꼴의 중심각의 크기 구하기 3 2 1
4점 = - = ①
4 4 4
최단 거리 구하기 3점

1
08 cos 60˘= 2 이므로
3x+18˘=60˘
∴ ∠x=14˘ ②

04 삼각비 8
09 △ABC에서 BC”= tan 30˘ =8÷ 3 =8'3
'3

01 BC”="√5¤ -3¤ ='ß16=4 △ADC에서 DC”=


8
=8÷1=8
tan 45˘
4 3 7
∴ sin A+cos A= + = ① ∴ BD”=BC”-DC”=8'3-8=8('3-1) ③
5 5 5

BC” BC”
02 △ABC에서 tan 60˘= =
2
='3 10 y=x+3에서
AB”
x=0일 때, y=3이므로 A(0, 3)
∴ BC”=2'3
y=0일 때, x=-3이므로 B(-3, 0)
BC” 2'3 '2
△DBC에서 sin 45˘= = = 따라서 직각삼각형 ABO에서
BD” BD” 2
O’A”=3, OB”=3이므로 AB”="√3¤ +3¤ =3'2
∴ BD”=2'6 ③
OB” 3 '2
∴ cos a= = = ③
AB” 3'2 2
03 △AOB에서 ∠OAB=180˘-(90˘+44˘)=46˘이므로
다른풀이
AB” AB”
sin 44˘= = =AB”=0.6947 tan a=1이므로 a=45˘ (∵ 0˘<a<90˘)
O’A” 1
'2
OB” OB” ∴ cos 45˘=
sin 46˘= = =OB”=0.7193 2
O’A” 1
∴ sin 44˘+sin 46˘=0.6947+0.7193
=1.414 ②
11 ㄱ. sin 0˘=0
'3
ㄴ. cos 0˘=1 ㄷ. sin 60˘=
2
1
04 (주어진 식)=1_1+1_0=1 ② ㄹ. cos 60˘= ㅁ. tan 60˘='3
2
∴ ㄱ<ㄹ<ㄷ<ㄴ<ㅁ ③

05 cos A= 3 인 직각삼각형 ABC


'5
B

를 그리면 오른쪽 그림과 같다.


3
12 ∠A=180˘-(90˘+50˘)=40˘이므로
A C BC”
BC”="√3¤ -('5)¤ ='4=2이므로 sin 40˘= =0.6428
'5 100
BC” 2 2'5
tan A= = = ⑤ ∴ BC”=64.28 ②
AC” '5 5

06 BC”="√3¤ +4¤ ='ß25=5 13 BC”="√5¤ +12¤ ='∂169=13


△ABCª△HBAª△HAC이므로 △ABCª△DEC(AA 닮음)이므로

∠x=∠C, ∠y=∠B ∠B=∠x

3 4 29 AB” 5 5
∴ sin x+tan y= + = ③ ∴ cos x=cos B= =
5 3 15 BC” 13 13

내신만점 테스트 109


(104~120)개념RPM3-2(정답) 2014.11.24 02:59 PM 페이지110 다민 2540DPI 175LPI

RPM 알피엠

A’H” 6 02 꼭짓점 A에서 BC”에 내린 수선의 A


14 △ABH에서 tan 45˘= = =1 ∴ BH”=6
BH” BH” 발을 H라고 하면 4
A’H” 6 '3 60˘
△AHC에서 tan 60˘= = ='3 A’H=
” 4 sin 60˘=4_ B C
CH” CH” 2 H
6
∴ CH”=2'3 =2'3
1 1
∴ △ABC= _(6+2'3)_6 BH”=4 cos 60˘=4_ =2, CH”=6-2=4
2 2
=6(3+'3) 6(3+'3) △AHC에서 AC”="√(2'3)¤ +4¤ ='ß28=2'7 ④

15 EG”="√5¤ +5¤ =5'2 A


03 4_BC”_sin 60˘=12이므로
AG”="√5¤ +5¤ +5¤ =5'3 yy` 5"3
5 '3
4_BC”_ =12, 2'3 BC”=12
△AEG는 ∠AEG=90˘인 직각삼각 x
G 2
E 5"2
형이므로 ∴ BC”=2'3(cm) ①
AE” 5 '3
sin x= = = yy`
AG” 5'3 3
EG” 5'2 '6 04 꼭짓점 A에서 BC”에 내린 수선 A
cos x= = = yy`
AG” 5'3 3 의 발을 H라고 하면 45˘ 30˘
6'2
'3 '6 '2 ∠CAH=30˘, ∠BAH=45˘
∴ sin x_cos x= _ = yy`
3 3 3 이므로 60˘
B C
H
'2 BH”=6'2 sin 45˘
3
'2
=6'2_ =6
2
단계 채점요소 배점
EG”, AG”의 길이 구하기 2점
'2
A’H=
” 6'2 cos 45˘=6'2_ =6
2
sin x의 값 구하기 3점
cos x의 값 구하기 3점 6 '3
AC”= =6÷ =4'3
cos 30˘ 2
sin x_cos x의 값 구하기 2점
'3
HC”=6 tan 30˘=6_ =2'3
3
16 0˘<A<45˘일 때, sin A>0, sin A-cos A<0 yy` ∴ (△ABC의 둘레의 길이)
∴ "√sin¤ A+"√(sin A-cos A)¤ =AB”+(BH”+HC”)+AC”

=sin A-(sin A-cos A)=cos A yy` =6'2+6+2'3+4'3


=6(1+'2+'3) ⑤
cos A

단계 채점요소 배점
0˘<A<45˘일 때, 근호 안의 식의 부호 구하기 5점 05 A’H=
” x라고 하면
주어진 식 간단히 하기 5점 △ABH에서 ∠BAH=45˘이므로
BH”=x tan 45˘=x
△AHC에서 ∠CAH=60˘이므로
CH”=x tan 60˘='3x
BH”+CH”=BC”이므로
(1+'3)x=8
05 삼각비의 활용
8
∴ x= =4('3-1) ④
x 1+'3
01 cos 46˘= 20 =0.6947 ∴ x=13.894

y
sin 46˘= =0.7193 ∴ y=14.386
20 06 A’H=
” h라고 하면 △ABH에서 ∠BAH=45˘이므로
∴ x+y=13.894+14.386=28.28 ② BH”=h tan 45˘=h

110 정답과 풀이
(104~120)개념RPM3-2(정답) 2014.11.24 02:59 PM 페이지111 다민 2540DPI 175LPI

● 본문 153쪽 ~ 155쪽 ●

△ACH에서 ∠CAH=30˘이므로 11 꼭짓점 A, D에서 BC”에 내린 수 A 3 cm D


'3 선의 발을 각각 P, Q라고 하면 4 cm 4 cm
CH”=h tan 30˘= h
3 △ABP에서 60˘
B P Q C
'3 3-'3 AP”=4 sin 60˘
BH”-CH”=BC”이므로 {1- } h=4, h=4
3 3
'3
12 AP=4_ =2'3(cm)
∴ h= =2(3+'3) ③ 2
3-'3
1
BP”=4 cos 60˘=4_ =2(cm)
2
CQ”=BP”=2 cm, PQ”=AD”=3 cm이므로
1
07 △ABC= 2 _BC”_AC”_sin (180˘-120˘)에서 BC”=2+3+2=7(cm)
1
1 '3 ∴ ABCD= _(3+7)_2'3
20'3= _10_AC”_ 2
2 2
=10'3(cm¤ ) ⑤
∴ AC”=8(cm) ①

12 OC”를 그으면 C
08 AC”를 그으면 A ∠AOC=180˘-30˘_2
ABCD 2 =120˘ 30˘
120˘
A B
=△ABC+△ACD 4 D 12 O
(부채꼴 AOC의 넓이)
1 150˘ "3
= _4_3_sin 60˘ 120
2 60˘ =p_12¤ _ =48p
B C 360
3
1 1
+ _2_'3_sin (180˘-150˘) △AOC= _12_12_sin (180˘-120˘)
2 2
1 '3 1 1 1
= _4_3_ + _2_'3_ = _12_12_
'3
=36'3
2 2 2 2 2 2
'3 7'3 ∴ (색칠한 부분의 넓이)
=3'3+ = ③
2 2
=(부채꼴 AOC의 넓이)-△AOC
=48p-36'3 ②

09 오른쪽 그림과 같이 원의 중심 O에 5 cm A
B H
서 각 꼭짓점을 연결하는 선분을 그 13 두 대각선이 이루는 예각의 크기를 a라고 하면 ABCD
45˘
으면 주어진 정팔각형은 합동인 이등 5 cm
C G 의 넓이 S는
O
변삼각형 8개로 나누어진다. 1
D F S= _5_4_sin a
∴ (정팔각형의 넓이) E 2
1 이때 0<sin a…1이므로 a=90˘일 때, ABCD의 넓이
=8△AOH=8_{ _5_5_sin 45˘}
2 가 최대가 된다.
1 '2 따라서 ABCD의 넓이의 최댓값은
=8_{ _5_5_ }
2 2
1
=50'2(cm¤ ) ③ S= _5_4_1=10(cm¤ ) 10 cm¤
2

10 두 대각의 교점을 O라고 하면 △OBC에서 14 △CAH에서 ∠CAH=90˘-30˘=60˘이므로


∠BOC=180˘-(50˘+70˘)=60˘ 1
A’H=
” 6 cos 60˘=6_ =3(m)
2
1
∴ ABCD= _8_9_sin 60˘
2 '3
CH”=6 sin 60˘=6_ =3'3(m)
2
1 '3
= _8_9_
2 2 △BCH에서 BH”=CH” tan 60˘=3'3_'3=9(m)
=18'3(cm¤ ) 18'3 cm¤ ∴ AB”=BH”+H’A”=9+3=12(m) 12 m

내신만점 테스트 111


(104~120)개념RPM3-2(정답) 2014.11.24 02:59 PM 페이지112 다민 2540DPI 175LPI

RPM 알피엠

15 △ABH에서 03 ∠PAO=∠PBO=90˘이므로 APBO에서


'2 ∠AOB=360˘-(90˘+60˘+90˘)
A’H”=600 cos 45˘=600_ =300'2(m) yy`
2 =120˘
따라서 △CAH에서 120
∴ (부채꼴 OAB의 넓이)=p_6¤ _
CH”=300'2 tan 60˘=300'2_'3=300'6(m) yy` 360
300'6 m =12p(cm¤ ) ③

단계 채점요소 배점
A’H의
” 길이 구하기 5점 04 ABCD가 원 O에 외접하므로
CH”의 길이 구하기 5점 AB”+CD”=AD”+BC”
AB”+14=8+13

16 오른쪽 그림과 같이 점 A에 ∴ AB”=7(cm) ①


A D
서 BC”에 내린 수선의 발을 4 cm
45˘
H라 하면 AH”=4 cm이므로 B H C 05 O’A를
” 그으면 △OAM에서 C
4 O’M=
” "√10¤ -8¤ ='ß36=6
AB”= 10
sin 45˘
∴ MÚD=
” OD”-O’M” O
=4'2(cm) yy` 10
=10-6=4 8
이때 ∠DAC=∠BAC(접은 각), A M B
따라서 △ADM에서
D
∠DAC=∠BCA(엇각)이므로 ∠BAC=∠BCA
AD”="√8¤ +4¤ ='ß80=4'5 ④
따라서 △ABC는 이등변삼각형이므로
BC”=AB”=4'2(cm) yy`

1
06 원 O의 중심에서 AB”, CD”에 내린 수 A 10 B
∴ △ABC= _4'2_4'2_sin 45˘ 선의 발을 각각 M, N이라고 하면
M
2 7
1 '2 AB”=CD”이므로 O’M”=O’N”이고 O
= _4'2_4'2_ N
2 2 O’M”⊥AB”이므로 A’M”=MÚB이
” 다. C D
10
=8'2(cm¤ ) yy` 1
△MOB에서 MÚB”=10_ =5이므로
8'2 cm¤ 2
O’M=
” "√7¤ -5¤ ='ß24=2'6
단계 채점요소 배점 ∴ MÚN”=2O’M”=2_2'6=4'6 ⑤
AB”의 길이 구하기 3점
BC”의 길이 구하기 3점
△ABC의 넓이 구하기 4점 07 중심 O에서 AB”에 내린 수선의 발
을 M이라고 하자.
O
큰 원에서 O’M”⊥AB”이므로 M
A C B
D
1 1
A’M=
” AB”= _18=9
2 2
작은 원에서
1 1
C’M”= CD”= _10=5
2 2
06 원과 직선
∴ AC”=A’M”-C’M”=9-5=4 ②
01 OF”⊥CD”이므로 CF”=DF”=4
또한 OE”=OF”이므로 AB”=CD”=2_4=8 ⑤
08 PB”는 원 O의 접선이므로 ∠PBO=90˘
원 O의 반지름의 길이를 r라고 하면
02 O’M”=O’N이
” 므로 AB”=AC”
△BPO에서 PO” ¤ =PB” ¤ +BO” ¤ 이므로
따라서 △ABC는 AB”=AC”인 이등변삼각형이므로
(8+r)¤ =12¤ +r¤ , 16r=80
1
∠B= (180˘-52˘)=64˘ ④ ∴ r=5 ①
2

112 정답과 풀이
(104~120)개념RPM3-2(정답) 2014.11.24 02:59 PM 페이지113 다민 2540DPI 175LPI

● 본문 155쪽 ~ 157쪽 ●

09 CD”=CE”, BD”=BF”, AE”=AF”이므로 점 C에서 D’A에


” 내린 수선의 D
6 cm
AB”+BC”+C’A”=2AF” 발을 H라고 하면 E 3 cm
6 cm
8+5+7=2AF” D’H=
” D’A-
” H’A” C
H 3 cm
∴ AF”=10 ② =6-3=3(cm)
A O B
△DHC에서

10 CF”=x라고 하면 CE”=CF”=x HC”="√9¤ -3¤ ='ß72=6'2(cm)


AD”=AF”=10-x, BD”=BE”=14-x 1
∴ ABCD= _(6+3)_6'2
2
그런데 AB”=AD”+BD”이므로
=27'2(cm¤ ) 27'2 cm¤
12=(10-x)+(14-x), 2x=12
∴ x=6 ③
15 AB”와 작은 원의 접점을 T라 하고
O’A,” OT”를 그으면 OT”⊥AB”이고 O
11 OQ”, OR”를 긋고 원 O의 반지 A A’T=
” BT”=6 cm이다. yy` r
17 cm P r'
름의 길이를 r cm라고 하면 8 cm 큰 원의 반지름의 길이를 r, 작은 원 A B
O
R 12 cm
OQCR는 정사각형이므로 의 반지름의 길이를 r'이라고 하면
T
B r cm Q C
AP”=AR”=(8-r) cm △OAT에서 O’A” ¤ -OT” ¤ =A’T” ¤ 이므로
BC”="√17¤ -8¤ ='∂225=15(cm)이므로 r¤ -r'¤ =6¤ =36 yy`
BP”=BQ”=(15-r) cm ∴ (색칠한 부분의 넓이)=pr¤ -pr'¤
그런데 AB”=AP”+BP”이므로 =p(r¤ -r'¤ )
17=(8-r)+(15-r), 2r=6 =36p(cm¤ ) yy`
∴ r=3(cm) ③ 36p cm¤

단계 채점요소 배점
12 BE”=x cm라 하면
O’A,” OT”를 긋고 A’T=
” BT”=6 cm임을 알기 2점
ABED는 원 O의 외접사각형이므로
피타고라스 정리 이용하기 4점
AB”+DE”=AD”+BE” 색칠한 부분의 넓이 구하기 4점
6+DE”=8+x
∴ DE”=x+2(cm)
16 △APO와 △BPO에서
또한 EC”=BC”-BE”=8-x(cm)
P’A”=PB” (접선의 성질)
△DEC에서 DE” ¤ =EC” ¤ +CD” ¤ 이므로
AO”=BO” (반지름)
(x+2)¤ =(8-x)¤ +6¤ , 20x=96
PO”는 공통
24
∴ x= (cm) ⑤ ∴ △APO™△BPO (SSS 합동) yy`
5
또한 △APO에서 ∠PAO=90˘이므로
P’A=
” "√10¤ -6¤ ='ß64=8
13 원의 중심을 O라고 하면 CD”의 C
1
연장선이 원의 중심을 지나므로 ∴ △APO= _8_6=24 yy`
A B 2
4 D
1 ∴ APBO=2△APO
AD”=DB”= _8=4 5
2
O =2_24=48 yy`
△AOD에서
48
DO”="√5¤ -4¤ ='9=3
∴ CD”=CO”-DO” 단계 채점요소 배점

=5-3=2 2 △APO™△BPO임을 알기 3점
△APO의 넓이 구하기 4점
APBO의 넓이 구하기 3점
14 DE”=D’A=
” 6 cm, CE”=CB”=3 cm이므로
DC”=DE”+CE”
=6+3=9(cm)

내신만점 테스트 113


(104~120)개념RPM3-2(정답) 2014.11.24 02:59 PM 페이지114 다민 2540DPI 175LPI

RPM 알피엠

07 원주각 ∠x+55˘=100˘
∴ ∠x=45˘
01 ∠BOC=2∠BAC=2_56˘=112˘ ∠DBC=∠x=45˘이므로 △FBC에서
1 ∠y=45˘+35˘=80˘
∴ ∠x= (180˘-112˘)=34˘ ④
2
∴ ∠x+∠y=45˘+80˘=125˘ ②

02 ∠ACB=∠ADB=50˘
따라서 △EBC에서 ∠DBC=80˘-50˘=30˘ ③ 09 ABQP, PQCD가 각각 원 A
E
O, O'에 내접하므로 P65˘
D
∠BAP=∠PQC O
03 BC”가 원 O의 지름이므로 ∠BAC=90˘ x O'
y
=∠PDE C
△ABC에서 ∠ABC=180˘-(35˘+90˘)=55˘ Q 115˘
=180˘-115˘=65˘ B
ABCD가 원 O에 내접하므로
∠x=2∠BAP=2_65˘=130˘
∠ABC+∠ADC=180˘
또한 ∠y=∠PDC=115˘
∴ ∠ADC=180˘-55˘=125˘ ⑤
∴ ∠x+∠y=130˘+115˘=245˘ ④

04 ③ △ABC에서 ∠ABC=180˘-(50˘+30˘)=100˘이므로
∠ABC=∠CDE
10 오른쪽 그림과 같이 μAB에 대한 원주 A
140˘ C
각을 그리면 70˘
⑤ ∠ABC=180˘-70˘=110˘이므로 O
∠ABT=∠ACB
∠ABC=∠CDE
1
따라서 ABCD가 원에 내접하는 것은 ③, ⑤이다. ∠ABT= ∠AOB B
2 T
③, ⑤
1
= _140˘=70˘ ⑤
2
05 OP”를 그으면 D
C
∠x+∠y=
1 1
∠AOP+ ∠BOP
y 11 △ECF는 CE”=CF”인 이등변삼각형이므로
2 2 x120˘ O
1
∠CEF= (180˘-60˘)=60˘
1 A B 2
= (∠AOP+∠BOP)
2
P ∠EDF=∠CEF=60˘
1 따라서 △DEF에서
= ∠AOB
2
∠DFE=180˘-(50˘+60˘)=70˘ ②
1
= _120˘=60˘ ①
2
12 ∠ATP=∠ABT=72˘이므로
06 AD”를 그으면 ∠DTP=180˘-(54˘+72˘)=54˘
A E ∴ ∠DCT=∠DTP=54˘ ③
∠CAE=∠CAD+∠DAE
=20˘+20˘ B D
=40˘ ③ 20˘ 13 ABCD가 원에 내접하므로 ∠D=∠ABP
C
△PCD에서 ∠D=180˘-(30˘+80˘)=70˘

07 △ABC에서 ∠ACB=90˘이므로 ∴ ∠ABP=70˘ 70˘

∠ABC=180˘-(90˘+30˘)=60˘
1 1 14 ∠CAB=∠CBT=60˘
∠ABD=∠CBD= ∠ABC= _60˘=30˘
2 2 ABCD는 원 O에 내접하므로
따라서 △EAB에서 ∠AED=30˘+30˘=60˘ ④ ∠ABC+∠D=180˘에서
∠ABC+105˘=180˘
08 ABCD가 원에 내접하므로 ∴ ∠ABC=75˘
∠BAC=∠BDC=55˘ 따라서 △ABC에서
∠BAD=∠DCE이므로 ∠ACB=180˘-(60˘+75˘)=45˘ 45˘

114 정답과 풀이
(104~120)개념RPM3-2(정답) 2014.11.24 02:59 PM 페이지115 다민 2540DPI 175LPI

● 본문 158쪽 ~ 161쪽 ●

15 ∠ATB=90˘, ∠ABT=∠ATP=30˘ yy` 03 PT”가 세 점 A, B, T를 지나는 원


T
△ATB에서 의 접선이므로 PT” ¤ =PA”¥PB”가 성
12
1 립해야 한다.
AT”=16 sin 30˘=16_ =8(cm) P x
2 12¤ =x(x+18) A 18 B

BT”=16 cos 30˘=16_


'3
=8'3(cm) yy` x¤ +18x-144=0, (x-6)(x+24)=0
2
∴ x=6 (∵ x>0) ③
1
∴ △ATB= _8_8'3=32'3(cm¤ ) yy`
2
32'3 cm¤ 04 PT” ¤ =P’A”¥PB”, P’T'” ¤ =P’A”¥PB”이므로
PT” ¤ =P’T'” ¤
단계 채점요소 배점
∴ PT”=P’T'”
∠ATB, ∠ABT의 크기 구하기 4점
PT” ¤ =4_(4+6)=40
A’T,” BT”의 길이 구하기 4점
△ATB의 넓이 구하기 2점
∴ PT”=2'ß10 (∵ PT”>0)
∴ T’T'”=2PT”=2_2'ß10=4'ß10 ③

16 BP”가 반원 O의 접선이므로
∠PBC=∠x yy` 05 PO”=x라고 하면 P’A”¥PB”=PC”¥PD”에서
∠BCP=90˘이므로 3_(2x+3)=4.5_4
6x=9
1 1
∠BPE= ∠BPC= (90˘-∠PBC)
2 2 ∴ x=1.5 ①
1
= (90˘-∠x) yy`
2
06 오른쪽 그림과 같이 원 O를 그리고 C
△BPE에서 ∠BPE+∠PBE=∠PEC이므로
CD”의 연장선이 원 O와 만나는 점 10
1 r D
(90˘-∠x)+∠x=180˘-115˘ 을 E라고 하면 AB”⊥CD”이므로 A 5 B
2 O
DE”=CD”=10 r-5
1
∠x=20˘ E
2 따라서 D’A”¥DB”=DC”¥DE”에서
∴ ∠x=40˘ yy` (2r-5)_5=10_10, 10r=125
40˘ ∴ r=12.5 ②

단계 채점요소 배점
∠PBC=∠x임을 알기 2점 07 원 O에서 EP”¥EQ”=E’A”¥EB”,
원 O'에서 EP”¥EQ”=EC”¥ED”이므로
∠BPE=`;2!;`(90˘-∠x)임을 알기 3점
E’A¥” EB”=EC”¥ED”
∠x의 크기 구하기 5점
12_4=3_ED”
∴ ED”=16 ③

08 O’A”=x cm라고 하면 PT” ¤ =P’A”¥PB”이므로


6¤ =2_(2+2x), 4x=32
∴ x=8(cm)
08 원주각의 활용 ∴ (원 O의 넓이)=p_8¤ =64p(cm¤ ) ④

01 OB”=x라고 하면 P’A”¥PB”=PC”¥PD”에서
4_(4+2x)=6_(6+2) 09 QA”¥QB”=QC”¥QD”에서 Q’A”_8=6_4
8x=32 ∴ x=4 ③ 8Q’A”=24
∴ Q’A”=3
02 PC”=x라고 하면 P’A”¥PC”=PB”¥PD”이어야 하므로 또한 PT” ¤ =P’A”¥PB”이므로 PT” ¤ =5_(5+3+8)=80
5_x=10_6, 5x=60 ∴ x=12 ④ ∴ PT”=4'5 (∵ PT”>0) ②

내신만점 테스트 115


(104~120)개념RPM3-2(정답) 2014.11.24 02:59 PM 페이지116 다민 2540DPI 175LPI

RPM 알피엠 ● 본문 161쪽 ●

10 QC”를 그으면 A 15 PT” ¤ =P’A”¥PB”=4_(4+5)=36


4 cm
△ABP와 △AQC에서 5 cm ∴ PT”=6(cm) (∵ PT”>0) yy`

∠BAP=∠QAC △PTA와 △PBT에서


B P
∠ABP=∠AQC C ∠PTA=∠PBT, ∠P는 공통이므로
1 cm Q
(μ AC에 대한 원주각) △PTAª△PBT (AA 닮음) yy`

∴ △ABPª△AQC (AA 닮음) 따라서 PT” : PB”=T’A” : BT”에서


AP”=x라고 하면 AB” : AQ”=AP” : AC”이므로 9
6 : 9=3 : BT”, 6BT”=27 ∴ BT”= (cm)
5 : (x+1)=x : 4, x¤ +x-20=0 2
yy`
(x+5)(x-4)=0 ∴ x=4(cm) (∵ x>0) ②
9
cm
2
11 AB”=AC”이므로 A
∠ABC=∠ACB 단계 채점요소 배점
5 cm
BQ”를 그으면 B C PT”의 길이 구하기 3점
P 4 cm
∠ACB=∠AQB △PTAª△PBT임을 알기 3점
BT”의 길이 구하기 4점
(μ AB에 대한 원주각) Q

∴ ∠ABC=∠AQB
따라서 AB”는 세 점 B, P, Q를 지나는 원의 접선이므로 16 ∠BAE=∠EAC이고
A
AB” ¤ =AP”¥AQ”=5_(5+4)=45 ∠EBC=∠EAC
∴ AB”=3'5 (cm) (∵ AB”>0) ② (μ EC에 대한 원주각) 9
이므로 ∠EBC=∠BAE yy` 6 D
B x C
12 BD”를 그으면 △ABD와 △AHC에서 5 A 4 따라서 BE”는 세 점 A, B, D를 지 E
∠ADB=∠ACH B 2 C 나는 원의 접선이다. yy`
H
(μ AB에 대한 원주각) O DE”=x라고 하면 BE” ¤ =DE”¥E’A”에서
∠ABD=∠AHC=90˘ 6¤ =x(x+9), x¤ +9x-36=0
∴ △ABDª△AHC (AA 닮음) D (x-3)(x+12)=0 ∴ x=3 (∵ x>0) yy`

따라서 AB” : A’H=


” AD” : AC”이므로 3
5 : 2=AD” : 4, 2AD”=20 ∴ AD”=10 단계 채점요소 배점
1 ∠EBC=∠BAE임을 보이기 3점
따라서 원 O의 반지름의 길이는 AD”=5이므로
2
BE”가 세 점 A, B, D를 지나는 원의 접선임을 알기 3점
(원 O의 넓이)=p_5¤ =25p ④ DE”의 길이 구하기 4점

13 ∠BAD=∠ACD이므로 AB”는 A
세 점 A, D, C를 지나는 원의 접
10
선이다. AB” ¤ =BD”¥BC”에서
B 5 C
10¤ =5_(5+CD”) D
5 CD”=75 ∴ CD”=15 15

14 AP” ¤ =AO”¥AB”에서
P Q
AP” ¤ =12_(12+12)
=288 A B
12 cm O O'
∴ AP”=12'2(cm) 6 cm 6 cm

(∵ AP”>0)
BQ”를 그으면 △APO'ª△AQB(AA 닮음)이므로
A’O'” : AB”=AP” : AQ”에서 18 : 24=12'2 : AQ”
18AQ”=288'2 ∴ AQ”=16'2(cm) 16'2 cm

116 정답과 풀이
(104~120)개념RPM3-2(정답) 2014.11.24 02:59 PM 페이지117 다민 2540DPI 175LPI

MEMO
(104~120)개념RPM3-2(정답) 2014.11.24 03:0 PM 페이지118 다민 2540DPI 175LPI

MEMO
(104~120)개념RPM3-2(정답) 2014.11.24 03:0 PM 페이지119 다민 2540DPI 175LPI

MEMO
(104~120)개념RPM3-2(정답) 2014.11.24 03:0 PM 페이지120 다민 2540DPI 175LPI

MEMO

You might also like